trust and estates outline

97
Trust and Estates - Ryan (Dukeminier) - Spring 2012 _____________________________________________________________________________________________ ___________________ CHAPTER 1. INTRODUCTION A. THE POWER TO TRANSMIT PROPERTY AT DEATH: JUSTIFICATIONS AND LIMITATIONS 1. The Right to Inherit and the Right to Convey a. Legal Institution of inheritance (Donee) – Right to Inherit (Donor) Right to Convey (transmit) Forced Succession (donor has no freedom) Testamentary freedom (donor gets to do what he wants with his property) Right to Disinherit (moving it to spouse from kids) Natural Right – (Locke) v. Civil Right - created by civil or municipal laws (Jefferson and Blackstone) b. Shaw Family Archives v. CMG Worldwide - California and Indiana postmortem right of publicity statutes recognize that an individual cannot pass by will a statutory property right that she did not possess at the time of her death. Note that: California now recognizes that publicity rights are devisable. 2. The Policy of Passing Wealth at Death a. Right v. Privilege: A decedent has the right to dispose of his or her property at death. Although states have broad authority to regulate the process states cannot completely abrogate the right. b. Public Policy debate: Some argue the power to transfer wealth at death is natural and good that it encourages one to save and promote family value while others argue that the power to transfer wealth at death perpetuates economic disparity and unfairly rewards those lucky enough to have been born to rich parents. c. Economics of Inheritance Justifications for passing wealth at death Arguments against passing wealth at death Society based on private property – this is least objectionable way to deal with property at owner’s death Transfer of fortunes perpetuates wide disparities in the distribution of wealth, concentrates inherited economic power in the hands of a few, and denies equality of opportunity to the poor. Incentive for recipients to do certain things (take care of parents, grandparents, so they will get) Danger - inherited wealth becoming the basis of enduring privilege Allows for the taking care of dependents (rather than State having to) Encourages productivity and control (work hard so children can have better life) Tends to reward chance of ficaortunate birth, rather than merit or productivity Encourages earnings and savings (b/c you know it will pass to children, so no wasting) Accumulation of wealth, rather than consuming it (which is better for society) 3. The Problem of the Dead Hand 1 | Levy

Upload: isabella-l

Post on 11-Apr-2015

151 views

Category:

Documents


5 download

DESCRIPTION

With charts and examples.

TRANSCRIPT

Page 1: Trust and Estates Outline

Trust and Estates - Ryan (Dukeminier) - Spring 2012________________________________________________________________________________________________________________CHAPTER 1. INTRODUCTION

A. THE POWER TO TRANSMIT PROPERTY AT DEATH: JUSTIFICATIONS AND LIMITATIONS 1. The Right to Inherit and the Right to Convey

a. Legal Institution of inheritance(Donee) – Right to Inherit (Donor) Right to Convey (transmit)Forced Succession (donor has no freedom) Testamentary freedom (donor gets to do what he

wants with his property)Right to Disinherit (moving it to spouse from kids)Natural Right – (Locke) v. Civil Right - created by civil or municipal laws

(Jefferson and Blackstone)b. Shaw Family Archives v. CMG Worldwide - California and Indiana postmortem right of publicity statutes recognize that

an individual cannot pass by will a statutory property right that she did not possess at the time of her death. Note that: California now recognizes that publicity rights are devisable.

2. The Policy of Passing Wealth at Death a. Right v. Privilege: A decedent has the right to dispose of his or her property at death. Although states have broad

authority to regulate the process states cannot completely abrogate the right.b. Public Policy debate: Some argue the power to transfer wealth at death is natural and good that it encourages one to

save and promote family value while others argue that the power to transfer wealth at death perpetuates economic disparity and unfairly rewards those lucky enough to have been born to rich parents.

c. Economics of Inheritance Justifications for passing wealth at death Arguments against passing wealth at deathSociety based on private property – this is least objectionable way to deal with property at owner’s death

Transfer of fortunes perpetuates wide disparities in the distribution of wealth, concentrates inherited economic power in the hands of a few, and denies equality of opportunity to the poor.

Incentive for recipients to do certain things (take care of parents, grandparents, so they will get)

Danger - inherited wealth becoming the basis of enduring privilege

Allows for the taking care of dependents (rather than State having to)Encourages productivity and control (work hard so children can have better life)

Tends to reward chance of ficaortunate birth, rather than merit or productivity

Encourages earnings and savings (b/c you know it will pass to children, so no wasting)

Accumulation of wealth, rather than consuming it (which is better for society)

3. The Problem of the Dead Hand“Dead Hand” control defined: A decedent may condition a beneficiary’s gift on the beneficiary behaving in a certain manner as long as the condition does not violate public policy.Validity: Dead hand control is generally upheld unless the condition constitutes a complete restraint on marriage, requires a beneficiary to practice a certain religion, encourages divorce or strife, or directs the destruction of property.

a. Restatement (Third) of Property: Wills and Other Donative Transfers §10.1 - The controlling consideration in determining the meaning of a donative document is the donor’s intention. The donor’s intention is given effect to the maximum extent allowed by law.

b. Shapira v. Union National Bank - Upholding and enforcing the provisions of the decedent’s will conditioning the bequests to his sons upon their marrying Jewish girls does not offend the Constitution of Ohio or the United States. The conditions contained in decedent’s will are reasonable restrictions. His unmistakable testamentary plan was for his possessions to be used to encourage the preservation of the Jewish faith. The condition did not pressure plaintiff into marriage by the reward of money because the seven year time limit is a reasonable grace period, which would give plaintiff ample time for reflection and fulfillment of the condition without constraint or oppression.

c. Restatement (Third) Trusts § 29 - A trust provision is ordinarily invalid if it tends to seriously interfere with or inhibit the exercise of freedom to marry by limiting the beneficiary’s selection of a spouse.

a) Ex: A transfers property to trust for her nephew, N. N’s interest in the trust terminates if “he should marry a person who is not of X religion.

1 | L e v y

Page 2: Trust and Estates Outline

B. TRANSFER OF THE DECEDENT’S ESTATE 1. Probate and Nonprobate Property

Probate property – property that passes through probate under the decedents will or by intestacy.

Non-probate property – property that passes outside of probate under an instrument other than a will. No court proceedings – distribution determined by the nonprobate documentCommon modes of non-probate transfer:(1) JT property, both real and personal - decedent's interests vanishes at death; survivor has the whole.(2) Life insurance - proceeds on decedent's life paid to beneficiary named in policy.(3) Contracts with payable on death provisions - Contract can be with employer, bank, etc., to distribute property to name beneficiary upon death (ex: pension plans)(4)Interests in trust - trustee holds

the property for the benefit of the named beneficiaries, then distributed per terms of trust.

2. Administration of Probate EstatesProbate is the default. The decedent must take affirmative steps (execute a valid will or create a valid non-probate instrument) to avoid having property pass through probate.

a. The Functions of Probate a) Provides evidence of transfer of title to the new ownerb) It protects creditors by providing a procedure for payment of debts c) Distributes the decedent’s property to those intended after the decedent’s creditors are paid

b. Probate Terminology and Historya) Personal representative – a (appointed) fiduciary who inventories and collects the property of the decedent;

manages and protects the property during the administration of the decedent’s estate; processes the claims of creditors and tax collectors; and distributes the property to those entitles.

1) Executor – The person named by the decedent in his will who is to carry out its terms and administer the probate estate.

2) Administrator – If the will does not name an executor, the court names a personal representative.b) Probate court - court that supervises the administration of the probate estate.c) Testate – a person dies testate if he devises real property to devisees and bequeaths personal property to

legateesd) Intestate – Real property descends to heirs, personal property is distributed to next-of-kine) At common law, a surviving spouse was not an heir- he or she had only curtesy or dower rights which are

rights to take a share of some of the decedent spouse’s property. However, today, in all states the statutes of descent and distribution name the spouse as an intestate successor whose share depends upon who else survives.

2 | L e v y

Page 3: Trust and Estates Outline

c. A Summary of Probate Procedure

(1) Opening probate - Will should first be probated in the jurisdiction where the decedent was domiciled (primary or domiciliary jurisdiction) at death.

- Ancillary administration– required to prove title if real property is located in another jurisdiction.- Letters testamentary to an executor or letters of administration to administrator- Common form or solemn form

(2) Formal versus informal probate - Formal Probate – Litigated judicial determination after notice to interested parties. UPC §3-401. Formal proceeding may become final judgments if not appealed. Court supervises the actions of PR in administering the state.- Informal Probate – After appointment, the personal representative administers the estate without going back to court. UPC §3-715.

***Estate may be closed by PR by filing a sworn statement that he has published notice to creditors, administered by the estate, paid all claims, and sent a statement and accounting to all known distributes. UPC §3-1003.

***UPC §3-301 sets forth the requirements for informal probate

(3) Barring creditors of the decedent - Non-claim statutes - Every state has a statute requiring creditors to file claims within a specified time period; claims filed thereafter are barred.- 2 Basic Forms (1) Bar claims not filed within a relatively short period after probate proceedings are begun. Generally 2-6 months (4 months under the UPC) (2) Whether or not probate proceeding are commenced, they bar claims not filed within a longer period after the decedents death, generally 1-5 yrs (1 under UPC).(4) Closing the estate

- PR is expected to complete the administration and distribute assets as promptly as possible.- PR is not discharged from fiduciary responsibility until the court grants discharge.

d. The Costs of Probate - Administrative costs – court fees, commission of the PR, attorney’s fees,, appraiser’s and guardian ad litem’s fees.

e. Is Probate Necessary? a) Statutes in almost every state permit heirs to avoid probate where the amount of property is small. See, UPC

§§ 3-1201 to 3-1204.b) Probate is not always necessary. All, or virtually all, of the property may pass free of probate. There may be

no creditors, or they may have been found and paid. Everyone may agree on who’s to get the property. Yes if you need to transfer title (clear title).

c) Even for items of personal property with evidence of ownership (document), ex. Automobile, summary procedures now exist to clear title and give transferee official recognition of his rights.

d) Common and statute provisions permit collections of small bank accounts or wage claims, or transfer of an automobile certificate of title to the decedent’s heirs, upon affidavit by their heirs.

3 | L e v y

Page 4: Trust and Estates Outline

f. Can probate be avoided? Yes, provided the property owner during life transfers all his or her property into a joint tenancy or a revocable or irrevocable trust or, in many states, executes a contract providing for distribution of contract assets to named beneficiaries on the owner’s death. (But some property is not suitable for joint tenancy or trust or is not subject to disposition by contract.) Further, even with property transferred by will or intestacy, probate is not always necessary – for example, establishment of the transferee’s title is not necessary for many items of personal property, such as furniture or personal effects b/c a purchaser will assume that the possessor has title. But it is necessary for property that requires a document of title (i.e., a car).

g. Universal Succession - (Louisiana & Europe) - the heirs or the residuary devisees succeed to the title of all the decedent's property; there is no personal rep appointed by a court. The heirs assume all decedent's obligations and liabilities

C. AN ESTATE PLANNING PROBLEM Key objectives: In advising a party about his or her estate plan, the key objectives that an estate planning attorney should keep in mind typically are (1) honoring the part’s intent, (2) avoiding estate taxes, and (3) avoiding probate.

D. PROFESSIONAL RESPONSIBILITY 1. Duties to Intended Beneficiaries Professional Responsibility: Under CL approach, the attorney owes no duty of care to, and is not in privity of contract with, intended beneficiaries. Under the modern trend, an attorney owes duty of care to intended beneficiaries, and intended beneficiaries are 3rd part beneficiaries with respect to the contract between the attorney and testator.

a. Simpson v. Calivas - Simpson brought an action for negligence and breach of contract alleging that his father's lawyer failed to draft a will that incorporated the father's actual intent to leave all his land to the intended beneficiary in fee simple. The trial court dismissed the claim and held that a lawyer who drafted a will owed no duty to Simpson. On appeal, on an issue of first impression, the court reversed and remanded the case. The court held that although there was no privity between a drafting lawyer and an intended beneficiary, the obvious foreseeability of injury to the beneficiary demanded an exception to the privity rule and that an identified beneficiary had third-party beneficiary status. The court further held that Simpson stated a cause of action simply by pleading sufficient facts to establish that an attorney negligently failed to effectuate the testator's intent as expressed to the attorney.

b. Duties to intended beneficiaries - Two types of actions can be brought by a beneficiary against a lawyer who drafted a will:

a) Tort: An attorney who drafts a will owes a duty of reasonable care to an intended beneficiary. (negligence/ malpractice suit). Simpson.

b) Contract: Where a client has contracted with an attorney to draft a will and the client has identified to whom he wishes his estate to pass, that identified beneficiary may enforce the terms of the contract as a third-party beneficiary. (breach of contract suit). Simpson.

c. Probate court jurisdiction:a) At most 10 states still follow the old rule that the lack of privity between the drafter and an intended

beneficiary prevents a malpractice action by beneficiaryb) In Simpson – validity and construction of the will were matters for the probate court to decide. Negligence of

lawyer was a matter for a court of general jurisdiction.2. Conflicts of Interest

a. Where a lawyer drafts a will, the lawyer has a duty to act in good faith towards people whom he advises about the will, even if the testator does not want the will disclosed. (In other words, he can’t misrepresent the will – but can say, I can’t show you the will.)

b. A v. B. - Mother's former law firm, which contemporaneously represented respondent father and his wife in planning their estates, sought to disclose the existence of respondent's illegitimate child to the wife. Appellant had jointly represented the husband and wife in drafting wills in which they devised their respective estates to each other. The devises created the possibility that the other spouse's issue, whether legitimate or illegitimate, ultimately would acquire the decedent's property. The court held that respondent's deliberate omission of the existence of his illegitimate child constituted a fraud on his wife. When discussing their respective estates with the firm, the couple reasonably could expect that each would disclose information material to the distribution of their estates, including the existence of children who were contingent residuary beneficiaries. Respondent breached that duty. The court concluded that appellant was permitted to inform the wife of the existence of the illegitimate child.

________________________________________________________________________________________________________________

4 | L e v y

Page 5: Trust and Estates Outline

CHAPTER 2. INTESTACY (NO WILL) AN ESTATE PLAN BY DEFAULT

A. THE BASIC SCHEME Default Rules:Testacy Defendant leaves a will that provides for the disposition of the probate property at death.Intestacy Decedent leaves no will. The probate estate passes by intestacy.Partial intestacy Decedent leaves a will that disposes of only part of the probate estate, the part of the probate estate disposed

of by the will passes by intestacy.1. Introduction – If there is no will…

a. Intestate statutes identify who takes a decedent’s probate property -- the probate property of a person of a person who dies without a will is governed by that state’s statute of descent and distribution.

if there is no valid will (total intestacy) OR to the extent that the will does not dispose of all of the property (partial intestacy)

b. Personal property – law of the state where the decedent was domiciled governs disposition.c. Real property – law of the state where real property is located governs disposition.d. Uniform Probate Code §§2-101, 2-102, 2-103, 2-105 73

2-101(a)“Intestate Succession

Any part of an estate that is not done through a will shall be passed by intestate succession in this code

2-101(b) A decedent, by will, may expressly exclude or limit the right of an individual or class of to succeed to the property of the decedent passing by intestate succession. If that person or class survives the decedent, he/they are treated as if he/they disclaimed their share of the decedent’s estate.

This also means that a person disclaimed is treated as if they had predeceased the decedent

2-102(1) “Share of Spouse”2-102(1)(A) - S; no D; no P2-102(1)(B) – S; D

The intestate share of a decedent’s surviving spouse is the entire estate if no descendent or parent of the decedent survives the decedent or all of the decedent’s surviving descendants are also descendants of the surviving spouse (who survived the decedent)

This can help avoid a guardianship to minor children of that marriage… unless the minors are step-children

2-102(2)-S; no D; P

The intestate share of the decedent’s surviving spouse is the 1st [300,000] plus 3/4ths of any balance of the intestate estate (if no descendant of the decedent survives but a parent of the decedent does)

2-102(3)-S; D and SS’s D

The intestate share of the decedent’s surviving spouse is the 1st [225,000] plus 1/2 of any balance of the intestate estate if all of the surviving descendants of the decedent are also descendants of the surviving spouse and the surviving spouse has one or more surviving descendants who are not descendants of the decedent

2-102(4)-S; D

The intestate share of the decedent’s surviving spouse is the 1st [150,000] plus ½ of any balance of the intestate estate if one or more of the decedent’s surviving descendants are not descendants of the surviving spouse

2-103“Share of Heirs other than SS”

Practiced in a minority of jurisdictions If part or all of the estate does not pass to a spouse because they are dead or non-

existent then the intestate estate goes to:o descendants by representation or if none theno to decedents parents equally or to just the surviving one but if none then too descendants of the decedent’s parents or either of them by representation but if

none then byo to surviving grandparents equally or to the one surviving one or to the

descendants of the decedent’s grandparents (paternal). The descendants shall take by representation. The other half goes to the maternal grandparents in the same way.

If there is no one from the grandparents of either side, then to the other side of the family in the same manner.

The UPC stops the line at the grandparents’ level2-105 If there is no one to take the estate under the provisions of this article, then the

5 | L e v y

Page 6: Trust and Estates Outline

“No Taker” intestate estate will pass to the statee. UPC Approach (favors SS)

Facts 1990 UPC § § 2-101 to 2-106 (rev. 2008) S; no D; no P §2-102(1)(A) all SS; D §2-102(1)(B) all S only if all D are also S’s and S’s only kids

§2-102(3) $225K + 1/2 S if D are also S’s but S has others; rest D§2-102(4) $150K + 1/2 S if one or more D is not S’s; rest D

S; no D; P §2-102(2) $300K + 3/4 S; rest Pno S; D §2-103(a)(1) all D (per capita at each generation)no S; no D; P §2-103(a)(2) all Pno S; no D; no P; B or S §2-103(a)(3) B or S (per capita at each generation)no S; no D; no P; no B or S; G or GD

§§2-103(a)(4) and (5) 1/2 paternal G; 1/2 maternal G or all to maternal or paternal if no survivors on other side – per capita at each generation

Spouse? = S , Descendant(s)? = D, Parent(s)? = P, Brother(s) or Sister(s)? = B or S, Grandparent(s)? = G, Descendant(s) of Grandparent(s)? = GDBASICALLY:

Who Takes? How much?1) Surviving Spouse 100% if no surviving issue, parents, or

100% I all decedent’s issue are also issue of surviving spouse and surviving spouse has no other issue; or

300,00 + 75% of rest if no issue but surviving parent; or 225,000 + 50% of rest if all issue are also issue of surviving spouse and surviving

spouse has other issue; or 100,000 + 50% of rest if one or more issue not issue of surviving spouse.

Any property not passing to a surviving spouse passes as follows:2) Issue Equally3) Parents Equally, or all to survivor4) Issue of Parents Equally5) Grandparents/issue 50% to paternal GP or survivor; otherwise to their issue equally;

50% to maternal GP or survivor; otherwise to their issue equally; If no surviving GPs or issue on one side, then all to the other side.

6) Escheat to the state 100%f. A typical intestate scheme (unless community property state) – less generous to SS

Who Takes? How much?1) Surviving Spouse 100% if no surviving issue, parents, or issue of parents or

50% if one child, or issue of one deceased child, or no child but parents, or issue of parents; or

33% if > one child (alive or deceased with issue).Any property not passing to a surviving spouse passes as follows:2) Issue Equally3) Parents Equally4) Issue of Parents Equally5) Grandparents Equally6) Issue of

GrandparentsEqually

7) Next-of-kin By degree of relationship8) Escheat to the state 100%Examples:Example #1: H dies intestate survived by W, and his 4 children, and his mom.

W gets 33%, and the rest is distributed equally to the children. His mom does not receive anything.

Example #2: W dies intestate survived by her children and her

Distributed equally to the children. Grandmother does not receive anything.

6 | L e v y

Page 7: Trust and Estates Outline

grandmother.g. Note: The Meaning of Heirs and the Transfer of Expectancy

a) To qualify as an heir (intestate taker), the heir must survive the decedent. Though laypeople often refer to others as their “heirs”, technically this usage is incorrect. Because an heir must survive the decedent, a person who is alive has no heirs, only “heirs apparent”.

b) Expectancy: Heir apparent’s expectation of receiving some property from their parent’s estate when a parent (the second parent) dies.

1) This is not a property interest. Because it is not, the general rule is that expectancy is not transferable.

2. Share of Surviving Spouse a. Marriage requirement: Generally, the term “spouse” assumes that the couple has gone through a valid marriage

ceremony and thus cohabitants do not qualify unless the jx recognizes common law marriage and the couple meets the requirements for common law marriage.

a) Separation: Once married, even if a couple legally separates, for inheritance purposes they continue to qualify as spouses until a court enters its final order of dissolution. Then, after dissolution, inheritance rights are gone.

b. Calculating shares:

a) Typical state statute (see chart above): SS takes 100% if no surviving issue, parents, or issue of parents, 50% if one child, or issue of one deceased child, or no child but parents, or issue of parents; or 33% if > one child (alive or deceased with issue).

b) UPC - 100% if no surviving issue, parents, or 100% I all decedent’s issue are also issue of surviving spouse and surviving spouse has no other issue; or 200,00 + 75% of rest if no issue but surviving parent; or 150,000 + 50% of rest if all issue are also issue of surviving spouse and surviving spouse has other issue; or 100,000 + 50% of rest if one or more issue not issue of surviving spouse.

c. Who is a spouse? This is usually a question of each state’s domestic relations law. Most states require a valid marriage between people of different genders.

a) Someone who has cohabited under a good faith, but mistaken, belief that he or she was married may be able to claim an intestate share as a putative spouse. See Restatement (Third) of Property § 2.2 comment.

d. Simultaneous death – A person succeeds to the property of the decedent only if the person survives the decedent for an instant of time.

a) Survival requirements: To be eligible to receive property from a decedent, a taker must “survive” the decedent. How long the taker must survive the decedent, and the burden of proof the taker has to satisfy, varies from jurisdiction to jurisdiction, and within any given jurisdiction it may vary based on the type of property involved. If claimant fails to meet the survival requirement, the claimant is treated as if he or she predeceased the decedent.

b) Two views:1) Common law : At the common law, to qualify as an heir one has to prove by a preponderance of the

evidence that he survived the decedent by a millisecond.1. Factors: Factors for determining if a person is dead: 1)no response to pain, 2)no movement

or breathing for an hour, 3)no blinking, no swallowing, and fixed and dilated pupils, 4)flats EEGs taken twice with at least a 24-hour period, and 5)absence of drug intoxication.

2) UPC §2-104 & 2-702 : Under the UPC (the modern trend), there must be clear and convincing evidence (more strict than the preponderance standard) that the heir survived the decedent by 120 hours (5 days).

e. When a person dies simultaneously with his or heir or devisee, does the heir or devisee succeed the person? ORIGINAL Uniform Simultaneous Death Act – if there is no sufficient evidence of the order of the deaths, the beneficiary is deemed to have predeceased the donor. Thus neither inherits from the other. If JT, tenancy by entirety, or community property, if both die simultaneously, ½ of the property is distributed as if A survived and ½ is distributed as if B survived.

7 | L e v y

Facts 1990 UPC § § 2-101 to 2-106 (rev. 2008) S; no D; no P §2-102(1)(A) all SS; D §2-102(1)(B) all S only if all D are also S’s and S’s only kids

§2-102(3) $225K + 1/2 S if D are also S’s but S has others;rest D§2-102(4) $150K + 1/2 S if one or more D is not S’s; rest D

S; no D; P §2-102(2) $300K + 3/4 S; rest P

Page 8: Trust and Estates Outline

f. Janus v. Tarasewicz – what constitutes as evidence? Husband and wife ingested Tylenol laced with cyanide. Husband was pronounced shortly after he was brought to the hospital. Wife was placed on life support for two days before being pronounced dead. Life insurance policy named wife primary beneficiary unless she failed to survive him, in which case his mother is the contingent beneficiary. LI proceeds were paid to wife’s to defendant, her father, and the administrator of her estate. The circuit court found otherwise and entered judgment in favor of the father. The mother and the administrator of the son's estate appealed. The court affirmed, holding that the evidence supported the circuit court's determination that the son predeceased the daughter and, accordingly, that the daughter's estate was entitled to the proceeds of the policy. Applying the USDA standard, the court found that the medical evidence demonstrated that the daughter exhibited positive signs of life after the son ceased showing such signs.

g. Determining time of death:Common Law Modern Trend

A person is dead when there is irreversible cessation of circulatory and respiratory functions.

Where circulatory or respiratory functions are maintained artificially, death occurs when there is irreversible cessation of total brain activity.

a) Two step process: 1) Did the claimant actually survive the decedent?2) Did the claimant legally survive the decedent?

b) Common Law and USDA approach : Preponderance of evidence that spouse survived by a millisecond required.

c) Clear and convincing standard: In some states, a claimant must prove by clear and convincing evidence that she survived the decedent by 120 hours.

1) UPC approach requires survival by 120 hours – 5 days. The most recent USDA requires the same.2) IF a will or non-probate instrument has a written express survival requirement, it applies.

3. Shares of Descendants Descendants defined: much broader than the term children; it refers to all of one’s offspring. “Children” is only the first generation of one’s issue.

Facts 1990 UPC § § 2-101 to 2-106 (rev. 2008) S; D §2-102(1)(B) all S only if all D are also S’s and S’s only kids

§2-102(3) $225K + 1/2 S if D are also S’s but S has others; rest D§2-102(4) $150K + 1/2 S if one or more D is not S’s; rest D

no S; D §2-103(a)(1) all D (per capita at each generation)a. Calculating shares –If a decedent’s issue takes under intestacy, they take equally. Example: Survived by 3 children? 1/3

each.a. Determining which issue take - 3 fundamental principles:

a) Issue of predeceased children take in their placeb) If a person takes, his or her issue does not.c) Absent adoption, only blood relatives qualify as heirs

1) Spouse from PRIOR marriage is not entitled to distribution of intestate property. Sons-in-law, daughters-in-law, and stepchildren do not qualify as eligible takers.

b. Three different approaches of how to calculate shares when the surviving issue are of unequal degree:Sub-issues that need to be answered: (1) at which generation should the decedent’s property be divided first? (2) at whichever generation the estate is divided first, how many shares should the estate be divided into? (3) how are the dropping shares distributed?

Summary of Distribution to IssueEnglish Per Stirpes (strict per stirpes)

Modern Per Stirpes (per capita with representation)

Per Capita at each generation

Where is the estate divided first?

First generation always

First generation live taker First generation live taker

How many shares is the estate divided into at that generation?

One share each party alive; one share each party dead but survived by issue

One share each party alive; one share each party dead but survived by issue

One share each party alive; one share each party dead but survived by issue

How to treat dropping shares?

Drop by bloodline Drop by bloodline (all descendants who are equally related take equally)

Drop by pooling (all descendants take equally – add the dropping shares & divide the total equally)

8 | L e v y

Page 9: Trust and Estates Outline

a) English per stirpes – Sometimes called “strict per stirpes, this approach divides at the first generation of descendants.

1) Those who prefer this system view the family tree in vertical terms. 2) Persons closer to the decedent may get smaller shares than those more distant3) People in the same generation may get widely differing shares; their shares depend on how prolific

their parents or grandparents were.b) Modern we stirpes (MO uses this) - The estate is divided into shares at the first generation leaving survivors.

1) Each line of descent treated equally beginning at first generation with a living taker.2) Older, “empty” generations are skipped3) Sometimes called division by “representation” or “per capita with representation.” 4) “Per capita” at the first level that has survivors, and “with representation” after that.

c) Per capita at each generation (1990 UPC § 2-106(b) ) - A few states and revised UPC § 2-106 emphasize equal treatment of each generation. This approach views the family horizontally. To use it, divide the estate in a series of steps. First, find the first generation with survivors and add the number of survivors plus the number of those who died leaving descendants who survive. Give each survivor in the older generation a share based on the total. Next, move down a generation and divide the remainder of the estate according to the same principle. How many survivors are there in this generation, and how many in this generation have died leaving descendants?

1) One share is given to each party who is alive, one share is given to each share who is dead but survived by issue

2) *UPC approach: Original version adopted the modern per stirpes while the revised version adopted the per capita at each generation.

EXAMPLES: Problem #1 (pg 90)

E F G DEnglish 1/3 1/6 1/6 1/3Modern 1/3 1/6 1/6 1/31990 UPC

2/9 2/9 2/9 1/3

D E FEnglish 1/2 1/4 1/4Modern 1/3 1/3 1/31990 UPC

1/3 1/3 1/3

D G H FEnglish 1/2 1/8 1/8 1/4Modern 1/3 1/6 1/6 1/31990 UPC

1/3 1/6 1/6 1/3

c. Note: Negative Disinheritance – assuming one does not want a particular heir to take any of his or her intestate property, what must one do to disinherit that particular heir?

a) Common law – Execute a valid will that disposes of all of the decedent’s property so that nothing passes through intestacy (thereby depriving heir of any chance of taking).

1) If the decedent’s will expresses an intent to disinherit the heir, but some or all of the decedent’s property is distributed through intestacy and the heir in question qualifies to receive a share, the heir takes despite the decedent’s clear intent.

b) Modern trend/UPC – by properly executing a will that expresses such an intent, even if some or all of the decedent’s property passes through intestacy and their heir otherwise would have qualified to take some of the property.

1) Heir is treated as if he or she predeceased the decedent. 2) “hereby disinherit” so and so and all his family etc.

4. Shares of Ancestors and Collaterals a. When a decedent dies intestate, his or her property is distributed first to his or her immediate family. If there is no

surviving spouse or issue, the property flows up to the decedent’s ancestors and collateral relatives.

9 | L e v y

A(D)

B C

E F G

D

A(D)

B C

D E F

A(D)

B C

D E F

G H

Page 10: Trust and Estates Outline

a) Rule of priority:1) Spouse2) Issue3) If no spouse and no issue:

i. Parentsii. If no spouse, no issue, and no parents:

1. Siblings2. If no siblings:

a. Descendant’s of siblings (i.e., nieces and nephews of decedent)b. If no descendant’s of siblings: jurisdictions differ.

b) Collateral relatives – The decedent, the decedent’s spouse, and the decedent’s issue are the decedent’s immediate family. All of the decedent’s other relatives are technically called “collateral relatives”.

c) First-line collaterals – Descendants of the decedent’s parents, other than the decedent and the decedent’s descendants.

1) No parent: If the decedent is not survived by a spouse, descendant, or parent, intestate property passes to brothers and sisters and their descendants. If a brother or sister is dead, the same representation rules from above apply. i.e., use strict per stirpes, modern per stirpes, and/or per capita at each generation to see what each sibling gets

d) Second-line collaterals – Descendants of the decedent’s grandparents other than the decedent’s parents and their descendants.

e) If the decedent is not survived by a spouse, descendant, or parent, in all jurisdictions intestate property passes to brothers and sisters and their descendants.

b. Two schemes used when there are no first-line collaterals:a) Parentelic system – intestate estate passes to grandparents and their descendants, and if none to great-

grandparents and their descendants, and if none to great-great-grandparents and their descendants, and so on down each line (parantela) descended from an ancestor until an heir is found.

b) Degree-of-relationship system – the intestate estate passes to the closest of kin, counting degrees of kinship. To ascertain the degree of relationship of the decedent to the claimant you count the steps (counting one for each generation) up from the decedent to the nearest common ancestor of the decedent and the claimant, and then you count the steps down to the claimant from the common ancestor. The total number of steps is the degree of relationship

c. Escheat: If the intestate dies without an heir who is entitled to take under intestacy, the intestate’s property escheats to the state.

d. Half-Bloods a) UPC 2-107 and Majority of States – a relative of half-blood is treated the same as relative of whole blood.b) Common law – only whole-blooded relatives are entitles to inherit.

e. SHARES OF ANCESTORS AND COLLATERALS SUMMARY:Facts 1990 UPC § § 2-101 to 2-106 (rev. 2008) S; no D; P §2-102(2) $300K + 3/4 S; rest Pno S; no D; P §2-103(a)(2) all Pno S; no D; no P; B or S §2-103(a)(3) B or S (per capita at each generation)no S; no D; no P; no B or S; G or GD §§2-103(a)(4) and (5) 1/2 paternal G; 1/2 maternal G or all to maternal or

paternal if no survivors on other side – per capita at each generationno S; no D; no P; no B or S; no G or GD

§2-103(b) stepchildren§2-105 escheat to state; therefore no “laughing heirs”; note: no great grandparents

Share of First-Line Collaterals - English Per Stirpes/ Share of First-Line Collaterals - UPC §§ 2-103(a)(3), 2Modern Per Stirpes 106(c) (per capita at each generation)

10 | L e v y

Page 11: Trust and Estates Outline

B. TRANSFERS TO CHILDREN 1. Meaning of Children

a. Adopted Children a) Straight adoption: You have a straight adoption from two natural parents to two adoptive parents. In that

situation, the adopted kid, for all purposes, becomes the “natural child” of the adopted parents. It has no rights toward the natural parents.

b) Step-parent adoption: You have a kid whose natural parent marries someone else and the kid becomes adopted by the new step-father/ step-mom. What typically happens is that the kid can inherit either from natural parent or from the adoptive parent.

c) Hall v. Vallandingham - The court held that because Md. Code Ann., Est. & Trusts § 1-207(a) eliminated the children's right to inherit from the natural parent, it concomitantly abrogated their right to inherit through the natural parent by way of representation. Court affirmed the circuit court's decision denying the adult children entitlement to inherit from the natural uncle's estate after their adoption by their widowed mother's new husband. Applied the general rule - the adopting parent steps into the shoes of a natural parent of the same gender, and the parent-child relationship with the natural parent is completely severed. Under the modern trend/UPC approach, because the adoption was by the stepparent (the spouse of one of the natural parents), the children can still inherit from and through the natural parent of the same gender as the adopting stepparent.

1) Because an adopted child has no right to inherit from the estate of a natural parent who dies intestate, it follows that the same child may not inherit through the natural parents by way of representation. Under UPC, children could have taken

2) What if instead Elizabeth died with Earl and Jim surviving – would her children take?1. MD/MO/ – statute pg 98 - Yes can inherit from custodial natural parent2. UPC – yes per 2-119(b)

d) Restatement (Third) of Property: Wills and Other Donative Transfers – 1) A nonmarital child is just as much a child of his or her genetic parents as a marital child. Thus, except

as circumstances warrant a different result, a child is the child of his or her genetic parents, whether or not they are married to each other. Exceptions to this rule include, in certain circumstances, a child who is adopted out of the family or who results from assisted reproductive technologies.

2) Adopted child's relationship to adoptive parent or parents. An adopted child is treated as a child of his or her adoptive parent or parents. Most intestacy statutes, including the Original and Revised UPC, treat an adopted child as a full member of the child's adoptive family.

e) Note: Inheritance rights of an adopted child vary considerably from state. 1) In Hall, an adopted child inherits only from adoptive parents and their relatives; 2) In others, an adopted child inherits from both adoptive parents and genetic parents and their

relatives; 3) In still others as provided by UPC 2-114(b)(1990), an adopted child inherits from adoptive relatives

and also from genetic relatives, if the child is adopted by a step-parent.

11 | L e v y

Parents

A(Decedent)

D

G(1/12)

I

E

O(1/12)

CB(1/4)

F(1/4)

H

L(1/36)

M(1/36)

N(1/36)

J(1/8)

K

P(1/8)

Parents

A(Decedent)

D

G(1/8)

I

E

O(3/40)

CB(1/4)

F(1/8)

H

L(3/40)

M(3/40)

N(3/40)

J(1/8)

K

P(3/40)

Page 12: Trust and Estates Outline

4) 2008 amendments to the UPC – revised the key determination to whether there is a parent-child relationship. If the relationship exists: “The parent is a parent of the child and the child is the child of the parent for purpose of intestate succession”.

1. §2-116 - Inheritance rights flow if there is a Parent-Child Relationship (“P-C”) 2. §2-118(a) - P-C exists between adopted child and adopted parents3. §2-119(a) - P-C does not exist between adopted child and genetic parents except

a. §2-119 (b) - Stepparent Adoptions˗ (1): P-C exists between adopted child and custodial genetic parent˗ (2): P-C exists between adopted child and noncustodial genetic parent, but

only for the purpose of the right of the adoptee or descendant of adoptee to inherit from or through the noncustodial genetic parent

f) Adult adoption - Most states treat just like regular adoption; typically done to avoid will contest. But, adoption of an adult for the purpose of bringing that person under the provisions of a preexisting testamentary instrument, when he clearly was not intended to be so covered, should not be permitted

1) Minary v. Citizens Fidelity Bank & Trust Co - Decedent's surviving heirs were eligible to receive the remaining portion of a testamentary trust upon its termination. However, one of decedent's children adopted his own wife prior to his death. Therefore, the question presented for review was whether the wife was eligible to inherit under the provisions of decedent's will as an "heir." On appeal, the court reversed the judgment of the trial court allowing the wife to inherit under the will as an heir of decedent. Instead, the court held that when an adult was adopted for the sole purpose of making him or her an heir and claimant to the estate of an ancestor under the terms of a testamentary instrument known and in existence at the time of the adoption, the practice was an act of subterfuge which thwarted the intent of the ancestor whose property was being distributed and cheated the rightful heirs. Accordingly, the court concluded that the wife was not able to inherit under the will as an heir of decedent.

As a general rule, adopted adults are treated the same as adopted children for inheritance purposes.g) Note: Doris Duke and Adoptive Parent’s Remorse - Adoption, unlike marriage, is not revocable if the

relationship turns sour.h) Equitable adoption applies where the natural parents transfer custody of their child to a couple (or individual)

who promises to adopt the child but who then fails to complete the proper paperwork to adopt the child legally. Equity treats the child as a child of the adoptive parent for purposes of distributing the adoptive parent’s intestate property. Requirements:

1) Must be an agreement between natural parents and adoptive parents to adopt a child: 1. O’Neal v. Wilkes – modern trend. Appellant child's mother died when she was a child, and

her father never legitimized or supported appellant. Appellant's aunt allowed the decedent to raise appellant and the decedent referred to appellant as his child. Appellant brought a virtual adoption action to obtain a share of the decedent's property after he died. The trial court entered judgment notwithstanding the verdict in favor of appellee executor. On appeal, the court affirmed. The court held that in order for a contract for adoption to be valid, it must be made between persons competent to contract for the disposition of the child. The court held that the aunt, who had physical custody of appellant but was never appointed legal custodian or guardian of appellant, had no authority to contract for appellant's adoption to decedent. The court also recognized that the consent of appellant's father was unnecessary because he had never legitimized nor supported the child.

2) The natural parents must fully perform (turn over child) to adoptive parents3) The child fully performs by moving in with the adoptive parents4) The adoptive parents die intestate.

i) UPC § 2-705: Class Gifts Construed to Accord with Intestate Succession; Exceptions – Adopted Children1) § 2-705(b) (General Rule): Construe class gifts in accordance with rules for intestacy regarding

parent-child relationships (§§ 2-116, 2-118, 2-119)2) Exception: § 2-705(f): Transferor Not Adoptive Parent

1. an adoptee is not considered a child of the adoptive parent unless:a. Adoption took place while adoptee was a minor (under 18) ORb. Adoptive parent was the adoptee’s stepparent or foster parent ORc. Adoptive parent functioned as parent while adoptee was a minor (under 18)

12 | L e v y

Page 13: Trust and Estates Outline

3) UPC § 2-705(f): G’s will creates a trust: “Income to my daughter, A, for life, remainder to A’s descendants, by representation”

b. Posthumous Children - a child who is conceived before, but born after, her father’s death. They are treated as being a child from the time of conception.

a) Rebuttable presumption of parenthood if born w/in 280 days after parent’s deathb) Child born more than 280 days after parent’s death has burden of proving parenthoodc) Children born from reproductive technology

1) Policy Issues:1. Did decedent intend to have these children inherent form the estate2. Interest in closing the estate w/out child coming forward 20 yrs. later

2) Majority approach – Sperm donor is not treated as father. Father = husband of the motherc. Nonmarital Children

a) Common law – a child born out of wedlock could not inherit from or through the child.b) Modern trend/UPC approach – A child has a parent-child relationship with both natural/genetic parents

regardless of their marital status. UPC §2-117. However inheritance from and through the natural father typically requires proof of paternity.

1) Majority of states permit inheritance of father where there is:1. Evidence of a subsequent marriage2. Acceptance by the father3. By adjudication during the life of the father or4. By clear and convincing evidence after death.

d. Reproductive Technology and New Forms of Parentage a) Woodward v. Commissioner of Social Security - Plaintiff wife gave birth to twin girls. The children were

conceived through artificial insemination using her deceased husband's preserved semen. Later, the wife applied for Social Security survivor benefits. Defendant commission denied benefits, concluding, among other things, the children did not qualify for benefits because they were not entitled to inherit from the husband under the Massachusetts intestacy and paternity laws. The court held that in certain circumstances a child resulting from posthumous reproduction may enjoy the inheritance rights. These limited circumstances include, where it is demonstrated: (1) a genetic relationship exists between the child and the decedent, and (2) the decedent affirmatively consented to posthumous conception and to support of any resulting child.

b) In re Martin B. - The court found it undisputed that the infants, although conceived after the son's death were the products of his semen. Although it could not be said that the grantor contemplated that his "issue" or "descendants" would include children who were conceived after his son's death, the trust instruments provided that the trust fund would benefit his sons and their families equally. Because the grantor intended all members of his bloodline to receive their share, and the infants were "issue" and "descendants" for all trust purposes.

c) Surrogate Motherhood and Married Couplesd) Assisted Reproduction and Same-Sex Couples e) Posthumous Conception & Intestacy – Alts. (pg. 125)

1) Woodward (MA) - Consent (“clearly and unequivocally”), time limit?, notice?2) Restatement (3rd) Property - § 2.5 - Inferred consent, “reasonable” time limit, no notice3) LA 9:391.1 - Inferred consent (if SS), born w/in 3 years, no notice4) UPA § 707- Written consent, no time limit, no notice5) Fl § 742.17 - Consent in will6) UPC § 2-120 - Consent (written or C &C), in utero w/in 3 years, no notice7) CA Prob. Code § 249.5 - Written consent, in utero w/in 2 years, notice w/in 4 month\

2. Advancements - If a child wishes to share in the intestate distribution of a deceased parent’s estate, the child must permit the administrator to include in the determination of the distributive shares the value of any property that the decedent, while living, gave the child by way of an advancement.

a. Common law - anytime gift by the decedent to a child presumed to be an advancement – in effect, a prepayment – of the child’s intestate share.

a) The doctrine is based on an assumption that the parent would want an equal distribution of assets among the children.

b) If child predeceases parent: When a parent makes an advancement to the child and the child predeceases the parent, the amount of advancement is deducted from the shares of the child’s descendants if other children of the parent survive.

13 | L e v y

Page 14: Trust and Estates Outline

b. UPC- lifetime gifts are not treated as advancements, unless decedent contemporaneously put in writing that gift was an advancement

a) UPC §2-109 - Under the UPC, inter vivos gifts do not count against an heir’s share of the decedent’s intestate estate unless there is:

1) a writing by the advancer (person who gave the advancement) contemporaneous with the inter vivos gift expressing such an intent OR

2) a writing by the advancee (person who received the advancement) acknowledging such an intent.1. If child predeceases parent, 2-109(c): If the recipient of the property predeceases the

decedent, the property is not taken into account in computing the division of the intestate estate, unless the decedent’s contemporaneous writing provides otherwise.

c. How it works: Where there is an advancement, the amount of the advancement is added back into the decedent’s intestate estate, and then each share of the hotchpot is determined. In distributing the decedent’s intestate property, an heir who receives an advancement has the value of the advancement credited against his or her share (of the hotchpot amount). If gift is treated as advancement then → how do you calculate shares at death?

a) T has 3 surviving children, no spouse. Estate at time of death is valued at $500k. While T was alive, child A received $100k.

1) Regular distribution → advancement deducted from the inheritance.1. $500k divided by 3 - $167k is what it would be without advancement. 2. The $100k is deducted from this, so A gets $67k.

2) Hotchpot distribution → A gives back advancement first, then calculated1. $500k + $100k (advancement) = $600k estate created2. The $600k divided by 3 = $200k each3. Then deduct advancement $100k → A would get $100k under hotchpot.

b) Sometimes it’s not worth it to take part in hotchpot. For example, if estate at death worth $125k, and A rec’d $100k during lifetime:

1) No hotchpot → $125k/3 = $41k. Nothing extra, A already took. But A has the $100k.2) Hotchpot → $125k + $100k = $225k / 3 = $75k (instead of $100k under reg distribution)

3. Guardianship and Conservatorship of MinorsUnder the intestate distribution scheme, if a decedent dies intestate and is survived by issue, there is a good chance that some property may be distributed to a minor. (1) If minor’s parent die and do not appoint a guardian in the will, the court will appoint one for them. Guardianship terminates when the child becomes of age; dies; or is adopted. (2) If designated by will, courts look with deep suspicion. Must have sufficient recording requirements.

a. Guardian of the Person a) A guardian of a person has responsibility for the minor child’s custody and care. b) Guardanship of the person for a minor is covered by UPC §§5-201 – 5-210

b. Property Management Options:Guardianship Conservatorship UTMA or UGMA

CustodianshipTrusteeship

Need Will? N N N YExpense? Highest lowest Depends Ct. Super- vision? Y – full supervision

(appointment, management, accountings)

Y – partial supervision (appointment, accounting)

N N

Title? N Y Y YFlexibility? lowest highestType of Distr.? Income Only Income and/or

PrincipalIncome and/or Principal

Income and/or Principal

Termination? Age of Majority Age of Majority Age of Majority No required termination date

a) Guardianship of the property – Guardian has a duty of preserving specific property left to the minor and delivering it to the ward t age 18, unless the court approves a sale, lease or mortgage.

1) Modern trend - modified guardianship and transformed it into a conservatorship.b) Conservatorship – Conservator takes title as trustee for the minor and has all the powers a trustee would

have over the property. Appointment and supervision by the court is required

14 | L e v y

Page 15: Trust and Estates Outline

c) Custodianship – a custodian has discretionary power to use the property for the benefit of the minor, as the custodian deems appropriate, without court approval

1) Custodian – person who is given a property to hold for the benefit of a minor under the UTMA (Uniform Transfers to Minors Act).

2) Custodian has no duty to account to the court, only to the minor when he turns 21. d) Trusts – Holding and managing a minor’s property through a trust. The terms of the trust control the scope of

the trustee’s power of the property, the trustee’s ability to use the principal and/or income for the benefit of the child, the trustee’s duty to account, and when the trust is to terminate and the property to be distributed.

C. BARS TO SUCCESSION 1. Homicide Where a party who otherwise is entitled to take from a decedent kills the decedent, the equitable principle that one should not profit from one’s wrongdoing argues against permitting the killer rom taking.

a. In re Estate of Mahoney - The wife was convicted of manslaughter for killing her husband. The husband died intestate and the probate court determined that it would be unjust to allow her to profit from her husband’s death and decreed the residue of his estate to his parents rather than to her as required by Vermont. The wife challenged the decision and the court reversed. There was no statutory basis for depriving the wife of her rights under Ch. 14, § 551(2) and the probate court lacked the equitable power to stray from the statutory mandate. Court said that it would be inequitable to permit the wife to profit from her own wrongdoing and adopted the constructive trust approach to the issue to ensure that the killer did not profit from her own wrongdoing.

b. Three approaches:a) Legal title still passes to slayer - Person who committed crime already penalized (criminal prosecution) - why

should slayer be penalized again?1) Policy: Devolution of the property of a decedent is controlled entirely by the statutes of descent and

distribution; further, that denial of the inheritance to the slayer b/c of his crime would be imposing an additional punishment for his crime not provided by statute, and would not violate the constitutional provision against corruption of blood

b) Legal title does not pass to slayer - Should not be able to benefit from wrongful conduct.1) Some jurisdictions will even bar next of kin (heir of slayer)2) The legal title will not pass to the slayer b/c of the equitable principle that no one should be

permitted to profit by his own fraud, or take advantage and profit as a result of his own wrong or crime. Criticisms: this is unwarranted judicial legislation.

c) Legal title passes to slayer but holds as a constructive trust for heirs or next of kin of decedent (In re Estate of Mahoney)

1) A constructive trust is an equitable remedy a court imposes to prevent unjust enrichment.” 2) Policy: B/c of the unconscionable mode by which the property is acquired by the slayer, equity treats

him as a constructive trustee and compels him to convey the property to the heirs or next of kin of the deceased.

Exceptions: (Not all jurisdictions have these exceptions)o Insanity – heir was insane when they killed decedento involuntary manslaughter – no intent to kill

c. UPC: Slayer also barred from nonprobate transfers - killer cannot profit from his wrong. Killer is treated as having disclaimed the property, and disclaimant is treated as having predeceased the decedent. Some jurisdictions permit the next of kin to take, others do not.

a) §2-803 (g) – preponderance of evidence standard2. DisclaimerSometimes an heir or devisee will decline to take the property. If a party disclaims, as a general rule, the party disclaiming is treated as if he or she predeceased the decedent. Pay careful attention to a disclaimer statute

a. Common Law – cannot disclaim; if heir refuses to accept, law treats it as passing through disclaimer then to next intestate successor.

a) Intestacy – treated as if disclaimant received property and transferred it to taker in default D HEIR#1 HEIR#2

b) Testacy – treated as if disclaimant predeceased testator, property treated as passing directly from decedent to taker in default D HEIR #1 HEIR#2

15 | L e v y

0

Page 16: Trust and Estates Outline

b. Modern trend/UPC – allow disclaimer; disclaimer treated as if he died before decedentc. Benefits of disclaiming:

a) Redistribute property - Disclaimers can be used to adjust who takes and how much they take after the death of the decedent.

b) Saving Estate taxes – If one accepts the property and then gives it to the next taker in line, gift taxes consequences to the transfer may result. If, however, one disclaims and the legal effect is simply to pass the property in question to the next taker in line, the disclaimer has no gift tax consequences.

c) Avoiding creditors – As a general rule, creditors are entitled to reach any transferable property that the debtor holds. If an heir or devisee is facing creditor claims, such that any inheritance or devise would, for all practical purposes, go directly to the credito

1) But when the creditor is the IRS, you cannot disclaim.1. Drye v. United States - testator dies, leaves property to son, Ron. Ron tries to disclaim, so his

daughter would get the money and not his creditors (IRS). Court said the IRS can reach the disclaimed assets because Ron ultimately had control of the property.

d. What is the effect of a disclaimer?a) State Statutory Reform - Intestacy & Testacy – treated as if disclaimant predeceased testator, property

treated as passing directly from decedent to taker in default1) NO gift tax consequences to disclaimant (if done within 9 months pursuant to I.R.C. § 2518)2) Creditor protection – generally, disclaimant’s creditors cannot get at the property

1. Exceptions: Federal tax lien, Medicaid, Bankruptcy (post-petition)2. Others? Possibly

e. Disclaimers to Qualify for Medicaid - A Medicaid recipient cannot disclaim his inheritance. a) Policy reason: public aid is intended only to those that truly need it. If a recipient is set to inherit money

whereby he could pay for his medical treatment, it is unfair for him to disclaim it.b) When you disclaim property, the disclaimer allows you to avoid tax liability you could have inherited, and

avoid your personal creditors reach that property, but can’t avoid notifying Medicare/ Medicaid

________________________________________________________________________________________________________________CHAPTER 3. WILLS: CAPACITY AND CONTESTS 4 requirements to a valid will: legal capacity, testamentary capacity, testamentary intent, and compliance with statutory formalities.

A. MENTAL CAPACITY 1. The Test of Mental Capacity

a. To be competent to make a will, Testator must be:a) adult (age of majority) b) capable of knowing and understanding in a general way

1) the nature and extent of his or her property2) the natural objects of his or her bounty3) the disposition that he or she is making of that property

c) capable of relating these elements to one another and forming an orderly desire regarding the disposition of the property

b. Observe that the test is one of capability, not actual knowledge. Nor must the testator be of average intelligence. Testator may have a mistaken belief (i.e., that his son is dead)

c. In re Estate of Washburn - Will devised to his friend one of this properties, his daughter another, and his interest in the 3rd to his granddaughter. He left his grandson & others $1 each. Will duly executed, with notary & witnesses signing. Will contested b/c of mental capacity. Many witnesses testified that he was of unsound mind, including those present at the signing of the will, and gave many examples of his weird behavior. Holding: not enough evidence of unsound mind. The legal presumption is always in favor of sanity, especially after attestation by subscribing witnesses, for it is the duty of the subscribing witnesses to be satisfied of the testator's sanity before they subscribe the instrument. The notary & witnesses has subscribed to the will, but later testified that he was of unsound mind at that time. Not enough evidence offered to overcome presumption of sanity that was created when they subscribed. Testamentary capacity cannot be destroyed by showing a few isolated acts, foibles, idiosyncrasies, moral or mental irregularities or departures from the normal unless they directly bear upon and have influenced the testamentary act. The only evidence was testimony that he was a drunk, kept to himself, lived alone and in a messy place, and did weird things (like pinning garbage to bushes & showing the neighbor his "roses"). These acts had no bearing on his ability to create the will. He knew the objects of his bounty, the property he owned, etc.

16 | L e v y

Page 17: Trust and Estates Outline

d. Wilson v. Lane – Majority approach is that once a proponent offers prima facie proof that a will was duly executed, it created a rebuttable presumption the testator had testament capacity and the burden is on the contestant to prove a lack of testament capacity.

e. Importance of BOP in Testamentary Capacity Cases: a) Minority Rule: Ultimate BOP on PROPONENT to show testamentary capacity by preponderance of the

evidenceb) Majority Rule (UPC § 3-407): Ultimate BOP on CONTESTANT to show lack of capacity by a preponderance of

the evidencef. Capacity Thresholds

Lesser Capacity Required Greater Capacity RequiredForm of Legal Obligation Marriage Will Irrevocable Lifetime Gift;

Contract; DeedCompeting Policies Protection of Property

v. Individual AutonomyProtection of Property v. Testamentary Freedom

Protection of Property v. Freedom of Contract

g. Why Require Mental Capacity? Protect Testator, Protect Testator’s Family, Protect the Legal Institution of Testacya) A will should be given effect only if it represents the testator's true intentb) A mentally incompetent man or woman is not defined as a "person."c) The law requires mental capacity to protect the decedent's family. d) To a large extent the public acceptance of law rests upon a belief that legal institutions, including inheritance,

are legitimate, and legitimacy cannot exist unless decisions are reasoned. e) Assures a sane person that the disposition the person desires will be carried out even if the person later

becomes insane and makes another will. f) May protect society at large from irrational acts. (but doesn’t protect society from a sane person acting

irrationally)g) May protect a senile or incompetent testator from exploitation by cunning persons.

2. Insane Delusion – a false perception of reality that the testator adheres to against all reason and evidence to the contrary.

Majority – Rational person test - if a rational person in the testator’s situation could not have reached the same conclusion, the belief is insane delusion.Minority – Any factual basis test – If there is any factual basis to supports the testator’s belief, it is not an insane delusion.

a. A person may have sufficient mental capacity generally to execute a will but may be suffering from an insane delusion so as to cause a particular provision in a will (or the whole will) to fail for lack of testamentary capacity (depending on how much of the will was caused by the insane delusion).

a) As a general rule, courts do not correct mistake. Delusion vs. mistake: An insane delusion is a belief not susceptible to correction by presenting the testator with evidence indicating the falsity of the belief. A mistake is susceptible to correction if the testator is told the truth.

b) Courts do not reform or invalidate wills because of mistake, whereas they do invalidate wills resulting from an insane delusion. Example: Suppose that testator falsely believes that her son has been killed and therefore executes a will leaving all her property to her daughter. In fact the son is alive. The testator is mistaken, not under an insane delusion, and the will is entitled to probate.

b. Nexus between delusion and will:a) Common law – no nexus required between insane delusion and the will because a mind that was unsound on

one subject was considered contaminated to the point that the testator’s entire mind was unfit to create a valid disposition.

b) Modern law – a connection must exist between the testator’s insane delusion and the property disposition in the will before the court may invalidate the will.

c. Approach: Was there an insane delusion?a) Yes -> Did the insane delusion affect a disposition in the will (i.e. causation)? MAJORITY: must prove

causation – (insane delusion materially affected or influenced the will) MINORITY: presume causation (insane delusion + unnatural disposition that might have been caused by insane delusion)

1) No -> Will is valid (assuming all other requirements met)2) Yes -> Affected portion of will is invalid

b) No -> Will is valid (assuming all other requirements met)d. In re Strittmater - Will gave everything to the National Woman's party (NWP); decedent had worked as volunteer for

NWP for long time, it was her whole life. Doctor testified that she had schizophrenia. She never married, and lived with her parents until they died. She was devoted to her parents when they were alive, but after they died, she wrote

17 | L e v y

Page 18: Trust and Estates Outline

nasty things about her parents, especially her father. She hated men, wished they would all be killed and looked forward for the day when women could bear children without men. However, her dealings with people such as male lawyer & male banker were entirely reasonable and normal. Court found that the evidence showed "incontrovertibly her morbid aversion to men," and "feminism to a neurotic extreme." Therefore, her mental state of paranoia about how evil men were led her to leave her estate to the NWP.

e. Breeden v. Stone – Court held that the testator suffered from insane delusion but there was not sufficient evidence that materially affected the provisions of his will.

f. In re Honigman - Will leaves wife only a life use of her min statutory share, plus $2500, while giving other members of his family a lot more. Wife objected to probate b/c he was not of sound mind when will made. Wife said he was obsessed with the incorrect belief that his wife was cheating on him. Court denied probate. It doesn’t matter if there existed other reasons why he might have disposed of his property in a specific way - the will is bad if the insane delusion did or might have affected the disposition.

a) Insane delusion vs. Mistake - An insane delusion is a belief not susceptible to correction by presenting the testator with evidence indicating the falsity of the belief. A mistake is susceptible to correction if the testator is told the truth. As a general rule, courts do not reform or invalidate wills because of mistake (although this rule is changing).

g. Note: Dead Man’s Statutes 3. Delusion v. Mental Capacity: Mental capacity is more about general state of mind. Your general ability to apprehend reality:

who you’re giving it to, etc. An insane delusion is about a specific fact. If people try to convince testator that her belief is mistaken, she will stubbornly hang on to the belief

a. The consequences of the two doctrines are different:a) Mental capacity: If someone is proven mentally incapable, the entire will is deemed invalid.b) Insane delusion: The will is deemed to be generally valid, but any provisions that are affected by the insane

delusion are invalidated. But it is possible that an insane delusion can void the entire will. Delusion is about a specific issue.

b. Two differences between Insane Delusion and Capacity:a) With an insane delusion, the testator had an opportunity to correct the mistake of fact, but stubbornly

refused to. b) With an insane delusion, the probate court has discretion to throw out the entire will or to invalidate only

those portions of the will that seem influenced by the insane delusion.

B. UNDUE INFLUENCEUndue influence –when the will of a person who becomes a testator is coerced into doing that which he or she does not desire to do.1. Restatement (Third) of Property: Wills and Other Donative Transfers §8.3 Undue influence - “A donative transfer is procured by

undue influence if the wrongdoer exerted such influence over the donor that it[:]a. overcame the donor’s free will andb. caused the donor to make a donative transfer that the donor would not otherwise have made….”

2. What Influence Is Undue? a. A testator who is unduly influenced has testamentary capacity but the capacity is subjected to and controlled by an evil

individual.b. Estate of Lakatosh – Burden on Roger to rebut presumption of undue influence. When the proponent of a will proves

that the formalities of execution have been followed, a contestant who claims that there has been undue influence has the burden of proof. The burden may be shifted so as to require the proponent to disprove undue influence. To do so, the contestant must prove by clear and convincing evidence that there was a confidential relationship, that the person enjoying such relationship received the bulk of the estate, and that the decedent's intellect was weakened.

c. Presumptions, Burden Shifting, and Undue Influence a) Presumption of undue influence arises if:

1) There was a confidential relationship between the defendant and the testator2) The defendant receives the bulk of the testator’s estate; and3) The testator was of weakened intellect4) SOME jurisdictions: put more emphasis on whether the defendant was active in the procurement or

execution of the will, either substituting it for the 3rd requirement or adding as a 4th).b) Procedural Steps in Will Contest based on Undue Influence

1) Proponent – Proponent of a will has burden of proving its validity (showing due execution). The person contesting the will then has the burden of proving undue influence directly or by proving facts

18 | L e v y

Page 19: Trust and Estates Outline

that give rise to a presumption of undue influence. The burden then shifts back to proponent to negate undue influence. – executed in accordance with required formalities (ch.4)

2) Contestant – BOP of showing undue influence either: directly (very hard) or Circumstantially which triggers rebuttable presumption of undue influence (confidential

relationship + suspicious circumstances) - Rest. (3rd): §8.3 - Factors indicating suspicious circumstances:

1. the extent to which the donor was in a weakened condition, physically, mentally, or both, and therefore susceptible to undue influence;

2. the extent to which the alleged wrongdoer participated in the preparation or procurement of the will or will substitute;

3. whether the donor received independent advice from an attorney or from other competent and disinterested advisors in preparing the will or will substitute;

4. whether the will or will substitute was prepared in secrecy or in haste;5. whether the donor's attitude toward others had changed by reason of his or her

relationship with the alleged wrongdoer;6. whether there is a decided discrepancy between a new and previous wills r will

substitutes of the donor;7. whether there was a continuity of purpose running through former wills or will

substitutes indicating a settled intent in the disposition of his or her property; and8. whether the disposition of the property is such that a reasonable person would regard it

as unnatural, unjust, or unfair, for example, whether the disposition abruptly and without apparent reason disinherited a faithful and deserving family member.

3) Proponent – BOP to negate presumption (or direct evidence) of undue influenced. In re Will of Moses - After the decedent's death, a document dated 1957 was admitted into probate as her last will

and testament. Subsequently, her attorney produced a document dated in 1964 and requested that it be probated as the decedent's last will and testament. The beneficiaries challenged the 1964 document, contending that the decedent was under the attorney's influence when it was executed. The trial court denied the attorney's petition and cancelled the attorney's interest in property that he purportedly purchased with the decedent before her death. On appeal, the court affirmed. The trial court err by finding that a presumption of undue influence arose as the evidence showed that at the time the decedent executed the 1964 document she was ill, disfigured by surgery, addicted to alcohol, and was involved in a romantic relationship with the attorney who was 15 years her junior. The presumption was not rebutted even though the attorney was not present when she executed the 1964 document because the evidence showed that it was drafted by the attorney's partner who gave the decedent no advice but merely wrote down her instructions.

e. In re Kaufmann’s Will - A will is invalid where the evidence shows that the testator did not freely and voluntarily creates his or her will because another individual exerted undue influence over his mind. The Court held that the devisee had exploited his arrangement with the testator to advance his own selfish interests. The devisee falsely testified that he first learned that he was a testamentary beneficiary after the testator's death, although evidence consisting of letters, other memorandum, and testimony indicated that the devisee was involved with the will and transfer of insurance policies.

f. Lipper v. Weslow - The test of undue influence was whether such control was exercised over the mind of the decedent as to overcome her free agency and free will and to substitute the will of another so as to cause her to do what she would not otherwise have done but for such control. The evidence showed that decedent told witnesses her reasons for disinheriting appellees, the will itself explained her reason, and she was of sound mind. While appellees did establish a confidential relationship and an opportunity for undue influence by appellant executor, they failed to prove that the will resulted from his substituting his mind and will for the decedent's.

g. Bequests to Attorneys - a) Majority approach - Many courts hold that a presumption of undue influence arises when an attorney-

drafter receives a legacy, except when related to the testator. The presumption can be rebutted only by clear and convincing evidence provided by the attorney.

b) Unethical Conduct - Model Rule of Professional Conduct 1.8(c) reads: A lawyer shall not solicit any substantial gift from a client, including a testamentary gift, or prepare on behalf of a client an instrument giving the lawyer or a person related to the lawyer any substantial gift unless the lawyer or other recipient … is related to the client.

19 | L e v y

Page 20: Trust and Estates Outline

c) Fiduciary Appointments - The comment to Rule 1.8 advises: In obtaining the client’s informed consent to the conflict, the lawyer should advise the client concerning the nature and extent of the lawyer’s financial interest in the appointment, as well as the availability of alternative candidates for the position.

d) Minority approach – Some jurisdictions are so concerned about gifts to the drafting attorney that they create an irrebutable presumption of undue influence.

1) Two exceptions:1. attorney is related to or married to the testator or2. the will was reviewed by an independent attorney who advised the testator about the potential

for undue influence to make sure the gift was the free and voluntary act of the testator.h. No-Contest Clauses – Clause provides that a beneficiary who contests the will shall take nothing, or a token amount,

in lieu of the provisions made for beneficiary in the will. a) Designed to discourage will contests (so no unmeritorious litigation, family quarrels, defamation of testator).

But, it may also inhibit lawsuits proving forgery, fraud, undue influence – basically nullifying safeguards.b) Majority - enforce no-contest clauses unless there is probable cause for the contest.c) Minority - enforce unless the contestant alleges forgery or subsequent revocation by a later will or codicil, or

the beneficiary is contesting a provision benefitting the drafter of the will or any witness thereto. i. Elements of undue influence - Traditional undue influence doctrine has four elements:

a) Susceptibility - that the testator was susceptible to undue influence, b) Motive - that the influencer had the disposition or motive to exercise undue influence,c) Opportunity - that the influencer had the opportunity to exercise undue influence, andd) Causation - that the disposition is the result of the influence.

j. Effect of a finding of undue influence - If part of a will is the product of undue influence, those portions of the will that are the product of such influence may be stricken and the remainder of the will allowed to stand, if the invalid portions of the will can be separated without defeating the testator's intent or destroying the testamentary scheme.

3. Planning for and Avoiding a Will Contest a. The estate plan might include any combination of a number of features designed to discourage bringing, or limit the

chances of succeeding at, a will contest.a) No-Contest Clauses - A no-contest clause denies benefits to someone who contests a will but the clause can

work only if it is accompanied by a gift to the potential contestants. Otherwise, they have nothing to lose by bringing the contest. In most capacity and undue influence cases, if the contest is successful, the no-contest clause will have no effect. It will fail with the will or the other challenged clauses. When contests fail, courts are divided on whether no-contest clauses are a good idea. Many jurisdictions refuse to apply no-contest clauses if there was “probable cause” to bring the contest. In addition, courts often construe such clauses narrowly.

b) Explanations - When a testator wants to leave out some family members or reduce their shares, one option is for the testator to explain in the will the reasons for the different treatment. If the testator is equalizing treatment among various takers, as when one child’s gift is reduced to take into account a lifetime gift, this technique may work well. When favoritism of one side of the family is prompted by ill will toward the other side, displaying the family laundry in public may fuel a contest.

c) Living Probate - A few jurisdictions allow wills to be admitted to probate before the death of the testator. Though the details vary, the basic idea is to allow a testator to give notice to interested parties of an intent to probate the will. If there are no objections, or if proponents overcome them, the court admits the will to probate. That will, unless it is later revoked, controls distribution of the estate.

d) Living Trusts and Other Gifts - Another way to get property to a favored beneficiary without risking a will contest is to make lifetime gifts, including creating a trust. Later, a will contest would be irrelevant because the property would not be in the estate.

e) Family Law Options - In some situations, testators may be able to protect their estate plans by getting married or by adopting an intended beneficiary.

b. Warning Signsa) New testamentary scheme makes a radical departure.b) Multiple or blended families.c) Imposes conditions that are likely to anger the beneficiary.d) Makes a disposition to a person unpopular with the testator’s familye) Unnatural disposition

C. FRAUD

20 | L e v y

Page 21: Trust and Estates Outline

Fraud occurs where someone intentionally misrepresents something to the testator, with the intent of influencing the testator’s testamentary scheme, and the misrepresentation caused the testator to dispose of his or her property in a way that he or she would not have otherwise.1. Fraud - Testator is deceived by a deliberate misrepresentation and as a result does that which he would not have done.2. Forms of Fraud:

a. Fraud in the Inducement - A misrepresentation causes the testator to execute or revoke a will, to refrain from executing or revoking a will, or to include particular provisions in the wrongdoer’s favor.

b. Fraud in the Execution - A person intentionally misrepresents the character or contents of the instrument signed by the testator, which does not in fact carry out the testator’s intent.

3. Puckett v. Krida - The nurse had been employed to provide 24 hour care for the decedent. After she was employed, the decedent's relationship with appellees began to change. Affirming, the court held that the evidence showed that the decedent had loved her family and was very close to them when the nurse had been employed. Subsequent to the nurse's employment, the deceased began to believe that the niece wanted to put her in a nursing home and that she had misappropriated funds. The evidence showed that neither of these beliefs were true. The evidence further showed that these false beliefs originated with the nurse, who systematically separated the deceased from her family and friends and isolated her from all those individuals with whom she had previously dealt, personally and professionally.

4. Elements of Fraud:a. Mens rea – misrepresentation must be made knowingly and for the purpose of influencing testator’s testamentary

scheme.a) an intent to misrepresent decedentb) plus an intent to acquire a share of will (normally you have to prove both – conduct that seems obviously

deceitful can prove both intent to misrepresent and intent to acquire a share of will)c) if the conduct seems reasonably deceitful, one can infer that the conduct was intentional.

b. Causation – fraud must cause the testator to dispose of his or her property in a way that he or she would not have otherwise.

a) This requires contestant to prove a counter-factual. She was planning to do X, and then she was lied to and did Y. Sometimes you have to show that she was planning to do X.

c. Remedy – depends on the effect of the fraudulent misconduct. A provision in a will procured by fraud is invalid, but the remaining portion stands unless the fraud goes to the entire will or the portions invalidated by fraud are inseparable from the rest of the will. (Diff from mental capacity where the whole will is void, b/c there can’t be capacity for some parts and not others.)

a) Invalid: A provision in a will procured by fraud is invalid. The remaining portion of the will stands unless the fraud goes to the entire will or the portions invalidated by the fraud are inseparable from the rest of the will.

b) Remedy: Where the probate court cannot do justice by refusing probate, the will may be probated and then a court with equity powers can impose a constructive trust on one or more of the beneficiaries to remedy the unjust enrichment caused by the fraud.

D. DURESS When undue influence becomes overtly coercive, it becomes duress. Transfers procured by duress are invalid.1. Rest (3d) Prop §8.3(c): A donative transfer is procured by duress if the wrongdoer threatened to perform or did perform a

wrongful act that coerced the donor into making a donative transfer that the donor would not otherwise have made.2. Latham v. Father Divine - Plaintiffs, decedent's first cousins, were not distributees of decedent's will, which gave almost her

entire estate to defendants, cult leader and others. Plaintiffs claimed that shortly prior to the death of the deceased she had attorneys draft a new will in which plaintiffs were named as legatees for a very substantial amount and that defendants' false representations, undue influence, and physical force prevented the deceased from executing the new will. Plaintiffs claimed that those facts, if proven, would have entitled them to a judicial declaration that defendants, taking under the already probated will, held what they had so taken as constructive trustees for plaintiffs, whom decedent wished to, tried to, and was kept from, benefiting. The court reversed the judgment of the appellate court dismissing plaintiffs' complaint because plaintiffs may have had a constructive trust claim.

3. Elements of duress - Duress requires:a. a wrongful and coercive actb. that causesc. the testator to change his testamentary disposition.

4. Appropriate remedy? a. Invalidate Portions of will tainted by duress - Probate the parts of the will that are valid.b. Invalidate entire will .

21 | L e v y

Page 22: Trust and Estates Outline

c. Constructive trust - A constructive trust may be imposed where no fraud is involved if the court thinks that unjust enrichment would result if the person retained the property. A constructive trust is not really a trust but an equitable remedy. It is a remedy that employs the language of trusteeship, even though it is not really a trust. When property has been acquired in such circumstances that the holder of the legal title may not in good conscience retain the beneficial interest, equity converts him into a trustee. Once converted into a constructive trustee, the holder of the property must transfer it to the constructive beneficiary

NOTE: CONSTRUCTIVE TRUST (IN CASES OF FRAUD, DURESS OR UNDUE INFLUENCE)• Constructive trust is a remedy; alternative to a will contest.• Where the probate court cannot do justice by refusing probate, the will may be probated and then a court with

equity powers can impose a constructive trust on one or more of the beneficiaries to remedy the unjust enrichment caused by fraud, duress or undue influence (unjust enrichment if devisee permitted to keep property that otherwise they would not have taken but for the wrongful acts).

• This type of post-probate procedure is much riskier than contesting the probate of the will before the property is distributed, since by the time a judgment is obtained, the Δ may have disposed of the property or not have assets to satisfy the value of the property.

E. TORTIOUS INTERFERENCE WITH AN EXPECTANCY Where a third party has intentionally committed tortious conduct in the testamentary process (undue influence, fraud, or duress), those who would have taken but for the misconduct van sue the third party for tortious interference with an expectancy. Raise this and argue it in the alternative!1. The plaintiff has to prove:

a. existence of expectancyb. a reasonable certainty that the expectancy would have been realized but for the interferencec. intentional interference with expectancyd. tortious conduct involved with the interference, such as fraud, duress, or undue influence ande. damages.

2. Schilling v. Herrera - Plaintiff, a decedent's brother, sued defendant, the decedent's caretaker, for intentional interference with an expectancy of inheritance. Plaintiff alleged the caretaker used undue influence to convince the decedent to prepare and execute a new will naming the caretaker as personal representative and sole beneficiary. The trial court held the complaint failed to state a cause of action and that the brother was barred from filing the action because he failed to exhaust his probate remedies. The appellate court disagreed with both findings. That the complaint failed to allege that the caretaker breached a legal duty owed to the brother was immaterial, as the tortious conduct required for the interference tort was directed at the testator, not the beneficiary. By alleging that the caretaker did not advise the brother of the decedent's death until after she had petitioned the probate court for discharge of probate, the brother sufficiently alleged that he was prevented from contesting the will in the probate court due to the caretaker's fraudulent conduct.

3. Advantages: Tort action has advantages over a suit in probate for fraud or undue influence.a. Not a will contest - Plaintiff may take under the will despite the no-contest clause.b. Punitive damages – Plaintiff is eligible to claim punitive damages.c. Longer stature of limitations – SOL does not begin to run until the party discovers or should have discovered the

misconduct.4. Note: Anna Nicole Smith and the Probate Exception to Federal Jurisdiction

________________________________________________________________________________________________________________CHAPTER 4. WILLS: FORMALITIES AND FORMS

A. EXECUTION OF WILLS 1. Attested Wills – most common type of will.An attested will is a will whose signing is witnessed. All states permit attested will as a valid will. Most basic requirements: writing, signature, and attestation by witnesses.

a. Introductiona) The Function of Formalities:

1) Ritual Function - The performance of some ceremony for the purpose of impressing the transferor the significance of his statements.

22 | L e v y

Page 23: Trust and Estates Outline

2) Evidentiary function – increase reliability of evidence to the court- shows there wasn't fraud/coercion (there are witnesses); signature in specific location

3) Channeling Function - the benefit of having requirements is piece of mind to testatrix to know that if

you follow requirements, then you know your will is going to be valid & in effect; Standardization of

form simplifies administration.4) Protective Function – more difficult for testator to be taken advantage of; Prophylactic purpose of

safeguarding the testatorb) Comparison of Statutory Formalities for Formal Wills:

Statute of Frauds (1677) Wills Act (1837) UPC (1990)Writing Writing WritingSignature Subscription (signature must be at the

“foot or end” of will)Signature

Attestation & subscription by three witnesses

Attestation & signature by two witnesses

Attestation & signature by two witnesses

c) Uniform Probate Code §2-502 - Execution; Witnessed Wills; Holographic Wills1) (a) Except as provided in subsection (b) and in Sections 2-503, 2-506, and 2-513, a will must be:

1. (1) in writing;2. (2) signed by the testator or in the testator's name by some other individual in the

testator's conscious presence and by the testator's direction; and3. (3) signed by at least two individuals, each of whom signed within a reasonable time

after he [or she] witnessed either the signing of the will as described in paragraph (2) or the testator's acknowledgment of that signature or acknowledgment of the will.

2) (b) A will that does not comply with subsection (a) is valid as a holographic will, whether or not witnessed, if the signature and material portions of the document are in the testator's handwriting.

3) (c) Intent that the document constitute the testator's will can be established by extrinsic evidence, including, for holographic wills, portions of the document that are not in the testator's handwriting.

b. Common Law approach to attested wills - Writing, Signature, and Attestation - Strict ComplianceUnder traditional law, for a will to be admitted to probate it must be in strict compliance with formal requirements of the applicable Wills Act. The will must be in writing, signed by the testator, and attested by at least two witnesses, plus any additional requirements that are mandated by the particular jurisdiction must be satisfied precisely. Majority of states still follow this.

a) The common law judicial approach to the statutory Wills Act formalities is to require absolute strict compliance with each Wills Act requirement, no matter how clear the testator’s intent that this document be his or her last will. If there is any deficiency in the execution ceremony, the document is not a valid will. Strict compliance approach to the Wills Act formalities. A number of requirements are common to most statutes:

1) Writing – As a general rule, oral wills are not permitted. To have a valid will, there must be a writing.2) Signature by the testator - Testator must intend it to be his signature. There is no requirement that

the testator signs his full name, however, if he intends to do so and does not complete it, the general rule is that the partial signature does not qualify as the testator’s signature. Thus, an X can count if he intended it to be his signature.

3) Signature by another – The will must be signed by someone other than the testator, as long as the person signs the testator’s name, in the testator’s presence, and at the testator’s direction. The testator’s direction must be express; it will not be implied.

Signature” in Will ExecutionPurposes: • Ceremonial (to provide evidence of finality) • Evidentiary (to provide evidence of genuineness)Type of Mark Location of Mark Order of “signing”1) Name in full2) A cross (X), abbreviation,

or nickname?3) Electronically printed

1) At the end of the document, generally

2) Subscription: “at the foot or end thereof” (required

Witnesses:1) At time of T’s signing or

acknowledgment, as part of one continuous

23 | L e v y

Page 24: Trust and Estates Outline

name in full (cursive font)?

4) Initials and date?5) Mark made by someone

else at the direction of T

by a few states)3) Somewhere else?4) Additions to will made

after “signing” may be invalid

transaction2) “Within a reasonable

time”?3) After T’s death?

4) Witnesses – most jurisdictions require that the testator sign or acknowledge in the presence of at least 2 witnesses, who are present at the same time. The witnesses must sign the will and, in some jurisdictions, the witnesses must know that what they are signing is the testator’s will.

1. Interested witness – witness who stands to benefit if the testator’s will is valid. Most common type is a beneficiary under the will.

a. Common Law – the entire will is void unless there was an extra witness to validate the will.

b. UPC 2-505(b) – the beneficiary takes property exactly as the testator specified in the will.

5) Presence – The testator must sign or acknowledge in the presence of the witnesses, and under the traditional approach, the witnesses must sign in the presence of the testator. Line of sight vs. Conscious presence test for witness acknowledgment/attestation:

1. 1837 Wills act required witnesses to be present at the same time when the testator signed or acknowledged the will. Although some states have retained this requirement, most states do not require the witnesses to be together either 1) when the testator signs or acknowledges the will, or 2) when the witnesses attest to the will.

2. Conscious presence test (strict)- Under the traditional approach, the actor who has to perform the specified act so that the second party either sees or has the opportunity of seeing the act. The latter means that the second party would have actually seen the specified act if the 2nd party had looked at the right moment.

3. Line of sight test (more flexible)- Under the modern trend approach, presence I defined by whether the party, in whose presence the act has to be performed, can tell from sight, sound, and general awareness of the events that the required act has to be performed.

a. In re Groffman – Testator signs will. Witness #1 comes into room & signs in testator’s presence, then leaves. Then Witness #2 comes into room & also signs in testator’s presence. Court held that the will not properly executed, b/c the law relevant here provides that the “signature shall be made or acknowledged in the presence of 2 or more witnesses present at the same time,” and “such witnesses shall attest & shall subscribe the will in the presence of the testator.

b. Stevens v. Casdorph - Will executed at bank; decedent signs in presence of bank employee (notary). Employee takes will for 2 witnesses to sign, at separate times; each did not see the other nor the testator sign. Will improperly executed. Statute requires that “signature shall be made or the will acknowledged by him in the presence of at least two competent witnesses, present at the same time; and such witnesses shall subscribe the will in the presence of the testator, and of each other." No one saw anyone sign; no communication between any of them either.

1) Note – if the conscious presence test was used here instead of the line of sight test, the execution would have been valid b/c everyone in bank knew what was going on (small town, etc.), and it was within a reasonable time (UPC approach).

2) Also, argument of substantial compliance was made here, but the court was unwilling to vary from the strict requirements of the law

Line of Sight Test Conscious Presence TestWhat does “presence” mean?

The testator does not actually have to see the witnesses sign, but must be able to see them were the testator to look.

The testator, through sight, hearing, or general consciousness of events, comprehends that the witness is signing

4. Modern trend – Abolished the requirement that the witnesses sign in the presence of the testator. Under such statutes, there is only one “presence” requirement; that the testator

24 | L e v y

Page 25: Trust and Estates Outline

sign or acknowledge the will in the presence of the witnesses. The witness can sign within a reasonable amount of time after the testator says that he signed it. Don’t have to actually see testator sign it. UPC § 2-502 - Execution: Wills - A will must be:

a. in writing,b. signed by the testator or in the testator's name by some other individual in the

testator's conscious presence and by the testator's direction; andc. signed by at least two individuals, each of whom signed within a reasonable time

after he witnessed either the signing of the will as described in paragraph (2) or the testator's acknowledgment of that signature or acknowledgment of the will.

b) The Meaning of “Writing” and Video or Electronic Wills 1) Videotaped wills – Although argument exists on both sides of the issue, to date no court has upheld

a videotaped will.2) Electronic wills – Nevada permits electronic wills executed under very strict requirements.

c) Estate of Morea - the issue was whether a bequest to the decedent's friend was void under N.Y. Est. Powers & Trusts Law § 3-3.2 in light of the fact that the friend was one of the three attesting witnesses and that decedent's son, whose legacy under the will was less than his intestate share as one of decedent's six surviving children, was also one of the attesting witnesses. The third attesting witness did not receive any disposition or appointment under the will. The court held that the decendent's son did not forfeit his legacy as a result of being both an attesting witness and a beneficiary because N.Y. Est. Powers & Trusts Law § 3-3.2(a)(3) permitted him, as a distributee, to receive the lesser of his intestate share or his legacy under the will. The court held that the disposition to the decedent's friend was not invalid under N.Y. Est. Powers & Trusts Law § 3-3.2(a)(1) because there were at least two other witnesses to the will who received no beneficial disposition thereunder.

d) Note: Recommended Method of Executing a Will e) Safeguarding a Will

1) Selection of witnesses 1. Must be disinterested - shouldn’t be anyone that benefits under the will2. Get 3 witnesses (maj jur require 2, so 3 just in case there’s a problem with one)3. If possibility of will contest/challenge, want witness who is credible & has known testator a

long time2) Present at Execution - Everyone should be present at time of execution (all witnesses, the lawyer,

testator & notary, no one else should be there). Door closed; no one enters or leaves until ceremony done.

3) Testator Recognition of Will & Signature 1. Ask testator: “Is this your will?” Have testator recognize will; make sure that it is the will

testator wants to execute, and that testator understands it. 2. Witnesses should be positioned so that they can see testator sign. Witnesses don’t need to

know contents of the will. 3. Testator signs on bottom of last page, with all pages firmly affixed together. Good idea to

also initial & date on each page. The will should also specify how many pages it consists of.4) Witness Attestation - An attestation clause is a provision of a will that recites that the testator duly

executed the will. In General, the Attestation Clause should say that:1. Three Witnesses

a. Present at the same timeb. Heard the testatrix declare the document to be her willc. And observed the testatrix sign her will in their presence

2. And that, then and only thena. Each witness did

i. in the presence of each other and the testator and ii. at the request of the testator

b. Sign the will as witnesses.5) Self-Proving Affidavit – typed at end of the will and signed by testator & witnesses, before a notary,

swearing that the will has been duly executed (notary then signs & attaches seal).1. Purpose: so that after testator's death, witnesses don’t need to testify to the execution of

will, because already done with the self-proving affidavit. This becomes important if

25 | L e v y

Page 26: Trust and Estates Outline

witnesses have died or if they moved far away. Although the will is still valid without an affidavit, it makes it easier to probate.

2. Self-Proving Affidavit vs. Attestation – attestation serves as evidence that the testator executed the document; the affidavit says that all the other formalities were done (duly executed).

Attestation clause Self-proving affidavitsprima facie evidence that the testator voluntarily signed the will in the presence of the witnesses

Sworn statements by eyewitnesses that the will has been duly executed

permits probate of a will when a witness forgets the circumstances of the will's execution or dies before the testator

Performs all functions of the attestation clause, plus has the further effect of permitting probate without requiring the appearance of either witness

the attestant expresses the present intent to act as a witness

the affiant swears that the will has already been witnessed

3. Two types of Self-Proving Affidavits (UPC § 2-504 authorizes both types):a. One Step (Combined attestation/affidavit) – the affidavit is part of the will, and both

affidavit (signed by witnesses, testator & notary) & attestation (signed by witnesses) are one document, so everyone only signs once, and is attached within the will and conclude the will.

b. Two Step – affidavit is attached but not technically part of the will (2 separate docs). The will is duly executed, as is the attestation clause (with witnesses’ signatures) that follows the testator’s signature. Attached to the executed and attested will, is the self-proving affidavit (signed by the testator, the witnesses, and a notary), which is signed after testator signs the will, and after witnesses sign attestation clause

One-Step Process Two-Step ProcessSelf-Proving Wills Witnesses sign ONCE

Affidavit language is a part of the attestation clause Notarization of the affidavit

Witnesses sign attestation clause THEN, witnesses sign a separate affidavit Notarization of the affidavit

6) Duplicates: Testator should not execute more than one copy – too many ambiguities might arise as to why this was done

7) After execution 1. Executed will should be placed somewhere safe; let people know where it is2. Keeping with attorney – looks like solicitation (b/c client must come back to see atty again &

this implies a continuation of the transaction)2. Curing Defects in the Execution of Attested Wills

a. Excusing Execution Defects by Ad Hoc Exceptiona) Traditional rule - Strict Compliance – to be validly executed, a will must comply strictly with the requirements

set by statute. Missouri is a strict compliance jurisdiction.1) In re Pavlinko’s Estate - Husband & wife sign each others’ wills by mistake. Will practically identical –

left property to each other, and to other family members. Court refuses probate of will. Statute requires that “every will shall be in writing and shall be signed by the testator at the end thereof.” Court refuses to vary from the strict requirements of the law, explaining that “once a court starts to ignore, alter, rewrite, or make exceptions to clear, plain and unmistakable provisions of the [statute] in order to accomplish equity and justice in that particular case, the [statute] will become meaningless, and the door will be opened wide to countless fraudulent claims which the [statute] successfully bars.”

b) In re Snide - Husband and wife sign each others’ wills by mistake; wills were identical except for the differences in names of the donors and beneficiaries on the wills. Court admitted will to probate. Court explained that testamentary intent does not attach to the specific document signed so long as the testator

26 | L e v y

Page 27: Trust and Estates Outline

intended to execute the will. Here, the two wills were practically identical. This clearly evinced a valid testamentary scheme. Also, no evidence of fraud. This case is an exception - most courts won’t fix this.

b. Curative Doctrines: Substantial Compliance and Harmless Error a) Substantial compliance - One solution for those frustrated with the tradition of strict interpretation of will

statutes’ requirements is to validate a will if there has been “substantial compliance” with the statutory elements. Under substantial compliance, the will is admitted to probate if there is clear and convincing evidence that the purposes of formalities – the evidentiary, cautionary, protective, and channeling functions – were served despite a defective execution, the will is admitted to probate.

1) In re Will of Ranney - The will was 4 pages & 5th page was a self-proving affidavit. At the execution, the testator signed the 4th page, but the witnesses only signed the 5th page - the self-proving affidavit. The testator and witnesses all thought they were signing and attesting the will. Will then notarized and all 5 pgs stapled together. The affidavit stated that each witness signed the will as witnesses in the presence of the testator (but this didn’t happen b/c actual will not signed). Holding: Will admitted to probate, although it did not adhere to the statutory formalities. Court explained that the statutory requirements serve an evidentiary function (giving court evidence of terms of will & testamentary intent), a ritual function (shows the seriousness of event), and prevents fraud & undue influence. The signatures on the self-proving affidavits satisfy these requirements. If the witnesses, with the intent to attest, sign a self-proving affidavit, but do not sign the will or attestation clause, clear and convincing evidence of their intent should be adduced to establish compliance with the statute.

2) In re Estate of Hall - Will #1 executed. 14 yrs later, testator & wife want to execute a Joint Will. Joint Will executed, but without required attesting witnesses present. Wife tears up Will #1 at testator’s direction. Court allowed probate of Joint Will. Although formal requirements are lacking, there was clear & convincing evidence that the testator intended to void Will #1 by destroying it and signing the Joint Will. By signing the Joint Will, testator intended to validate it.

b) Harmless Error/Dispensing Power - Under the dispensing power doctrine, the court has the power to probate the will if clear and convincing evidence shows that the decedent intended the document to constitute his will. The key is: testamentary intent

1) Uniform Probate Code §2-503 Although a document or [writing added - codicil] was not executed in compliance with [2-502], the document or writing is treated as if it had been executed in compliance with that section if the proponent of the document or writing establishes by clear and convincing evidence that the decedent intended the document or writing to constitute

1. the decedent's will, 2. a partial or complete revocation of the will, 3. an addition to or alteration of the will, or 4. a partial or complete revival of his [or her] formerly revoked portion of the will.

2) Rest. (3d) Prop. § 3.3. - A harmless error in executing a will may be excused if the proponent establishes by clear and convincing evidence that the decedent adopted the document as his or her will.

3) The justification is that the purpose of the statute is to protect against fraud. But if evident there’s no fraud, then it’s ok.

c) Substantial Compliance Versus Harmless Error - Note carefully that while substantial compliance focuses on being close, harmless error ignores the traditional statutory elements and focuses directly on whether the testator intended the document to be effective.

3. Notarized Wills - Revised UPC: Pursuant to the 2008 revisions to UPC §2-502(a)(3), a will is valid if signed by two witnesses or a notary..or (3) . . . (B) acknowledged by the testator before a notary public or other individual authorized by law to take acknowledgments.” The logic behind the new provision is that:

a. A single notary can serve as a pair of witnesses, andb. Under the harmless error doctrine, such a will would be valid even without the notarization.

4. Holographic Wills A holographic will is a will written by the testator’s hand and signed by the testator; attesting witnesses are not required. Holographs need not be witnessed but states require additional elements to assure genuineness of the document. UPC § 2-502(b) requirements: the signature and “material portions” must be in the testator’s handwriting. More traditional statutes require the document to be “entirely” in the testator’s handwriting and may require a date or some of the elements applicable to attested wills. Allowed in only about ½ states.

27 | L e v y

Page 28: Trust and Estates Outline

a. Holographic Wills - A testamentary disposition need not be set out in formal document; a letter can also be a valid holographic will, as can a variety of other documents.

a) Conditional wills - Kimmel’s Estate - Testator mailed a letter to 2 of his children. The letter was written in Kimmel's handwriting, signed at the end "Father," dated Dec 12, 1921, and contained the following: "I have some very valuable papers I want you to keep fore me so if enny thing happens all [of my property] goes to [my sons] Kepp this letter lock it up it may help you out." Kimmel died the same day he sent the letter. The Court probated the letter as a holographic will. Court focused on the issue of testamentary intent (to create a will). Here, the words, "if enny thing happens," strongly support the idea of testamentary intent. Also, he signed it "Father." When taken in context of the letter, "Father" was intended as a completed signature.

b. Requirements for holographic wills:a) Writing – as with attested wills, holographic wills must be in writing.b) Signed – holographic wills must be signed. However, unlike attested wills, only the testator can sign a

holographic will. In almost all states permitting holographs, a holograph may be signed at the end, at the beginning, or anywhere on the will, but if not signed at the end there may be doubt about whether the decedent intended his name to be a signature.

c) Dated – some states require that holographic will to be dated to be valid.1) But, the UPC does not require this. UPC §2-502.

d) Handwritten – holographic wills must be in the testator’s writing.1) Entirely - First generation statutes – The entire will must be handwritten by testator Some wills

struck down even if just a couple words were printed/stamped or if will written on stationary w/ pre-printed info.

2) Material provisions - Second generation statutes/Old UPC – Only material provisions of the will must be in T’s handwriting. 5states still require this.

3) UPC approach - Third generation statues/New UPC – Only material portions of will must be in T’s handwriting. Only most important words must be in T’s handwriting.

e) Testamentary intent – holographic wills must establish testamentary intent.1) Estate of Gonzalez - The decedent had filled in certain blank spaces in a preprinted will form with

various bequests. He then signed the form. The trial court found this to have been a valid holographic will. The will did not qualify as a will under Me. Rev. Stat. Ann. tit. 18, § 2-502 (1998) because it was not signed by any witnesses. The children argued that the will was not a valid holographic will because a material provision of the will-evidence of testamentary intent-appeared in the preprinted portion of the document, and was not handwritten. They maintained that the handwritten words were a list of what the decedent wanted to do with his property, but the handwritten words did not indicate that the conveyances were testamentary in nature. The appellate court held that printed portions of a will form could be incorporated into a holographic will where the trial court found a testamentary intent, considering all of the evidence. The preprinted portions of the form at issue stated testamentary intent, and because the blanks in those portions were filled in by the decedent's handwriting, they became a valid statement of testamentary intent sufficient for a holographic will.

c. 3 types of Holographic Statutes: A holographic will must be: (i) signed by the testator, and (ii) written by the testator’s hand (How much?)

a) First Generation: requires “entirely, written, signed and dated” in the T handwriting (10 states)b) Second Generation (1969 UPC): requires “signature and the material provisions” in the T handwriting (7

states)c) Third Generation (1990 UPC § 2-502(b)): requires “signature and material portions” in the T handwriting (9

states)d. Handwriting – whether entirety, or if not, how much of the holograph must be in the T handwriting. -- this was issue in

Gonzalesa) First Gen: holographs with one or two printed words on the page may be struck down

1) Thorn: a will written in T own handwriting struck down because. he had stamped the name of his home, Cragthorn, 2 times within the text

2) Holographs with another’s handwriting may be struck down3) Dobson: T took entirely handwritten will to banker to discuss with him. Banker penciled in some

numbers and edits.1. Held: could not be probated because it was not entirely in handwriting of T

28 | L e v y

Page 29: Trust and Estates Outline

4) Dated- some jurisdiction require full date in T own handwriting – (of 10 states with 1st gen – only 2 require full date in T handwriting)

b) 2nd Gen: Signature and Material Provisions (1969 UPC) – Gonzales1) meant to allow some immaterial provisions to be handwritten2) Problem – is the pre-printed will forms, like Gonazales, where operative transfer language is typed.3) 1969 UPC led to inconsistent results4) One Approach: ignore the printed words and force handwritten words to stand alone

1. created ambiguities by removing words from context 5) Other (Az/Me): reason that the fact that the printed words cannot be given dispositive intent does

not mean that the printed text cannot be used to give context to handwritten wordsc) 3rd gen – 2-502(b)Drafters of UPC unhappy that courts still striking down seemingly reliable holographs –

particularly the pre-printed will forms1) did 2 things:

1. #1: Changed from material provisions to material portions.a. intended to allow probate even if immaterial parts are printedb. specifically – comments say “I give, devise bequeath – should not disqualify a pre

printed will form as a valid holograph if rest is filled out by hand2. #2 – 2-502(c): explicitly allows extrinsic evidence to be used to establish testamentary intent

a. Comments: intent that document be the T will can be established by extrinsic evidence, including for holographs, portions of the document not in T handwriting

b. so encourages courts to look at printed words in addition to handwritten ones.d) UPC § 2-502(b)-(c)

1) (b) [Holographic Wills.] A will that does not comply with subsection (a) is valid as a holographic will, whether or not witnessed, if the signature and material portions of the document are in the testator’s handwriting.

2) (c) [Extrinsic Evidence.] Intent that a document constitute the testator’s will can be established by extrinsic evidence, including, for holographic wills, portions of the document that are not in the testator’s handwriting.

e. Judicial Approacha) In re Estate of Kuralt - Testator, married, had a longtime affair with another woman. He was her primary

source of financial support. In 1989 he drafted a holographic will bequeathing Montana property to lover. In 1994, he executed a formal will giving to wife & kids, which included no mention of the Montana property. In 1997, he gave lover money & sold part of Montana prop to her. He planned to make another such transaction, but he got sick before he could do so. In hospital, he wrote a letter to lover indicating his intent to give her the rest of the Montana property, but died before he could arrange it. Court allows the letter in as a codicil; the evidence clearly establishes this was his testamentary intent. He intended to transfer Montana prop to lover. Evidence to support this: (1) long term relationship, (2) he was her main financial support for her & her family, (3) previously transferred part of the property to her for no real consideration, (4) sent lover letter indicating he wanted her to have the property.

b) Codicil: A codicil is a testamentary instrument that amends it prior will; it does not replace it. 1) Example: A holographic will devises certain property not mentioned in the formal will. The

holographic will is a codicil b/c it amends the prior formal will

B. REVOCATION OF WILLS Before you dive into revocation, you must first determine if the document was a valid will in the first place. Wills are executed inter vivos but are not effective until death. If testator changes his mind afer executing the will, the testator can revoke it, replace it, or amend it anytime. A will can be revoked (1) by act, (2) by writing, (3) by presumption, and (4) by operation at law.1. Revocation by Writing or Physical Act

a. Revocation by a physical act such as destroying, obliterating, or burning the will.a) Common law requirements:

1) Capacity to revoke2) Intent to revoke3) Performance of the revocatory act by the testator’s presence (line of sight) by cutting, tearing,

burning, obliterating, canceling, destroying the will, or the signature (UPC only asks for conscious presence).

29 | L e v y

Page 30: Trust and Estates Outline

1. Harrison v. Bird - Decedent executed a will in 1989. Attorney retained the will and the duplicate original given to decedent. In 1991, decedent called her attorney and said she wanted to revoke her will. Attorney then tore the will into 4 pieces and sent it to decedent along with a letter explaining what had been done, and said "As it now stands, you are without a will." When decedent died, the letter from the attorney was found, but the 4 pieces of the will were not. Under CL, revocation of the will must be done by testator or in the testator’s presence. So the act of tearing up the will did not revoke it. However, there is a presumption that the will has been revoked per the lost wills doctrine, since it cannot be found & it was last in testator’s possession.

2. Revocation by proxy – Most states permit a testator to designate a proxy to perform the actual revoking physical act on the will. Just as proxy signatures of testator’s name, most states require the proxy to act at the testator’s direction and in the testator’s presence when the physical act occurred.

4) The physical act must affect the written portion of the will.1. Revocation by Cancellation: If written words are to be used for the purpose of revoking a

will, they must be so placed on the will as to physically affect the written portion; merely writing on blank parts of the paper is not enough. Cancellation includes “any act which would destroy, revoke, recall, do away with, overrule, render null and void, the instrument.”

a. Thompson v. Royall – Decedent executed a will, then 11 days later executed a codicil. 4 days after that, decedent decided will should be destroyed. On the back of the will, someone else (not decedent) wrote that the will is "null and void," and mentioned reason for not actually destroying will was to keep as reference. Decedent signed and dated this; a similar note was made on the back of the codicil. The only question presented by the record was whether the testatrix's will had been revoked shortly before her death. The notations in question were not wholly in the handwriting of the testatrix, nor were her signatures thereto attached attested by subscribing witnesses; hence under the statute they were ineffectual as some writing declaring an intention to revoke. The faces of the two instruments bore no physical evidence of any cutting, tearing, burning, obliterating, canceling, or destroying. The heirs argued that the notation written in the presence, and with the approval, of the testatrix, on the back of the manuscript cover in the one instance, and on the back of the sheet containing the codicil in the other, constituted canceling within the meaning of Va. Code Ann. § 5233. The proof established the intention to revoke but the testatrix failed to carry out that intent as required under the statute.

Revocation by execution of subsequent will – the notations on the back must have been a properly executed will or codicil. Since no witnesses, not an attested will. Since not completely in testator’s handwriting, not a holograph. So not revoked this way.

Revocation by physical act – the will was in no way destroyed. Revocation by cancellation – No. Revocation by cancellation contemplates

marks or lines across the written parts of the instrument or a physical defacement, or some mutilation of the writing itself, with the intent to revoke. Null and void written on a blank part of the will (on the back).

2. BUT, UPC § 2-507: "A burning, tearing, or canceling is a 'revocatory act on the will,' whether or not the burn, tear, or cancellation touched any of the words on the will." However, the words of cancellation must be written on the will itself, not on another document.

§ 2-507: Revocation of Wills : a will or any part thereof is revoked:By subsequent writing (a)

Executed with wills act formalities (§ 2-502) or allowable under harmless error (§ 2-503)

Revokes previous will (or part) expressly or by inconsistency (see § 2-507(b)-(d))By physical act (b)

“revocatory act on will”- Includes burning, tearing, canceling, obliterating, or destroying will or part of it. Does not need to touch the words of the will.

30 | L e v y

Page 31: Trust and Estates Outline

Performed by T with intent and for purpose of revoking the will or if another individual performed the act in the T conscious presence and by the T direction.

§ 2-507 (c): A subsequent will that does not expressly revoke a prior will but makes a complete disposition of the T’s estate is presumed to replace the prior will and revoke it by inconsistency § 2-507(d): A subsequent will that makes only a partial disposition of the T estate – is NOT presumed to revoke the prior will in wholeINSTEAD viewed as a supplement to the existing will (AKA “codicil”) revoking only the part of the previous will that is inconsistent with the current will. UPC § 2-507(c)-(d) need C + C evidence to rebut the above presumptions

b. Revocation by subsequent writing executed with testamentary formalities – a will may be revoked by a subsequent writing only if the subsequent writing qualifies as a valid will. This writing may be 1) a new will, 2) an amendment to the existing will – a codicil, 3) a document that revokes the prior will but does not contain replacement dispositive provisions.

b) Common law requirements :1) intent to revoke2) A subsequent will or codicil, or another writing declaring an intention to revoke, and executed in the

manner in which the will was required to be executed.c) Uniform Probate Code §2-507 (a)(1) - A will or any part thereof is revoked . . .by executing a subsequent will

that revokes the previous will or part expressly or by inconsistency.d) Express revocation - The most common, and surest, way to revoke a will is by executing a later document that

expressly revokes the will. Documents intended to revoke a will in part are called “codicils.”e) Revocation by Inconsistency – the provision in the instrument executed closes to the time of death controls

the inconsistencies.1) A subsequent will wholly revokes the previous will by inconsistency if the testator intends the

subsequent will to replace rather than supplement the previous will. 2) A subsequent will that does not explicitly revoke the prior will but makes a complete disposition of

the testator’s estate is presumed to replace the prior will and revoke it by inconsistency.3) If the subsequent will does not make a complete disposition of the testator’s estate, it is not

presumed to revoke the prior will but is viewed as a codicil. c. Revocation of a photocopy of the will is not a valid revocation. However, if evidence of intent to revoke, court may

impose a constructive trust (In Estate of Tolin). d. Partial Revocation – Traditional approach is that types of alterations are ineffective and the will is given effect in its

pre-changed form. Modern approach and many (but not all) states allow partial revocation by physical act. Often the question is whether the physical act was intended to revoke the whole will, or only a particular section. A different sort of partial revocation problem arises if there are both a will and a codicil, but only one document is touched. Imagining the will as a base, with the codicil resting on top, helps the general rules make sense. If the intention is to revoke both documents, destroying the will is usually sufficient. If the intention is unclear, however, the codicil may remain valid if it can sensibly stand alone, but it will fail if it needs the will’s support. Conversely, tearing up a codicil normally leaves the supporting will in place.

2. Presumptions – Proponent of the will must prove that the will is indeed the testator did not revoke the will. a. Presumption of nonrevocation – To use this presumption, both source of the will and the will itself must be free of

suspicion (no large eraser marks, torn pages taped together, a signature page that looks like it belongs to a different document).

b. Presumption of revocation – A will proponent who cannot produce the original will is usually confronted with a presumption of revocation. Proponent has a heavy burden to rebut this presumption and show that the will was inadvertently destroyed in a natural disaster.

a) Lost wills doctrine - If the evidence establishes that a person had possession of her will prior to her death, but the will is not found among her personal effects after her death, a presumption arises that she destroyed the will. Additionally, if the copy of a will that is in the testator’s possession is destroyed, a presumption arises that the testator revoked the will and all duplicates that may exist.

1) Probate of Lost Wills - If a will is lost, destroyed w/o consent of testator, or destroyed w/ testator’s consent but not in compliance with the revocation statute, it can still be admitted into probate if its contents are proved by clear & convincing evidence.

31 | L e v y

Page 32: Trust and Estates Outline

2) Some jurisdictions don’t like this approach, and statutes prohibit the probate of a lost or destroyed will unless the will was in existence at the testator's death (and destroyed thereafter) or was fraudulently or mistakenly destroyed during the testator's life.

3. Dependent Relative Revocation and Revival a. Dependent Relative Revocation (implied conditional revocation) - if the testator purports to revoke his will upon a

mistaken assumption of law or fact, the revocation is ineffective if the testator would not have revoked his will had he known the truth. First ask: Did this happen by mistake? If not, ignore DDR.

a) The doctrine would imply a condition on testator’s revocation by physical act of Will #1 (testator revoked will one on assumption that Will #2 is effective). Because Will #2 is invalid, the condition was not satisfied and thus the revocation Will #1 is ineffective. Courts look closely at what the testator intended would the testator rather have the testator’s estate pass under Will #1 or via intestacy?

b) Example: Suppose Hans Schmidt wanted to redo his will. At the execution ceremony for the new will, Hans somehow managed not to sign it. Wrongly believing the new will to be effective, he tore up the old one. When the mistake is discovered after Hans’ death, a court unable to probate the new will (because there is no signature) might well use DRR to ignore the revocation of the old will. In theory, Hans’ revocation of the old will was dependent upon the new will being effective; since the new will fails, the old will was never revoked when Hans tore it up.

1) The doctrine does not give Hans what he really wanted, and thought he had: the new will. Rather, he gets a consolation prize: the old will he thought he had revoked. Most courts look at the circumstances, compare the various wills and the intestate statute, and then choose what they believe the testator would have done had the testator known what hindsight has revealed.

2) Minority of jurisdictions that have adopted the harmless error or substantial compliance rule, however, will give the court power to excuse trivial errors.

c) DRR sustains a revoked gift if a testator cancels or destroys a will with a present intention of making a new one immediately as a substitute and either the new will is not made or it fails of effect for any reason. When the gift fails, the law presumes that testator would prefer the original will over intestacy, and evidence of intent is crucial.

1) Tendencies: Where the revocation is by act, the mistake is a mistake of law in that the testator attempted a new will or codicil that is invalid. Where the revocation is by writing, the mistake s a mistake of fact that must then be set forth in the valid revoking instrument.

2) LaCroix v. Senecal - Testator's will left part of residuary to nephew. Then, she executed a codicil, revoking the previous residuary clause, & replacing it with an identical one, except that she used nephew's given name, rather than his nickname as in the will. Problem was that the codicil was not properly executed. DRR applies here. It’s clear that the only reason for the change was to clarify the name of the nephew. The testator's intent to revoke the will was conditioned upon the clarifying terms of the codicil (to make sure nephew gets, using his legal name) and not to void the existing gift. Therefore, the testator would prefer revival.

1. On appeal, the court affirmed, holding that there was no room for doubt that the sole purpose of the testatrix in executing the codicil was, by making the very minor change in referring to her nephew, to eliminate any uncertainty as to his identity. Obviously, it was furthest from her intention to make any change in the disposition of her residuary estate. When the will and codicil were considered together, as they must be, to determine the intent of the testatrix, it was clear that her intention to revoke the will was conditioned upon the execution of a codicil which would be effective to continue the same disposition of her residuary estate. Therefore, when it developed that the gift under the codicil to the beneficiary was void, the conditional intention of the testatrix to revoke the will was rendered inoperative, and the gift to the beneficiary under the will continued in effect.

d) If a testator revokes a later will under the mistaken belief that by doing so a prior will is reinstated, DRR applies and renders the revocation ineffective and the later will is admitted to probate.

1) Estate of Alburn –The trial court had found that the decedent had first executed the Milwaukee will. Later she executed the Kankakee will. Before her death she tore up the Kankakee will mistakenly believing that by doing so she would revive the Milwaukee will. Decedent never executed another will prior to her death. The appellant argued that there was insufficient evidence to support the trial court's conclusions. On review, the court affirmed the trial court's judgment. The court held that there was uncontroverted testimony from the decedent's sister-in-law that decedent had told her she intended the Milwaukee will to stand, that the testatrix had indicated that she did not wish to

32 | L e v y

Page 33: Trust and Estates Outline

die intestate, that she took no steps after destroying the Kankakee will to make another will, and that both wills had similar provisions not providing for next-of-kin. On this basis it was not against the great weight and preponderance of the evidence that testatrix destroyed the Kankakee will in the mistaken belief that she was reviving the Milwaukee will. Thus, the doctrine of dependent relative revocation applied to support the probate of the Kankakee will.

1. Is the Milwaukee will revived? NO. b. Revival Remedy– refers to the reinstatement of a will that the testator has already revoked. A testator could trigger

revival by reexecuting the old will or by including an express statement in a new will that the old will is to be effective. Assuming a testator validly executes will #1, and thereafter validly executes will #2 that expressly or implicitly revokes will #1, and thereafter revokes will #2 intending to give effect to will # 1, jurisdictions are split over what the testator must do to revive will #1. Different approaches:

a) English common law – Will #1 was never really revoked so it could be probated. Will #2 would have revoked will #1 only if will #2 had remained in effect until the testator died.

1) Approach focuses on the principle that a will has no effect until testator’s death.b) American approach – No-revival approach. The general rule is that a will is effective the moment it is properly

executed. The moment the testator properly executes will #2, will #1 is revoked – it is null and void. Revoking will #2 does not automatically give effect to will #1.

c) Minority Approach – Will 2 revokes Will 1 when Will 2 is executed. Will 1 cannot be revived unless duly re-executed or republished by a later duly executed will (by going through the Wills Act formalities).

d) Majority approach/ Uniform Probate Code – Minority approach is too burdensome. All the testator has to do to revive will #1 is to intend to revive will #1. However, the key to proving the testator’s intent is how the testator revoked will #2.

1) §2-509 – If revocation of Will 2 by physical act : 1. Under section (a), if Will #2 wholly revoked Will #1, the revocation of Will #2 by a revocatory

(physical) act does not revive Will #1 unless the proponent of Will #1 shows that the decedent intended the revocation of Will #2 to revive Will #1.

2. Under section (b), if Will #2 only partially revoked Will #1 (i.e. Will #2 is a codicil to Will #1), then the revocation of Will #2 (codicil) by a revocatory (physical) act revives Will #1, unless the party arguing against revival shows that the decedent did not intend the revocation of Will #2 (codicil) to revive the revoked parts of Will #1.

3. Under section (c), if Will #2 revoked Will #1, and Will #2 is thereafter revoked by a later Will #3, Will #3 does not revive Will #1 unless the text of Will #3 indicates such a result is what the testator intended.

2) If revocation of Will 2 by subsequent writing:1. (Either wholly or partly) If Will 2 that revoked Will 1 is then revoked by Will 3, the Will 1

remains revoked, unless it is revived. Will 1 is only revived by the terms of Will 3e) Example: T gives B $100; $100 is crossed out & changed to $2000 If the jurisdiction recognizes partial

revocation, B would not get anything. But you might bring back the Will 1 (giving B $100) with DRR. Different Common Law approaches:

1) (English) Will 1 is not revoked unless Will 2 remains in effect until the testator's death. Therefore, revival of Will 1 is not necessary.

2) (Majority) Will 2 revokes Will 1 at the time of Will 2's execution and Will 1 can be revived if the testator so intends. Most commonly followed.

3) (Minority) Will 2 revokes Will 1 at the time of Will 2's execution but cannot be revived unless duly executed or republished

4. Revocation by Operation of Law: Change in Family Circumstances Revocation by Divorce – UPC § 2-804 | Revocation by Marriage – UPC § 2-301 |Revocation by Birth of Children – UPC § 2-302. Most states now have statutes providing that upon divorce, all provisions of a will during marriage in favor of ex-spouse are void.

a. Divorce - In a handful of states, statutes provide that a divorce revokes any provision in the decedent’s will for the divorced spouse. In the remaining states, revocation occurs only if divorce is accompanied by a property settlement.

a) Traditional approach (still the Majority approach)– Apply revocation by operation of law doctrine only to wills, not to the will substitutes – life insurance, joint tenancy, pension plans, and other nonprobate arrangements.

b) Modern trend/Uniform Probate Code §2-804 – UPC applies the revocation by operation of law doctrine not only to wills, but also will substitutes.

33 | L e v y

Page 34: Trust and Estates Outline

b. Marriage – if the testator executes her will and subsequently marries, statutes in a large majority of states give the spouse his intestate share, unless it appears from the will that the omission was intentional or the spouse is provided for in the will or by a will substitute with the intent that the transfer be in lieu of a testamentary provision.

c. Birth of Children - (Minority/CL) – marriage followed by birth of issue revokes a will executed before marriage. (Majority) – Pretermitted Child Statutes: If a child is born after execution of parent’s will, and has not been provided in the will, the child will.

C. COMPONENTS OF A WILL – 4 IMPORTANT DOCTRINES1. Integration of Wills The doctrine of integration addresses the question in the physical sense: which pieces of paper were meant to be in the will when it was executed? Integration is seldom a problem because usually all of the will’s pages are found stapled together, with the signatures at the end.

a. The scope of the will starts with the threshold issue of determining what constitutes the pages of the will. The doctrine of integration provides that those pieces of paper that are physically present at the time of execution and that the testator intend to be part of the will constitute the pages of the will.

b. External Integration – process of establishing the testator’s will by piecing together all of the testator’s wills, codicils, and other testamentary instruments.

c. Internal Integration – process of making certain that the testator’s will contains no fewer and no more pages that it did when the testator executed it.

d. All papers present at the time of execution, intended to be part of the will, are integrated into the will.A) In re Estate of Beale – Testator dictated will to his secretary; will consisted of 14 pages. Testator then took the

3 copies of the will, and had witnesses sign them. All pages had testator’s initials. Testator then had secretary re-type page 12 and 13 with some changes. Those pages also had testator’s initials. Court held that they would admit the will as it existed before the changes were made, b/c pages 12 and 13 were not present at execution.

B) Estate of Rigsby - two pages were not fastened together and the testator signed the first page, leaving two and a half inches of blank space below her signature. The court only admitted the first page, not the second, which could easily be interpreted as a work sheet listing Decedent’s assets as a preliminary step before drafting the first page.

e. Integration can bring in a document that has not been duly executed.2. Republication by Codicil

a. Republication refers to the process of treating an old will as if it were executed at a later date.b. Executing a codicil to a will “re-executes” and “republishes” the underlying will. A will is treated as if it were when its

most recent codicil was executed, whether or not the codicil expressly republishes the prior will, unless the effect of so treating it would be inconsistent with the testator’s intent.

a) Republication applies only to a prior validly executed will, while incorporation by reference can apply to incorporate into a will language or instruments that have never been validly executed.

c. Fundamental difference between republication by codicil and the doctrine of incorporation by reference is that republication applies only to a prior validly executed will, whereas incorporation by reference can apply to incorporate into a will language or instruments that have never been validly executed.

a) In some jurisdictions that don’t recognize incorporation by reference, courts have sometimes used republication to give effect to will that are invalid for some reason other than faulty execution.

1) New York – generally doesn’t permit incorporation of unattested documents into a will, but a codicil can republish (thus giving testamentary effect) a will that was invalid b/c of mental capacity or undue influence, but a codicil cannot republish a instrument that was never duly executed.

3. Incorporation by Reference Incorporation by reference is a way to give testamentary effect to a document not present at the execution ceremony. In most states, the following elements must be met for a document to be incorporated by reference:

The document being incorporated must exist at the time of the execution ceremony the will must indicate an intention to incorporate the will must refer to the document sufficiently to allow its identification, and the will must say that the document is in existence

Note: because the last element has proved troublesome in practice, UPC § 2-510 requires only the first three. a. Uniform Probate Code §2-510 – A writing in existence when a will is executed may be incorporated by reference if 1)

the language of the will manifests this intent and 2) describes the writing sufficiently to permit its identification.

34 | L e v y

Page 35: Trust and Estates Outline

a) A properly executed will may incorporate by reference into its provisions any document or paper not so executed and witnessed, if it was in existence at the time of the execution of the will and is identifiable by clear and satisfactory proof as the paper referred therein.

b) Clark v. Greenhalge – testator executed will in 1977, naming her cousin as executor. The will left everything to her cousin, except for items she designated by a memo & as she otherwise wished (this was referred to in the will). Memo was written in 1972, and modified in 1976. She also had a notebook where she made such entries. One of the entries in notebook gave a valuable painting to her friend; evidence that this was entered in notebook in 1980. In 1980, testator executed 2 codicils to her will. When she died, her cousin refused to give the painting to her friend. The court held that the notebook was incorporated by reference in the terms of the will. Although the document was not in existence at the time she executed her will, the court said the codicil was a republishing of the will, giving it effect in 1980, so the notebook predates the will, and is therefore incorporated by reference.

a. Simon v. Grayson- The testator's will left a sum of money to his executors to be paid as directed in a separate letter dated the same day as the will. A large portion of this money was to be paid to the decedent. A letter was found, but it had a different date than the will. No letter with the same date as the will was found. The decedent's executrix claimed the funds mentioned in the letter, and this was challenged by the residuary legatees under the testator's will. The trial court found in favor of the executrix, and on appeal, the court affirmed. Despite the discrepancy in dates, the evidence of identity was sufficient to overcome the effect of that discrepancy. The letter did not have to be identified with exact precision; it was enough that the words and circumstances combined to produce a reasonable certainty that it was the one referred to by the testator in his will. The letter was written by the testator, its terms conformed to the one described in the will, it identified itself as the letter mentioned in the will, and it dealt with the identical subject matter referred to in the will

b. Uniform Probate Code §2-513 - A will may refer to a written statement/list to dispose of personal property that is not specifically disposed of in a will (except money). The writing must be signed by the testator and must describe the items and devisees with reasonably certainty. The writing may be written before or after execution of will, may be altered, & may have no significance apart from its effect on the dispositions made by the will.

c. Johnson v. Johnson - The testator left a will that contained three typewritten paragraphs. At the bottom of the sheet of paper was a handwritten provision giving his brother only a nominal amount. The county court denied its admission to probate and the district court, after a de novo trial, affirmed that decision. The court reversed the decision. It noted that there was no question that the typewritten instrument was not signed, dated, or attested, that the testator intended that instrument to be his will, and that it made a complete disposition of his estate. There also was no question that the handwritten words were wholly in the testator's handwriting and were testamentary in character. The court found that the handwritten portion was a valid holographic codicil and that it incorporated the prior will by reference and republished and validated the prior will as of the date of the codicil giving effect to the testator's intention.

a) Testator typed his will – not attested, not dated, not duly executed. Then at the bottom, in his own handwriting, writes “to my brother I give $10 only. This will shall be complete unless hereinafter altered, changed or rewritten.” He signs & dates it (this would be a valid holograph). Court says this is a codicil that republishes the typewritten will, making the typewritten document a valid will.

b) Note: Prof says the statement of law is wrong here – a valid codicil only republishes a validly executed will. A codicil doesn’t create a will. It might have been possible to say the handwritten portion is a holographic will which incorporates by reference the typewritten portion.

4. Acts of Independent Significance - A will may dispose of a property by reference to acts or events that have significance separate and apart from the effect of their disposition made in the will. This is true even though the phrasing of the will leaves it in the testator’s power to alter the beneficiaries or the property by a non-testamentary act.

a. Uniform Probate Code §2-512 - A will may dispose of property by reference to acts and events that have significance apart from their effect upon the dispositions made by the will, whether they occur before or after the execution or the will or before or after the testator’s death. The execution or revocation of another individual’s will is such an event.

b. Example: T’s will devises “the automobile I own at my death” to B. At time will is executed, T owns a Toyota worth $4,000. T then buys a Cadillac worth $40,000, then dies. B gets the Cadillac. While T’s act in buying the Cadillac had the practical effect of increasing the gift to B, it is unlikely that this is what motivated the purchase; more likely that T bought Cadillac b/c T wanted a Cadillac

D. CONTRACTS RELATING TO WILLS A person may enter into a contract to make a will or a contract not to revoke a will. Contract law, not the law of wills, applies. To enforce a contract, the 3rd party beneficiary must sue under the law of contracts and prove a valid contract. If, after a contract

35 | L e v y

Page 36: Trust and Estates Outline

becomes binding, a party dies living a will not complying with the contract, the will is probated but the contract beneficiary is entitled to a remedy for the broken contract.1. Contracts to Make a Will

a. Contract Requirements: Contracts relating to wills – contracts to make a will (or provision) or not to revoke a will (or provision) must meet the standard contract requirements of offer, acceptance, and consideration.

a) SOF – to ameliorate problems if proof, many states now subject contracts to make a will to a SOF provision, thus requiring contracts to be in writing to be enforceable (Modern Trend).

1) Noncompliance with SOF – beneficiary, although not entitled to enforce the contract, may nonetheless be entitled to a restitution of the value to the decedent of services rendered (quantum meruit). The trick is that the beneficiary still must prove the contract.

b. Remedy: Constructive trust: If a contract concerning a will is established and the testator breaches the agreement by executing a different will, the probate court still probates the will the testator executed, but a constructive trust based on the contract typically is imposed on the testator’s probate property. The devisees are then ordered to give the property to the contract beneficiary.

2. Contracts Not to Revoke a Will: arises when a party agrees not to revoke a will or a provision in a will.a. Questions respecting contracts not to revoke a will typically arise where a husband and wife have executed a joint or

mutual wills.a) Joint wills - One will executed by two or more parties, typically husband and wife, as one testamentary

instrument.b) Mutual wills – Also known as mirror wills, are separate wills of two or more persons that contain reciprocal

provisions, executed individually.b. To discourage litigation about whether there was such a contract, UPC § 2-514 requires contracts concerning

succession to be in writing or, at the very least, to be referred to in a will and then proved by extrinsic evidence. Uniform Probate Code §2-514 provide that a contract relating to a will may be established only by:

a) Provisions of a will stating the material provisions of the contractb) An express reference in a will to a contract and extrinsic evidence proving the terms of the contract orc) A writing signed by the decedent evidencing the contractd) The execution of a joint will or mutual wills does not create a presumption of a contract relating to wills

c. Via v. Putnam - dispute between respondent, decedent's surviving spouse, who claimed a share of decedent's estate under the Florida pretermitted spouse statute and petitioners, the children of decedent's first marriage, who claimed that the mutual wills executed by their parents, naming them residuary beneficiaries of their parents' estates, gave rise to a creditor's contract claim that had priority against respondent's claim against the estate. The lower court held that respondent's right to receive either an elective share or pretermitted spouse's share of the decedent's estate had priority over petitioners' claims. The lower court further noted conflict with a previous decision. The court affirmed the decision of the district court, concluding that with the enactment of the statute, Florida evidenced a strong public policy concerning the protection of the surviving spouse of the marriage in existence at the time of a decedent's death. The court refused to amend § 732.301 and determined that, as third-party beneficiaries of a mutual will, petitioners did not have priority over the statutory rights of respondent, a surviving spouse.

a) Third party beneficiaries (contract creditors) do not take priority over statutory rights of a pretermitted spouse.

3. Analysis:a. Did the contract exist?

a) Express provisions in the will andb) Elements of contract law: Intent and (1) offer; (2) acceptance; (3) consideration

b. What are the terms of the contract?c. Were the terms breached?d. What are the damages?

________________________________________________________________________________________________________________CHAPTER 5. CONSTRUCTION OF WILL

A. MISTAKEN OR AMBIGUOUS LANGUAGE IN WILLS 1. The Traditional Approach: No Extrinsic Evidence, No ReformationA majority of jurisdictions still follow (or purport to follow) 2 traditional rules that bar the admission of evidence to vary the term of the wills: 1) plain meaning or no extrinsic evidence rule, 2) no reformation rule.

36 | L e v y

Page 37: Trust and Estates Outline

a. The plain meaning of a will cannot be disturbed by extrinsic evidence that another meaning was intended. Only when there is ambiguity can extrinsic evidence be admitted.

b. Restatement 3rd Property § 10.1 - “The controlling consideration in determining the meaning of a donative document is the donor’s intention. The donor’s intention is given effect to the maximum extent allowed by law.”

c. Plain meaning rule – extrinsic evidence is not admissible to show that the testator used the words to mean something other than their plain meaning. Extrinsic evidence is admissible only to construe a word or phrase in a will if there is ambiguity.

a) If there is ambiguity, the burden lies on the contestant of the will to prove by a preponderance of the evidence that the testator meant otherwise.

d. No reformation rule - The court will not reform a mistaken term in the will to reflect the testator’s intent. Reformation is an equitable remedy that, if applied to wills, would correct a mistaken term in the will to reflect what the testator intended to say.

a) Justification for refusing to reform wills – Courts are compelled to interpret the words the testator actually used and NOT the words that the testator is purported to have intended to use.

b) Mahoney v. Grainger – When the testatrix died her sole heir at law was her maternal aunt. The testatrix was a single woman of about 64. She lived alone but maintained cordial relationships with her aunt and several first cousins. The will's residuary clause provided for the residue to be distributed equally to the testatrix's heirs at law living at the time of her death; however, the real property was not to be sold for five years unless the personal property was insufficient to pay her specific legatees. The will gave sizeable general legacies to two first cousins. The probate court ruled that the testatrix's statements to the attorney were admissible only to show the material circumstances at the time of executing the will. Regardless of language that the heirs were to divide the residue among them equally, it was clear that the testatrix had but one heir. There was no latent ambiguity in the will permitting the testatrix's statements to be introduced to prove her testamentary intent. The court affirmed on appeal, finding no ambiguity in the will; the words referred alone to the aunt, not the cousins. The use of the plural word "heirs" did not prevent the aunt from taking the entire gift.

1) Court held it will not bring in extrinsic evidence that testator meant her cousins. The will document is clear & there is no ambiguity with what “heirs at law” means, so court will not look at extrinsic evidence.

2) Rule: It is only where testamentary language is not clear in its application to facts that evidence may be introduced as to the circumstances under which the testator used that language in order to throw light upon its meaning. Where no doubt exists as to the property bequeathed or the identity of the beneficiary there is no room for extrinsic evidence; the will must stand as written.

c) In Estate of Smith (Note’s case) – testator left to “Perry Manor Inc.” Perry Manor was a nursing home at the time of execution. Perry Manor, Inc. then sold the nursing home to another corp.; nursing home still called Perry Manor. The bequest went to the corp., although testator intended it go to the nursing home. Court says since there’s no ambiguity on the face of the will, extrinsic evidence to show testator’s intent excluded.

e. Note: Plain Meaning, Ambiguity, and Extrinsic Evidence a) Patent Ambiguity vs. Latent Ambiguity

1) Patent ambiguity – an ambiguity that appears on the face of the will and does not convey a sensible meaning to the reader. Increasingly, extrinsic evidence is allowed to aid in interpreting a patent ambiguity, HOWEVER, some jurisdiction still follow the traditional rule and won’t allow extrinsic evidence in to clarify a patent ambiguity (four corners of the will).

1. Example: will leaves a specific portion of estate to A, then leaves entire estate to B. 2) Latent ambiguity – an ambiguity that does not appear on the face of the will but manifests itself

when the terms of the will are carried out. Oral declarations of intent to scrivener of will be admitted in most jurisdictions (scrivener exception).

1. 2 types of latent ambiguity:a. Equivocation - When a will clearly describes a person or thing, but two or more

people exactly fit that description. Extrinsic evidence of direct expressions of testator’s intent allowed b/c it does not add anything to the will, it just made the terms more specific. Where there is an equivocation, direct expressions of the testator’s intent are admissible in evidence.

i. Ex.: a devise “to my niece Alicia,” when in fact the Testator has 2 nieces named Alicia.

ii. Personal use exception – If extrinsic evidence shows that testator always referred to a person in an idiosyncratic manner, the evidence is admissible

37 | L e v y

Page 38: Trust and Estates Outline

to show that the testator meant someone other than the person with the legal name of the legatee.

Moseley v. Goodman – Testator always referred Trimble as Mrs. Moseley, b/c her husband worked at Moseley’s cigar store. Court allowed the extrinsic evidence to mean Trimble, not the actual Mrs. Moseley – the wife of the store’s owner.

b. When no person or thing exactly fits the description, but two or more people or things partially fit the description. (more common)

i. Ihl v. Oetting – Testator devised his home to Mr. and Mrs. Wendell Hess residing at No. 17 Barbara Circle. Mr. Hess divorced his wife, sold the home at No. 17, and remarried. The new wife claimed Mrs. Hess’ share but the Court found latent ambiguity from the description “residing at No. 17 Barbara Circle”. Court stated that because "a latent ambiguity is only disclosed by extrinsic evidence, it may be removed by extrinsic evidence." "The type of extrinsic evidence contemplated is evidence of objective, operative facts ... which give precise and explicit meaning to the language used by the testator and, thus, compel a clear inference of the testator's exact intent."

b) Restatement 3 rd of Property: Wills and Other Donative Transfers § 11.2, comment d (2003) – Once an ambiguity, patent or latent, is established, direct as well as circumstantial evidence of the donor’s intention may be considered in resolving the ambiguity in accordance with the donor’s intention.

2. Slouching Toward Reformation: Correcting Mistakes Without the Power to Reform WillsA court has no power to correct or reform a will or change any of the language therein by substituting or adding words, but may disregard obviously mistaken references when necessary.

a. The Causes and Effects of Will Defects a) Extrinsic Evidence is also admissible to prove fraud, duress, and undue influence. The court will look

at evidence of the circumstances surrounding the will‘s execution in order to ascertain whether the will reflects the true wishes of the testator.

Effect: Lack of Volition Effect: Mistaken TermsCause: Intentional Wrongdoing Undue Influence, Duress (relief

granted)Fraud (relief granted)

Cause: Innocent Acts Lack of Capacity, Insane delusion (relief granted)

Mistake (no relief)

1) Fraud – if intentional wrongdoing causes a mistaken term in a will, the term can be struck or its effects undone.

2) Lack of capacity – if a lack of volition has an innocent cause, the will is not given effect.3) Insane delusion - if a bizarre mistaken belief about a member of the testator’s family influences the

testator’s dispositive scheme, the courts will remedy this mistake by calling it an insane delusion.b. Arnheiter v. Arnheiter - Decedent left a last will and testament that specifically devised a parcel of real property to

certain beneficiaries. The will referenced an erroneous house number for the street address describing the property. Plaintiff executrix applied to the court to correct the mistake and to change the street number in the will to the proper address. The court noted that it did not have the power to correct or reform a will or change any of the language in it by substituting or adding words. Instead, the court granted relief to plaintiff by construing the will provision under the doctrine falsa demonstratio non nocet, which holds that where a description of a thing or person consisted of several particulars and all of them did not fit any one person or thing, less essential particulars could be rejected provided the remainder of the description clearly fit. By applying this doctrine, the court dropped the house number and, because the decedent only owned one parcel of real property on the described street, and did not otherwise dispose of it in any other provision of the will, was able to conclude that the remaining description of the property was sufficient to identify the property.

a) Will referred to the testator’s interest in “304 Harrison Avenue,” but testator did not have any interest in 304, but actually 317 Harrison Avenue. Court will not reform (fix) the mistake on the will. Instead, though, Court scratched out the 304, and so it said “my interest in Harrison Avenue,” which was sufficient to identify the property.

c. Estate of Gibbs - A decedent left a will that purportedly left a devise to the stranger, who could not properly account for the reason for the bequest. However, but for a mistaken middle initial, the bequest would have gone to the friend, who had been left bequests in prior wills of the decedent. The trial court found that the decedent intended to leave the

38 | L e v y

Page 39: Trust and Estates Outline

bequest to the friend, not the stranger, and so ordered. The stranger appealed. The court first found that the stranger could not offer any logical reason why the decedent would have left anything to him in the will. However, the court observed that there was no ambiguity -- the stranger was the only person bearing the name of the legatee in the will. Ordinarily, where there is no ambiguity, courts would not reform wills. However, the court found where there were details subject to mistake, such as middle initials and addresses, courts overlooked such details in preference to the intent of the decedent. Here, the clear intent of the decedent was to leave property to the friend, not the stranger, despite the erroneous middle initial.

a) Will leaves property to Robert J. Krause, but the person testator intended to leave to was Robert W. Krause. Although court said you cannot reform a will and that there was no ambiguity in the will, it still corrected the mistake, explaining that small things like this are susceptible to mistake, so just disregard the middle initial, and the intended beneficiary is clear (the J guy was a stranger).

3. Openly Reforming Wills for MistakeIf a scrivener's error (lawyer’s mistake) has misled the testator into executing a will on the belief that it will be valid, extrinsic evidence of that error is admissible to establish the intent of the testator. Furthermore, if the scrivener's error and its effect on the testator's intent are established by clear and convincing evidence, they will be sufficient to establish that "provision has been made in such will for such contingency."

a. Erickson v. Erickson - Decedent prepared a will two days before his marriage to the executrix and provided that the executrix receive the estate. Although Conn. Gen. Stat. § 45a-257(a) provided that a will was revoked by marriage where there was no provision for the marriage, and the decedent's will did not provide for the marriage, the probate court admitted the will to probate. The court found that the will was improperly admitted because there was no language in the will providing for the contingency of the subsequent marriage. However, extrinsic evidence should have been admitted to establish the decedent's intent so that his will would be valid notwithstanding the subsequent marriage if a scrivener's error misled the decedent into executing a will on the belief that it would be valid despite the marriage. In order to rebut the presumption that the decedent did not intend for the will to survive his marriage, the executrix had to prove by clear and convincing evidence that the will did not substantially state the decedent's true intent and that the decedent intended for the will to survive the marriage.

a) Testator executed will 2 days before getting married. By law, the will is revoked once married, unless there's a contingency clause in the will. The will specifically mentioned that his property would go to the wife he was about to marry. The attorney who wrote the will knew the state law, but didn’t provide a provision in the will addressing it. Court held that a scrivener’s error that has misled the testator into executing a will on the belief that it will be valid is similar to relying on a fraudulent statement, so extrinsic evidence of that error is admissible.

b) Limited to cases where the lawyer screws up. b. Restatement 3 rd of Property: Wills and Other Donative Transfers § 12.1 - Reforming Donative Documents to Correct

Mistakes - An unambiguous donative document may be reformed to conform the test to the donor’s intent if it can be established by clear and convincing evidence (1) that a mistake of fact or law, whether in expression or inducement, affected specific terms of the document; and (2) what the donor’s intent was.

c. Uniform Probate Code §2-805 – The court may reform the terms of a governing instrument, even if unambiguous, to conform the terms to the transferor’s intention if it is proved by clear and convincing evidence that the transferor’s intent and the terms of the governing instrument were affected by a mistake of fact or law, whether in expression or inducement.

d. Langbein, Curing Execution Errors and Mistakene. Terms in Wills f. Fleming v. Morrison - testator formally executed a will leaving property to a girl in his will with the intention to trick her

into sleeping with him. Court said b/c the lawyer knew the will was drafted only for that reason and that testator did not intend to actually give her anything, court held will invalid.

a) Some courts have held a validly executed will to be invalid b/c of extrinsic evidence that testator did not intent to execute the will.

B. DEATH OF BENEFICIARY BEFORE DEATH OF TESTATOR Specific or general bequest Residue Class Gift

CL (min jurisdiction)

to residuary or intestacy No residue of residuary – to intestacy lapsed part

To the surviving class members

Anti-lapse (majority jurisdiction)

If devisee is descendant, then to substitute taker (issue) or intestacy

Lapsed part to substitute taker or intestacy (not other residuary)

To substitute taker or other class members

39 | L e v y

Page 40: Trust and Estates Outline

UPC If devisee is grandparent or lineal descendant of grandparent, then to substitute taker (issue)

Lapsed part to substitute taker or other residuary takers

To substitute taker or other class members

1. Introduction - To take under a will, the beneficiary must survive the testator. If a beneficiary doesn’t survive the testator, the devise lapses (fails). Lapse – death of beneficiary before testator

2. Common Law rules regarding lapsed devices . These are the default rules:a. Specific or general devise – If a specific or general devise lapses, the devise falls into the residue.b. Residuary devise – If the residuary devise lapses, the heirs of the testator take by intestacy.

a) No-residue-of-a-residue rule - If only a share of the residue lapses, such as when one of 2 residuary devisees predeceases the testator, at common law the lapsed residuary share passes by intestacy to the testator’s heirs rather than to the remaining residuary.

c. Class gift – If the devise is to a class of persons, and one member of the class predeceases the testator, the surviving members of the class divides the gift.

d. Void devise – Where a devisee is already dead at the time the will is executed, or the devisee is a dog or cat or some other ineligible taker, the devise is void. Voided gifts are treated the same as lapsed gift

a) Void - invalid gift (to dog) or if beneficiary dead at time of executionb) Estate of Russell - Decedent's will stated the her dog was to receive one-half her residuary estate and the friend

was entitled to receive the other half. In the heir's suit to challenge the gift, the trial court admitted, over the heir's objection, the friend's extrinsic evidence that tended to show that the decedent did not want to die intestate. The trial court attempted to carry out the decedent's intent and ruled that decedent intended to leave her entire estate to the friend with the hope that he would care for the dog. On appeal, the court ruled that because the language of the will was clear and unambiguous, the trial court erred by admitting the friend's extrinsic evidence. The court ruled that the intended gift to the dog was invalid because a dog could not be a beneficiary under a will. Moreover, the court ruled, again because the will was clear and unambiguous, a trust such as the one found by the trial court was not intended by the decedent and was thus improperly imposed. Through intestate succession, the heir was entitled to receive the one-half interest intended for the dog.

c) Small minority of states still follow the no-residue-of-the-residue rule. In a vast majority that have rejected the rule, assumption is the testator would probably prefer for the other remaining residuary devisee to receive the entire residue.

e. Analysis: determine what type of gift it is:a) If gift is specific, general or demonstrative → goes to residue or if no residue, to intestacy b) If gift is residue → lapsed part to intestacy (no residue of residue rule)c) If class gift → to surviving members of the class (not intestacy)

3. Antilapse Statutes – a typical antilapse statute provides that if a beneficiary is of a specified relationship to the testator and is survived by descendants who survive the testator, the descendants are substituted for the predeceased beneficiary. This changes the common law to give the predeceased beneficiary’s gift to the beneficiary’s descendants unless the testator provides otherwise.

a. Traditional antilapse statutes - Because antilapse statutes vary in detail, it can be helpful to have a series of questions you can ask about whatever statute you face:

a) Is the predeceased beneficiary in a class protected by the statute? The answer depends upon the relationship between the testator and the beneficiary. Some jurisdictions protect gifts to all beneficiaries; some apply only to gifts to relatives; some only cover gifts to descendants.

b) Did the beneficiary die before the will was executed? Under the common law, a gift to someone dead when the will was executed did not lapse; rather, it was “void.” Most statutes expressly cover both lapsed and void gifts, but some do not.

c) Did the beneficiary leave survivors who qualify under the statute to take the gift? This time the key is the relationship between the predeceased beneficiary and that person’s survivors. If appropriate survivors exist, they will take the gift. By far the most common substitute takers are issue of the beneficiary.

d) How does the document’s language affect the statute? Antilapse statutes will yield to contrary expressions of intent. Default rules – antilapse statute designed to implement presumed intent unless the will expressly states the testator’s actual intent.

1) Allen v. Talley - Testator wrote her will "to my living brothers and sisters." At time she executed the will, she had 5 siblings. At time of death, all siblings had predeceased her except one brother. The other siblings left surviving children. So if anti-lapse applied, the siblings’ issue would get too; otherwise only brother gets. Courts says the language “living” are words of survivorship, so no anti-lapse. Only living brother gets.

40 | L e v y

Page 41: Trust and Estates Outline

2) Survivorship- Authorities sometimes say a mere survivorship requirement is enough to override the statute. Estate of Rehwinkel. The law is not clear, however, so relying upon that approach is risky. UPC § 2-603, comment.

3) Class gifts - When the statute is not clear, courts are divided. The Restatement’s position is that antilapse statutes do apply to single-generation class gifts (e.g., children), but not to multiple-generation gifts (e.g., descendants). See Restatement (Third) of Prop. § 5.5 cmt. j.

4) Powers of Appointment - When the testator is exercising a general power, courts usually will apply an antilapse statute, if its other terms are met. On the other hand, if the power is special, courts may refuse to apply the statute.

5) Does the statute apply to documents other than wills? By their terms, virtually all antilapse statutes apply only to wills. Some authorities extend the coverage to will substitutes.

b. Presumed intent – theory is that for certain predeceasing devisees, the testator would prefer a substitute gift to the devisee’s descendants rather than for the gift to lapse. Scope – an antilapse statute applies to a lapsed devise only if the devisee bears the particular relationship to the testator specified in the statute.

c. Uniform Probate Code §2-605 – Antilapse; Deceased Devisee; Class Gifts - If a devisee who is a grandparent or a lineal descendant of a grandparents of the testator is dead at the time of execution of the will, fails to survive the testator, or is treated as if he predeceased the testator, the issue of the deceased devisee who survive the testator by 120 hours take in place of the deceased devisee and if they are all of the same degree of kinship to the devisee they take equally, but if of unequal degree then those of more remote degree take by representation. One who would have been a devisee under a class gift if he had survived the testator is treated as a devisee for purposes of this section whether his death occurred before or after the execution of the will.

d. Ruotolo v. Tietjen - The will's residuary clause bequeathed half of the estate residue to a legatee, if she survived the decedent. The legatee died shortly before the decedent. The beneficiary was the child of the deceased legatee. The probate court concluded that, as Conn. Gen. Stat. § 45a-441 was not operative, the bequest to the legatee lapsed and passed to the intestate estate. The appellate court held that the antilapse statute was enacted to prevent operation of the rule of lapse and unintended disinheritance. Therefore, words of survivorship such as "if she survives me" alone did not constitute a "provision" in the will for the contingency of the death of a beneficiary, as the statute required, and thus were insufficient to negate operation of Conn. Gen. Stat. § 45a-441. Should a testator desire to avoid application of the antilapse statute, the testator had to either unequivocally express that intent or simply provide for an alternate bequest. Because the testator in the present case did neither, the appellate court held that the protections of the antilapse statute applied. Accordingly, the bequest to the legatee did not lapse, but rather descended to the beneficiary.

a) Under Connecticut law, the antilapse statute applies unless a provision has been made in the will for such contingency.

b) Antilapse statutes will apply unless testator's intention to exclude its operation is shown with reasonable certainty. This is to establish a strong rule of construction, designed to carry out presumed intention based on the constructional preference against disinheriting a line of descent. Consequently, these statutes should be given the widest possible sphere of operation and should be defeated only when the trier of fact determines that the testator wanted to disinherit the line of descent headed by the deceased devisee. Hence, the burden is on those who seek to deny the statutory protection rather than on those who assert

c) In order to prevent application of the antilapse statute a testator must clearly and unequivocally indicate his intent that the statute not apply. Words of survivorship, such as "if she survives me," alone do not constitute a "provision" in the will for the contingency of the death of a beneficiary, as the Connecticut antilapse statute requires, and thus are insufficient to negate operation of the antilapse statute.

d) The testator is presumed to know the law and that his will is drawn accordingly.e) To prevent operation of the antilapse statute it is necessary that the testator, in apt language, make an

alternative provision in his will providing that in the event such relative predeceases or fails to survive the testator such devise shall be given to another specifically named or identifiable devisee or devisees.

e. UPC § 2-603 expands and clarifies antilapse statutes while covering class gifts, testamentary exercises of powers of appointment, and void gifts. (3) also provides that the anti-lapse statute applies despite a testator’s use of express survivorship language. Sections 2-706 and 1-201 extend antilapse protection to a variety of will substitutes. Its most ambitious and controversial step is to clarify the rules surrounding what language will preclude the application of the statute and who the alternate takers ought to be in various circumstances. The language “if she survives me,” without more, is not enough to prevent the statute from substituting alternative takers if the named beneficiary does not survive. Only if testators clearly express another intention can they avoid the statute’s solutions. The best way to avoid the statute is to provide an alternative gift.

41 | L e v y

Page 42: Trust and Estates Outline

f. Analysis: a) First, determine whether anti-lapse statue applies:

1) There must be a protected relationship with the testatora. Majority – devisee is a descendant of testator. Some also permit testator's siblings, other

relatives, or relative of spouse (but not spouse).b. 9 states – no requirement for protected relationship.c. UPC – grandparents & lineal descendants of grandparents. 1990 UPC also includes

stepchildren.2) There is a substitute taker

a. Majority – lineal descendants (surviving issue of deceased devisee)b. 3 states allow any heir (Iowa, Maryland, New Hampshire)c. UPC – issue of devisee who survive testator by 120 hours

3) There are no express words of survivorship that preclude use of anti-lapse statuteb) If anti-lapse applies:

1) If gift is specific, general or demonstrative → to substitute taker or intestacy2) If gift is residue → lapsed part to substitute taker or intestacy (not other residuary)3) If class gift → to substitute taker or other class members

4. Class Gifts - disposition to beneficiaries described as a group label, not by specific names; membership in class may fluctuate “to my children,” etc. Generally, naming an individual in a bequest prevents the gift from becoming a class gift.g. Restatement (Third) of Property: Wills and Other Donative Transfers §§13.1, 13.2 h. Dawson v. Yucus - Will leaves property as follows: ½ to nephew Wilson, ½ to nephew Burtle; there is also a residuary

clause. Burtle dies before testator. If class gift, all goes to Wilson (other class members); if not, then to residuary. Court says this is not a class gift b/c she specifically stated the names of the nephews that would get. So share goes to residuary.

i. Note: Application of Antilapse Statutes to Class Gifts

C. CHANGES IN PROPERTY AFTER EXECUTION OF WILL 1. Ademption by Extinction - refers to the failure of a specific gift because property is not in the testator's estate when the

testator dies.a. Types of devises:

a) Specific Devise – disposition of specific piece of testator’s property (my diamond ring)b) General Devise – when testator intends to confer a general benefit and not confer a specific asset (money, a

diamond ring)c) Demonstrative Devise – general devise, payable from a specific source. d) Residuary Devise – whatever is left over

b. What happens if a will includes a specific devise, but that specific item is not in the testator’s estate at death (testator sold it or gave it away)?

a) Ademption by Extinction -Only applies to specific devises. There are two approaches – identity and intent.1) Identity theory (common law) – if the specific gift is not in the decedent’s estate, the gift is

extinguished. If the property is not in the estate, the beneficiary gets nothing.1. 1969 UPC 2-609 – identity theory, but with 5 exceptions for beneficiary to still get if specific

property not in testator’s estate: i. the remaining balance on purchase price of specific property sold

ii. unpaid amount of condemnation awardiii. unpaid insurance proceeds after destructioniv. property owned by testator as a result of foreclosing on a mortgagev. sale price of specifically devised property by conservator.

2) Intent theory (newer) – if the specific gift is not in the decedent’s estate, the beneficiary may still be able to get the cash value of the specific item, if he can show this is what testator wanted. Ask why the property is not in the estate? What would the decedent wanted?

1. 1990 UPC 2-609: lean towards intent theory. Devisee gets:i. the specifically devised property

ii. the amount generated (paid or unpaid) by its sale or destructioniii. the property owned by the testator acquired as a replacement; oriv. if devisee recovers nothing under the above factors, the value of the specifically

devised property unless evidence indicates the testator intended ademption.

42 | L e v y

Page 43: Trust and Estates Outline

v. Note also: Under this provision, where property was sold or destroyed by a conservator, the devisee has a right to a general pecuniary devise equal to the net sale price of the specified gift.

b) In re Estate of Anton -The stepdaughter deeded a piece of property to her stepmother and father. The father and stepmother built a duplex on the property. After the father's death, the stepmother became the sole owner of the duplex. In the stepmother's will, she bequeathed half of her interest in the duplex to the stepdaughter. The stepmother later executed a durable power of attorney authorizing her daughter to manage her financial affairs. The daughter sold the duplex to pay for the stepmother's nursing home expenses. The net proceeds of the sale were $ 133,263. At the time of the stepmother's death, the remaining balance was $ 104,317. The stepdaughter filed a claim with the estate, asserting that she was entitled to $ 72,625 because of the bequest. The daughter, acting as executor of the estate, denied the claim. The trial court denied the claim. The court of appeals upheld the decision. On review, the court held that the sale of the duplex did not result in ademption of the bequest. The stepmother did not know that the duplex was going to be sold; she had only a general knowledge that assets might need to be sold for her support at some time in the future. The stepdaughter was entitled to $ 52,158.

c. The identity theory can be harsh when assets are missing even though testators have taken no action to remove the assets from the estate. In response, courts have developed avoidance devices:

a) Trace assets when someone acting on behalf of the incompetent has transferred property.b) Manipulate the general/specific distinction to avoid the ademption issue altogether. c) Apply a “change-in-form” rule.

d. Uniform Probate Code §2-606 - Nonademption of Specific Devises; Unpaid Proceeds of Sale, Condemnation, or Insurance; Sale by Conservator or Agent - adopts an intention-driven rule, expands the change-in-form rule, details specific situations in which tracing is appropriate, and sets up a presumption against ademption by extinction.

a) A specific devisee has a right to the specifically devised property in the testator's estate at death and:1) any balance of the purchase price, together with any security agreement, owing from a purchaser to

the testator at death by reason of sale of the property;2) any amount of a condemnation award for the taking of the property unpaid at death;3) any proceeds unpaid at death on fire or casualty insurance or on other recovery for injury to the

property;4) property owned by the testator at death and acquired as a result of foreclosure, or obtained in lieu of

foreclosure, of the security interest for a specifically devised obligation;5) real or tangible personal property owned by the testator at death which the testator acquired as a

replacement for specifically devised real or tangible personal property; and1. (replacement property – must be used in the same manner as the original property)

6) (Residual/catch-all) if not covered by paragraphs (1) through (5), a pecuniary devise equal to the value as of its date of disposition of other specifically devised property disposed of during the testator's lifetime but only to the extent it is established that ademption would be inconsistent with the testator's manifested plan of distribution or that at the time the will was made, the date of disposition or otherwise, the testator did not intend that the devise adeem.

1. Limits: (Burden of proof is on the person/specific devisee trying to avoid ademption to prove that the testator would not have wanted ademption) (Presumption in favor of ademption).

b) If specifically devised property is sold or mortgaged by a conservator or by an agent acting within the authority' of a durable power of attorney for an incapacitated principal or if a condemnation award, insurance proceeds, or recovery for injury to the property' are paid to a conservator or to an agent acting within the authority of a durable power of attorney for an incapacitated principal, the specific devisee has the right to a general pecuniary devise equal to the net sale price, the amount of the unpaid loan, the condemnation award, the insurance proceeds, or the recovery. (categorical exception)

c) The right of a specific devisee under subsection (b) is reduced by any right the devisee has under subsection (a).

2. Aunt Fanny has a collection of snuff bottles, some of which may be worth a lot of money, all of which are rare. Aunt Fanny bought hers in the 1950s through the 1970s, one at a time. She kept no records and they were not insured. She bequeaths some of them to Wendy. At her death, they are not found in her house. No one knows how many there were. What are Wendy’s rights under common law? UPC?

a. Identity rule (Massachusetts)- clearly tells us that this is a specific devise that is no longer there and thus is adeemed.b. UPC intent rule however gives a presumption that the devisee is intended to receive the value of the bottles. If a devise

is against a testator’s intent then it should be adeemed. If there is intent to give it away, and the property is lost or

43 | L e v y

Page 44: Trust and Estates Outline

stolen then the devisee should get the property3. Stock Splits and the Problem of Increase 4. Ademption by Satisfaction – failure of a testamentary gift because the testator has already transferred the property to the

beneficiary between the time of will execution and time of death. a. Satisfaction of General Pecuniary Bequests - Generally applies to general devises, but might also apply to

demonstrative and residuary gifts. (Analogous to advancement that applies to intestate succession)b. Common law – doctrine applied only to gifts of personal property.c. Modern trend – permits gifts of real property to be satisfied as well as personal property. d. Intervivos Transfer and general gifts:

a) Intervivos transfer 1) CL: - Ejusdem generis – the character of the testamentary gift and inter vivos gift has to be the same

before satisfaction takes place.2) Modern Trend: Intervivos transfer – testamentary gifts may be satisfied by a wide variety of inter

vivos transfers.3) If the testator makes an intervivos transfer to the devisee after executing the will, under Common

law there is a rebuttable presumption that the gift is in satisfaction of the gift made by the will.1. Ex: T’s will gives $50,000 to A. After executing will, T gives A $30,000. Presumption that this

gift was in partial satisfaction of will’s bequest, so A gets $20,000 at T’s death.4) Under UPC 2-609 and most states , no presumption. Requires evidence of intention of testator to

adeem by satisfaction. Unless the decedent says so in writing or the will itself says so. 5. Changes in Value – the change in value of specifically gifted property between the time of will execution and time of death is

not considered when distributing the testator’s property. Beneficiary of a specific gift receives that item, regardless of any changes to its value. If testator is not content with this result, express directions to the contrary must be included in the will.

6. Exoneration of Liens - When a will makes a specific devise of property that is subject to a mortgage, it is presumed that the testator wanted to debt to be paid out from the residuary estate. This is subject to language in the will indicating otherwise.

a. Exoneration decrees that a specific gift transfers free from any outstanding mortgage or lien, which then has to be paid by the residuary estate. Some jurisdictions recognize the doctrine, while others do not.

b. Common law – exoneration presumed; free and clear of the mortgage.c. Many states and § 2-607 - no right of exoneration unless there is express language in the will that says it passes free

from creditors’ encumbrances (or it can also say that it is subject to them).7. Abatement - When the estate has insufficient assets to pay debts as well as all devises; some devises must be abated (reduced

or eliminated to pay an obligation of the estate or a testamentary gift of higher priority). Abatement addresses what to do when there is not enough money to go around. Typically, gifts to will beneficiaries “abate,” or fail in the face of inadequate funds, in the following order: (1) residuary, (2) general, and (3) demonstrative and specific as a single class. Multiple gifts in the same class abate proportionately.

a. Order of abatement: This is default only under CL and UPC– only applies unless will indicates otherwise:a) Property passing via intestate succession(if T dies partially intestate)b) Residuary – divided pro ratac) General– divided pro ratad) Specific/demonstrative – divided pro rata

b. UPC - However, if the testamentary plan would be defeated by the usual order of abatement, the order of abatement may change to give effect to testator’s intent.

c. Common law – requires personal property in each category to be exhausted before real property from that category may be used.

d. Other states and UPC 3-902– no longer make a distinction between real and personal property.________________________________________________________________________________________________________________CHAPTER 7. RESTRICTIONS ON THE POWER OF DISPOSITION: PROTECTION OF THE SPOUSE AND CHILDREN

A. RIGHTS OF THE SURVIVING SPOUSE 1. Introduction to Marital Property SystemsCommunity Property Separate Propertya. All property (including earnings) acquired during the

marriage is community property, unless both spouses agree to separate ownership.

b. Sharing of acquisitions as equals in marital economic

a. No automatic sharing of earnings; whatever individual earns is his or hers.

b. Protection against disinheritance provided through elective share.

44 | L e v y

Page 45: Trust and Estates Outline

partnership. c. Individual autonomy over acquisitions.a. 2 Basic marital property:

a) Separate Property – Common law of England – husband and wife own separately all property each acquires except those items one spouse has agreed to put into joint property with the other.

1) Whatever the worker earns is his or hers. b) Community Property - Community property states have created a form of property ownership that

recognizes the mutuality of marital relationships. In particular, rather than treat spouses’ earnings as the separate property of each, this doctrine lumps together “the fruits of the marriage” and calls them “community property.”

1) AZ, CA, ID, NV, NM, TX, WA, WS are community property states.2. Rights of Surviving Spouse to a Share of Decedent’s Property

a. The Elective Share and Its Rationale – All but one of the separate property states give surviving spouse an elective (forced) share. Underlying Policy - the surviving spouse contributed to the decedent’s acquisition of wealth and deserves to have a portion of it; also to provide spouse with adequate support. SS must evaluate whether to take the share the deceased spouse intended the SS to receive under the will or to disregard that intent and choose the statutory share.

a) UPC §2-211 – SS must be certain to make the election within the period of time specified in the appropriate statute (later of (1) nine months after the deceases spouse’s death, or (2) six months after the deceased spouse’s will was admitted to probate).

b) Commonly used schemes :1) Elective share law in common-law states - is 1/3 of all of the decedent spouse’s probate property plus

certain non-probate transfer the 50 percent share of the couple's combined assets that the partnership theory would imply.

a. Many statutes apply the elective share formula on an augmented state rather than the net probate estate.

1. Augmented estate may contain the value of nonprobate assets such as the deceased spouse’s share of jointly held property passing because of rights of survivorship and life insurance proceeds that are not payable to SS.

2. Policy – prevents deceased spouse from reducing the SS’s elective share by using probate avoidance techniques to dispose of the property.

2) UPC (1990) approach – gives the surviving spouse a sliding-scale percentage of the elective share amount, based on the duration of the marriage (>1yr=3%, >15yrs=50%)

3) Partnership theory - (Uniform Probate Code, Article II, Part 2, General Comment) award the surviving spouse 1/2 of the decedent’s property acquired during the marriage. Wife’s share is her share and she would be able to pass on to her heirs 1/3 of the probate estate.

a. Under this approach, the economic rights of each spouse are seen as deriving from an unspoken marital bargain under which the partners agree that each is to enjoy a half interest in the fruits of the marriage, i.e., in the property nominally acquired by and titled in the sole name of either partner during the marriage (other than in property acquired by gift or inheritance).

4) Support theory – The court may elect for the surviving spouse to take against the will and take her intestate share instead, if it finds that it is necessary to provide adequate support for the surviving spouse. Requires support only during life.

c) Traditional statutes provide the surviving spouse with an election:1) The spouse can take under the decedent’s will; or2) The spouse can renounce the will and take a fractional share of the decedent’s estate.

d) Contrast with Intestate Share - Intestate deals with when there's no will 1) You're entitled to half if there are mutual children; if parent & no children, etc. 2) Only getting probate assets – but, some jurisdictions start to include nonprobate assets b/c testator

would transfer assets to nonprobate so they don’t leave to surviving spouse intentionallye) Intestacy – subject to the control of the property.

b. Property Subject to the Elective Share a) Judicial responses - Assets held in an inter vivos trust are not part of the estate for purposes of the

surviving spouse’s elective share, as they are nonprobate assets.1) Sullivan v. Burkin - Plaintiff widow exercised her right to take a share of her husband's estate. She

sought a determination that assets held in an inter vivos trust created by her husband during the

45 | L e v y

Page 46: Trust and Estates Outline

marriage should be considered as part of the estate in determining that share. The probate court dismissed the complaint. The widow appealed. The widow's claim was that the inter vivos trust was an invalid testamentary disposition and that the trust assets constituted assets of the estate. The court concluded that the trust was not testamentary in character and that the husband effectively created a valid inter vivos trust. The assets of the inter vivos trust were not to be considered in determining the portion of the estate of the deceased in which the widow had rights. The surviving spouse was denied any claim against the assets of a valid inter vivos trust created by the deceased spouse, even where the deceased spouse alone retained substantial rights and powers under the trust instrument. For the future, however, as to any inter vivos trust created or amended after the date of the opinion, the estate of a decedent, for the purposes of § 15, was to include the value of assets held in an inter vivos trust created by the deceased spouse as to which the deceased spouse alone retained the power during his or her life to direct the disposition of those trust assets for his or her benefit. The widow here obtained no right to share in the assets of that trust when she made her election under § 15.

a. A trust with remainder interests given to others on the settlor's death is not invalid as a testamentary disposition simply because the settlor retained a broad power to modify or revoke the trust, the right to receive income, and the right to invade principal during his life.

1. Restatement (Second) of Trusts § 57 comment h (1959) - a trust is "not testamentary and invalid for failure to comply with the requirements of the Statute of Wills merely because the settlor-trustee reserves a beneficial life interest and power to revoke and modify the trust. The fact that as trustee he controls the administration of the trust does not invalidate it."

b. Looked at whether the widow has special interests which should be recognized, even if the trust was not testamentary on general principles.

c. The rule for the future: assets of an inter vivos trust created during the marriage by the deceased spouse over which he or she alone had a general power of appointment would be part of the estate of the deceased exercisable by a deed or by the will. Objective test will not consider motive or intention of the spouse in creating the trust .

d. Rights of surviving spouse of decedents “Estate” – best handles by the legislatione. Prof says: not really equitable, but court doesn’t want to go against precedent. In divorce,

the transfer would be counted as marital prop & wife could still take. Doesn’t make sense that entitled to it at divorce, but not at death.

2) Three different tests to determine when nonprobate transfers are subject to elective share:a. Illusory Transfer - court looks at whether it's a good faith transfer or divestment, or made

solely to disinherit spouse.1. Newman case held that an illusory revocable trust is a valid trust, but it counts as

part of the decedent’s assets subject to the elective share – the trustee may have to contribute some of the trust assets to make up the elective share.

2. Most widely adopted test for subjecting nonprobate property to the elective share. b. Intent to Defraud - court looks at the state of mind of the person making an inter vivos

transfer - whether transferor intended to defraud surviving spouse. Some Look for subjective intent while others look for objective evidence of intent.

c. Present Donative intent - looks at whether the transferor intended to make a present gift3) In most states, a revocable trust created by the decedent spouse is included in determining the

surviving spouse’s elective share. Restatement 3rd of Property §9.1, comment j.4) Trusts created from assets acquired outside of marriage not eligible for the elective share.

a. Bongaards v. Millen - mom created trust for benefit of her daughter. Daughter had a limited power of appointed. Daughter then doesn’t leave trust to her husband, but to her sister. Husband wants his forced elective share. Court says not an asset for elective share purposes – asset not created during marriage, was not created by the daughter but the daughter’s mother.

b. Sullivan only applies to assets of a trust created during marriage by the deceased spouse. b) Statutory schemes

1) NY – specifically lists which nonprobate transfers subject to the elective share2) Delaware – property subject to elective shares is all property includible in the decedent’s gross

estate under fed estate tax

46 | L e v y

Page 47: Trust and Estates Outline

3) 1969 UPC – surviving spouse entitled to 1/3 of augmented estate, which includes the probate estate and certain nonprobate & made without consideration during the marriage.

a. Example #1: D died in 2000 survived by spouse and children. Probate estate = $10K. His will leaves all of his property to his children.

1. During the marriage: Titled real estate in JT w/ his children (D paid 100% of PP) = $90K. Created a revocable inter vivos trust naming himself as sole income beneficiary, his children as remainder beneficiaries, reserving the right to revoke the trust and naming himself as trustee = $11K. Purchased life insurance w/ children as beneficiaries = $10K

2. Traditional Elective Share Statute – 1/3 of probate estate. SS would take: 1/3 * $10,000 = $3,333

3. 1969 UPC elective share amt. = 1/3 of Augmented Estatea. Probate estate = $10Kb. Owns real estate in JT w/ his children = $90Kc. Revocable trust - $11Kd. 1/3 * $111K = $37,000

4. Life insurance w/ children as beneficiaries is exempted.b. Example #2: D died in 2000 survived by spouse and children. Probate estate = $10K (will

leaves all of his property to his children)1. During marriage: Titled real estate in JT w/ his SS (D paid 100% of PP) = $90K.

Created a revocable inter vivos trust naming SS as sole income beneficiary, giving SS a general power of appointment over trust assets = $11K. Purchased life insurance w/ SS as beneficiary = $10K.

2. Traditional Elective Share Statute – 1/3 of probate estate. SS would take: 1/3 * $10,000 = $3,333

3. 1969 UPC elective share amt =1/3 of Aug. Estatea. Probate estate = $10Kb. Owns real estate in JT w/ SS = $90Kc. Revocable trust - $11Kd. Insurance payable to SS = $10Ke. 1/3 * $121K = $40,333

c. 1969 UPC § 2-207: Credit Spouse with Property Received Gratuitously from D during D Lifetime

1. Elective Share Amt. = $40,333a. SS is credited with value of property already received

JT property - $90K Rev. trust property - $11K Life Insurance - $10K

$111K2. Since property already received ($111K) > elective share ($40,333)

b. SS would not be entitled to any contribution from any recipient of property from the augmented estate

4) 1990 UPC – more like community property system – all up all property of both spouses and slit it according to a percentage based on length of marriage.

a. Life insurance included.5) 2008 UPC - 4 Step Approach 2008 UPC:

a. Determine the value of the augmented estate. § 2-203 (a) = add up value of § 2-204 – § 2-207).

1. Net probate estate - § 2-2042. Nonprobate transfers to others (other than SS) - § 2-2053. Nonprobate transfers to SS - § 2-2064. SS net assets at D death + SS nonprobate transfers to others - § 2-207

b. Determine the “Marital Property Portion” of the Augmented Estate - § 2-203(b) – 1. [augmented estate (step #1)] * [applicable § 2-203(b) %]

c. Calculate elective share amount - § 2-202(a) –

47 | L e v y

Page 48: Trust and Estates Outline

1. [marital property portion of augmented estate (step #2)] * 50%d. Satisfy the elective share amount. § 2-209 –

1. Credit SS with property SS already owns or has received from D, including:a. Amounts includable in augmented estate under § 2-204 (SS share of

probate estate under will or by intestacy)b. Amounts includable in augmented estate under § 2-206 (D nonprobate

transfers benefiting the SS)c. Marital property portion of amounts includable in augmented estate under

§ 2-207 (property owned by SS) - [value of amts. Incl. in augmented estate under § 2-207] * [applicable § 2-203(b) %]

2. Satisfy remaining elective share amount balance with assets from D probate estate & D nonprobate transfers to others.

e. Example #1: A & B married 20 years. A died and left nothing to B. A’s died owning a house, titled in his own name, valued at $300K, and a retirement plan (401(k)) valued at $100K, with his daughter Jane named as sole beneficiary. B owned $200K in her own name.

1. Determine the value of the augmented estate. § 2-203 (a) = add up value of § 2-204 – § 2-207):

a. Net probate estate - § 2-204 = $300Kb. Nonprobate transfers to others (other than SS) - § 2-205 = $100Kc. Nonprobate transfers to SS - § 2-206 = $0d. SS net assets at D death + SS nonprobate transfers to others - § 2-207

= $200Ke. Augmented estate = $600K

2. Determine the “Marital Property Portion” of the Augmented Estate - § 2-203(b)a. [augmented estate (step #1)] * [applicable § 2-203(b) %]b. = $600K * 100% = $600K

3. Calculate elective share amount - § 2-202(a).a. [marital property portion of augmented estate – step #2] * 50% b. =$600K * 50% = $300K

4. Satisfy the elective share amount. § 2-209.a. Credit SS with property SS already owns or has received from D, including:

i. Amounts includable in augmented estate under § 2-204 (SS share of probate estate under will or by intestacy) = 0

ii. Amounts includable in augmented estate under § 2-206 (D nonprobate transfers benefiting the SS) = 0

iii. Marital property portion of amounts includable in augmented estate under § 2-207 (property owned by SS) = $200K

1. [value of amts. Incl. in augmented estate under § 2-207] * [applicable § 2-203(b) %]

2. = $200K * 100% = $200Kb. Satisfy remaining elective share amount balance with assets from D

probate estate & D nonprobate transfers to othersi. B entitled to $100K from Jane

1. Elective share amt ($300K) – amt owned by B ($200K).c) The Uniform Probate Code §§2-202, 2-203

c. Waiver – spouse can waive all or part of right to spousal support, gifts under previously executed will, and elective share. Usually done through pre-nuptial agreement but can be done through post-nuptial agreements (waiver agreed to during the marriage).

a) In just over half states, the enforceability of a premarital agreement is governed by the UPAA. The Uniform Premarital Agreement Act overrides the presumption of fraud that some courts attached to a premarital agreement if the agreement made inadequate provision for a spouse in light of the other spouse’s wealth.

b) Uniform Probate Code §2-213 - waiver of right to elect and of other rights1) (a) The right of election of a surviving spouse and the rights of the surviving spouse to homestead

allowance, exempt property, and family allowance, or any of them, may be waived, wholly or

48 | L e v y

Page 49: Trust and Estates Outline

partially, before or after marriage, by a written contract, agreement, or waiver signed by the surviving spouse

2) (b) A surviving spouse's waiver is not enforceable if the surviving spouse proves that:a. (1) he [or she] did not execute the waiver voluntarily; orb. (2) the waiver was unconscionable when it was executed and, before execution of die

waiver, he [or she]:i. (i) was not provided a fair and reasonable disclosure of the property or financial

obligations of the decedent;ii. (ii) did not voluntarily and expressly waive, in writing, any right to disclosure of the

property or financial obligations of the decedent beyond the disclosure provided; andiii. (iii) did not have, or reasonably could not have had, an adequate knowledge of the

property or financial obligations of the decedent.3) (c) An issue of unconscionability of a waiver is for decision by the court as a matter of law.4) (d) Unless it provides to the contrary, a waiver of "all rights," or equivalent language, in the

property or estate of a present or prospective spouse or a complete property settlement entered into after or in anticipation of separation or divorce is a waiver of all rights of elective share, homestead allowance, exempt property, and family allowance by each spouse in the property of the other and a renunciation by each of all benefits that would otherwise pass to him [or her] from the other by intestate succession or by virtue of any will executed before the waiver or property settlement.

c) Reece v. Elliott - Appellant widow sought rescission of an antenuptial agreement based upon her claim that her late husband failed to make a full disclosure regarding his assets and financial condition. Appellees, co-executrices of the estate, relied on the agreement. The widow's argument was that she did not enter into the agreement with full knowledge of the value of the deceased's assets because there was no value disclosed regarding the stock in a company. The appellate court held that the widow had full knowledge that her husband was a man of wealth, as shown by the list of assets that was provided. The fact that there was no value listed for one particular asset, even though it was significant, did not invalidate the agreement that she entered freely. She consulted with independent counsel and admitted to clearly understanding what the agreement meant and that she would have no claim to any of those assets. She had the opportunity to ask questions about the assets and did not have her counsel investigate, stating that it "did not matter" because she knew the assets would not be hers. She admitted that her husband was never dishonest with her and was very straightforward and open with her about his financial dealings and never misled her. The widow did not avail herself of the opportunity to ask her husband the value of the stock nor make any independent investigation; therefore, the agreement was binding and enforceable on the widow.

1) Prenuptial/antenuptial agreements are favored by public policy in Tennessee, and will be upheld so long as the parties enter into the agreements voluntarily and knowledgeably.

2) "Knowledgeably" is interpreted as meaning that the proponent of the agreement had to prove that a full and fair disclosure of the nature, extent and value of the party's holdings was provided or that such disclosure was unnecessary because the spouse had independent knowledge of the same. The extent of what constitutes "full and fair" disclosure varies from case to case depending upon a number of factors, including the relative sophistication of the parties, the apparent fairness or unfairness of the substantive terms of the agreement, and any other circumstance unique to the litigants and their specific situation. While disclosure need not reveal precisely every asset owned by an individual spouse, at a minimum, full and fair disclosure requires that each contracting party be given a clear idea of the nature, extent, and value of the other party's property and resources. Though not required, a fairly simple and effective method of proving disclosure is to attach a net worth schedule of assets, liabilities, and income to the agreement itself.

3. Rights of Surviving Spouse in Community Property – Spouses own undivided interests in the property they acquire from earnings during marriage.

4. Migrating Couples and Multistate Property Holdings – the general rule us that the ownership of earnings between spouses is governed by the law of the spouse’s domicile at the time the property was acquired.

5. Spouse Omitted from Premarital Will a. In re Estate of Prestie - The decedent and the widow were married in 1987. They were divorced two years later. In

1994, the decedent simultaneously executed in California a pour-over will and an inter vivos trust. The pour-over will devised the decedent's entire estate to the trust. The decedent's son was named as both the trustee and a beneficiary of the trust. Neither the will nor the trust provided for the widow. In 2001, the decedent amended the trust to grant

49 | L e v y

Page 50: Trust and Estates Outline

the widow a life estate in his condominium upon his death. A few weeks later, the decedent and the widow remarried. The decedent passed away approximately nine months later. A probate commissioner found that under Nev. Rev. Stat. § 133.110, the decedent and the widow did not have a marriage contract. Therefore, the probate commissioner recommended that the decedent's will be revoked as to the widow. The trial court upheld the recommendation. On appeal, the court held that the amendment to the trust could not serve to rebut the presumption that the will was revoked as to the widow. The California law referenced in the will and the trust did not apply. Finally, the doctrine of equitable estoppel had no application to the facts of the case.

b. Uniform Probate Code §2-301 – Entitlement of Spouse, premarital willa) If a testator's surviving spouse married the testator after the testator executed his [or her] will, the surviving

spouse is entitled to receive, as an intestate share, no less than the value of the share of the estate he [or she] would have received if the testator had died intestate as to that portion of the testator's estate, if any, that is neither devised to a child of the testator who was born before the testator married the surviving spouse and who is not a child of the surviving spouse nor devised to a descendant of such a child or passes under sections 2-603 or 2-604 to such a child or to a descendant of such a child, unless

1) it appears from the will or other evidence that the will was made in contemplation of the testator's marriage to the surviving spouse;

2) the will expresses the intention that it is to be effective notwithstanding any subsequent marriage; or

3) the testator provided for the spouse by transfer outside the will and the intent that the transfer be in lieu of a testamentary provision is shown by the testator's statements or is reasonably inferred from the amount of the transfer or other evidence.

b) In satisfying the share provided by this section, devises made by the will to the testator's surviving spouse, if any, are applied first, and other devises, other than a devise to a child of the testator who was born before the testator married the surviving spouse and who is not a child of the surviving spouse or a devise or substitute gift under sections 2-603 or 2-604 to a descendant of such a child, abate as provided in section 3-902.

c. Often pour-over wills include a number of dispository provisions (cash to individuals, real estate or specifically identified items of personal property to particular family members), and then give the rest of the testator’s property to a preexisting trust. The effect is to pour probate assets into the trust.

B. RIGHTS OF DESCENDANTS OMITTED FROM THE WILL Common law – parents have no obligation to provide testamentary gifts for their children, even if they are minors. A parent may intentionally disinherit one or more of the parent’s children. Modern Trend – to protect a child from an accidental or inadvertent disinheritance, state legislatures have enacted statutes that may provide a forced share of the parent’s estate for a pretermitted child under circumstances.UPC 2-302 – Under the laws of most states, a child must be born or adopted after the testator executed the will to receive a forced share as a pretermitted child. Few States – An omitted child who was born before the parent executed the will can claim a forced share.1. Protection from Intentional Omission

a. A Look Abroad: Family Maintenance Statutes a) Lambeff v. Farmers Co-operative Executors & Trustees Ltd. - George Lambeff executed a will that did not

include a provision for the plaintiff, his daughter from a previous marriage. The plaintiff seeks a provision from Lambeff’s estate under a statute that distributes a deceased’s estate to certain family members who were are financially supported by their relative in his or her will or through intestacy.

1) Because the evidence showed that the deceased did not provide for his daughter for half of her childhood, the court believed that she was entitled to support, even though she did not necessarily need the support. It is enough that her life would have been better and a father is responsible for helping his child to succeed in life.

2. Protection from Unintentional Omissiona. Gray v. Gray- The son was born after the decedent executed the will in question, which devised all of his estate to his

wife and did not include his two children from a previous marriage. The decedent did not change his will after he and his wife were divorced. The probate court found that the son was entitled to a distribution from the estate under the pretermitted child statute, Ala. Code § 43-8-91, equal in value to the share he would have received had the decedent died intestate. The state supreme court found that one of the exceptions in the pretermitted child statute, Ala. Code § 43-8-91(a)(2), was that an omitted child was not entitled to a share of the estate if, when the will was executed, the testator had one or more children and devised substantially all of his estate to the other parent of the omitted child.

50 | L e v y

Page 51: Trust and Estates Outline

The state supreme court found that in light of the fact that when the decedent executed his will, he had two other children, and his will devised all of his estate to the son's mother, the exception under Ala. Code § 43-8-91(a)(2) applied, and the son could not receive a share of the estate. Thus, the probate court's judgment was incorrect.

b. Uniform Probate Code §2-302 – Example: When T executed her will, she had 2 living children, A and B. Her will devised $7,500 to each child. T then has another child, C. T dies without changing her will, what, if anything, is C entitled to under 2-302?

a) C is entitled to $5,000b) T’s devise to A and B = $15,000c) C is entitled to equal share - $15,000/3d) Each of A and B’s devises will abate (be reduced) ratably by $2500 each. So:

1) A will get $5,0002) B will get $5,0003) C will get $5,000

c. Note: Anna Nicole Smith and Blanket Disinheritance of Children d. Kidwell v. Rhew - The decedent established a trust. The decedent was named trustee and the daughter was a successor

trustee upon the decedent's death. The decedent never executed a will and she died intestate in March 2004. Following her death, the administrator identified three separate assets that were potentially includable in the decedent's estate, including parcels of real estate that had been transferred to the trust. The administrator argued that the pretermitted-heir statute should apply to dispositions made by testamentary will substitutes, such as an inter vivos trust. The reviewing court held that Ark. Code Ann. § 28-39-407(b) (2004) applied only to wills and not to trusts created during the life of the settlor. The pretermitted-heir statute spoke only in terms of wills, and not of trusts, and the administrator cited no convincing authority that would compel the reviewing court to reach the conclusion she urged. The pretermitted-heir statute, which spoke only in terms of the execution of a will, did not apply in instances in which there was no will.

________________________________________________________________________________________________________________CHAPTER 8. TRUSTS: INTRODUCTION AND CREATION

A. INTRODUCTION 1. Background - A trust is a device whereby a trustee manages property as a fiduciary for one or more beneficiaries.2. The parties to a trust – must have these for trust to be valid

a. The settlor (Grantor) - the person who raised the trust - puts assets in the trustb. The Trustee – holds legal title to the property in trust; the person who has the fiduciary obligations; can usually sell

trust property and replace it with more desirable property.a) Individual v. Corporate Trusteesb) Trustee owes fiduciary duties to beneficiaries

1) Duty of Loyalty2) Duty of Prudence 3) Duty of Impartiality4) Duty Not to Commingle5) Duty to Inform and Account

c) Trustee Functions1) Investment2) Administration 3) Distribution

c. The beneficiaries - holds equitable title to the trust; benefits from the trust (entitled to trust income)a) A trust compared with legal life estateb) Rights of Beneficiaries - Can go after trustee for breach of duty; \ Beneficiaries have access, when there's a

breach, to the trust raisedd. Fundamental requirements for a valid express trust:

a) The settlor must intend to split legal and equitable title and impose fiduciary duties on the holder of legal title (trustee) for the benefit of the holder of equitable title (beneficiary).

b) The settlor must have capacity to make a conveyance in trust form.

51 | L e v y

Page 52: Trust and Estates Outline

c) In certain situations, the settlor’s trust intent must be documented by a written instrument.

d) Purpose of the trust must not be illegal or against public policy

e) The settlor must place property into the trust and the trust must continue to hold the property

f) A trustee

g) Beneficiary

e. Fiduciary Duty → central to a trust. So, you can create a trust where you name yourself as trustee, and also as beneficiary. But, you need to also have another beneficiary - b/c there needs to be a fiduciary relationship to be able to hold someone liable for breach of the fiduciary duty.

a) Trustee owes fiduciary duties to beneficiariesf. Inter vivos trust – created during life. Elements:

a) present donative intentb) delivery to the doneec) Acceptance by the donee.

g. Testamentary trust – given after death, usually by will. 3. Three General Rules

a. If the trustee is also the beneficiary, you need another beneficiary due to fiduciary duty requirements.b. A trust will not fail for want of a trustee

a) If settlor doesn’t name a trustee or if the trustee named waives or doesn’t want to do it - Court can always appoint someone as trustee who can fulfill the role

1) Note: personal trustee exception - If settlor gives some inclination that this is the person that he wants to be the trustee, and that person can’t/won’t do it then court would find that the trust fails

c. If grantor/trustee as with a declaration of trust, you do not need formal transfer/delivery of the propertya) This is b/c trustee/grantor same person. So not necessaryb) Note: NY exception - Still need to go through the formal procedure

B. CREATION OF A TRUSTFour Considerations for Trust Validity

Whether there was intent to create a trust by the settlor. Whether there was trust property (trust raised) Whether there are (valid) trust beneficiaries Whether the trust instrument was in writing, if [writing is] required

1. Basic Trust Rules: a. Same party can be the settlor, trustee, and beneficiary at the same time as long as there is another trustee or another

beneficiary. If the same party is both trustee and beneficiary, the legal title and equitable title are said to merge, and the trust is terminated.

b. A trust is not created until it is funded.c. A trust will not fail for want of a trustee.d. Generally all trustees (co-trustees) must consent to any proposed action. The UTC rejects this common law rule and

permits action based on majority vote of the trustees.2. Intent to create a trust

a. UTC §402(a)(2) - A transferor of property has trust intent if the transferor:a) divides title to the property into legal and equitable components, andb) imposes enforceable fiduciary duties on the holder of legal title to deal with the property for the benefit of the

equitable title holder.b. No particular words are necessary to create a trust - look at language to see if there was evidence indicating that

settlor wanted the person to whom property is being transferred to be for the benefit of someone elsea) Factors to consider leaning towards creation of a trust

1) The person wished to create duties2) The express goal would be best implemented by finding a trust3) The person wished to create a legal obligation

52 | L e v y

Page 53: Trust and Estates Outline

1. Precatory Language → nonbinding/nonenforceable language – “I hope, “I recommend”2. Equitable Charge - If a testator devises property to a person subject to the payment of a

certain sum of money to a third person, the testator creates an equitable charge.a. Equitable Charge creates a security interest in the transferred propertyb. No fiduciary duties - the relationship is more in the nature of a debtor and

secured creditorb) The language “shall be maintained” and “shall not be sold” is strong indication of intent to create a trust

1) Lux v. Lux – The will said "shall be maintained for the benefit of said grandchildren and shall not be sold until the youngest of said grandchildren reaches 21 yrs”. Issue is it outright gift or a trust – court says the language indicates intent to create a trust.

c) The words “in trust” not necessary to create a trust so long as the transfer was made with the intent to vest ownership in a third party.

1) Jimenez v. Lee – The daughter's paternal grandmother and a client of the father both made separate gifts for the daughter's benefit. The father invested the proceeds of both gifts in bank stock. He took title thereto as custodian for his children. The trial court found that the father did not hold the funds in trust for the benefit of the daughter, but as custodian. On appeal, the court held that the intent of the donors was enough to vest beneficial ownership in the daughter, thereby creating a trust. The father, an attorney, also demonstrated that he knew the savings bond was held in trust. The court found that the father breached his duty to the daughter to administer the trust solely in the interest of her as beneficiary where he never provided any accounting and used some of the money for purposes other than the daughter's educational needs

1. Father was given money from grandma as a gift to his children; dad took money and invested in stock. Gifts never given using the words “to be held in trust.” Court says a trust was created & dad had a fiduciary duty as trustee. So although dad says he used money to pay for various things for daughter, court says as a trustee, your fiduciary duty includes the duty to do the accounting. Daughter not entitled to entire proceeds from stock dad used with trust money – need to take out what was used for her education.

d) Precatory trust – arises where there is an outright gift from a donor to done, but the donor includes some language that expresses the hope, the wish BUT no legal obligation that the property be used for benefit of another. A precatory trust is not a trust.

c. Trust vs. Gifta) To make an outright gift of personal property, as compared to a gift in trust, the donor must deliver the

property to the donee, and the donee must accept the gift. So if you die before you give a gift, it does not have to be given, but if it were made in trust, then it must be devised.

1) Delivery need not be physical1. Constructive delivery – something that gives you access to the gift (keys)2. Symbolic delivery – symbolic of the gift (a picture of the gift)

2) The mere intent to give property to someone else is not enough to create a trust when the property has not changed hands; there must be a clear intent to impose the duties of a trustee on the donor.

b) Failed gifts - The Hebrew University Association v. Nye(1961)- The decedent inherited a large library from her late husband, a professor. During her lifetime, she told the university that she was making a gift of the library to it and even went so far as to attend a luncheon in her honor and make a formal announcement of her gift. However, she never actually tendered possession of the library to the university, although she did prepare it for shipping. Upon her death, the decedent bequeathed the bulk of her estate to a charitable trust or foundation. The university was not mentioned in the decedent's will. On appeal, the court held that the trial court's judgment declaring that the university was the owner of the library was without support in the findings and could not stand. The court held that, at most, the decedent had the requisite donative intent such that, had she subsequently made a delivery of the property while that intent persisted, there would have been a valid, legal gift inter vivos.

1) Court will not save a gift that fails of delivery by imposing a trust where no facts indicate a trust was intended.

2) For the oral declaration to have created a trust, there must have been obvious intent that widow was imposing on herself the enforceable duties of a trustee. However, court said this was a valid inter vivos gift, because symbolic delivery was made when she announced she was giving it to them, as actual delivery would have been impractical.

53 | L e v y

Page 54: Trust and Estates Outline

c) The Hebrew University Association v. Nye (1966)- A university brought an action to determine whether a decedent had made a gift of her late husband's library, a large collection of books. The court held that the delivery of a memorandum, coupled with the decedent's acts and declarations clearly showing an intention to give and to divest herself of any ownership of the library, were sufficient to complete the gift. The circumstances under which the gift was made, a public announcement at a luncheon attended by a head of state, accompanied by a document that identified in itemized form what was being given, were a sufficient substitute for a formal instrument purporting to pass title. It was undisputed that the decedent intended to give the books to the university so that they might be easily accessible to scholars. There was ample reason for equity to impose a constructive trust over the library, as denying the university's claim would have frustrated the intent of the decedent.

d. Trust vs. Other Legal Relationshipa) Trust vs. Equitable Charge: “I leave my real estate located in St. Louis, MO to brother, John Smith, provided

he pays $20,000 to Bob Cosas.b) Trust vs. Debt: Lakisha transferred $50,000 to Detmar in return for Detmar’s promise to repay the $50,000 in

10 years with interest payable each year at 5 percent.2. Necessity of trust property - To create a trust there must a formal declaration of trust and a separate allocation of trust

property.a. To create a trust there must a formal declaration of trust and a separate allocation of trust property.

a) Unthank v. Rippstein - Shortly before his death, decedent wrote a letter to respondent donee, in which he acknowledged that he wold send her monthly payments for a period of five years, even in the event of his death. Respondent first attempted to probate the writing as a codicil of decedent's will, and then filed an action for payments already matured, as well as having sought a declaratory judgment in the remaining payment. The trial court granted summary judgment in favor of petitioner estate executors, and the appellate court reversed, finding the establishment of a voluntary trust, with legal title in decedent's heirs. On appeal, the court determined that the writing was insufficient to establish a voluntary trust, because a voluntary trust would have divested decedent of the exercise of further dominion. To so construe the writing would have been in contradiction with the language in the writing itself, and therefore no reasonable implication could give rise to a voluntary trust. The court held that the writing was merely a promise to make a series of gifts in the future, and was therefore unenforceable, and the court affirmed the judgment of the trial court.

b) What happens to trust res if the trust fails for some reason?1) To settlor - reversionary interest - an outright transfer2) Resulting trust - some courts will impose this for successor; trustees appointed (instead of

reversion) - goes back to grantor's heirs in trust3) Purchase money trust

b. What can be trust property: a) To have a trust, there must be trust property - there must be something for a trustee to protect. In many

cases, there is really no question about trust propertyb) In some situations the items claimed to be held in trust can be very insubstantial, like future profits or the

beneficial interest in a life insurance policy. Then the question becomes whether the item can be defined as “property” for the purpose of sustaining a trust.

c) Future profits – can be used as trust property if there is a legal right to receive the future profits and a real expectation to receive them.

1) Brainard v. Commissioner - In 1927, the taxpayer declared a trust of his stock trading during 1928 for the benefit of his family upon certain terms and conditions. During 1928, the taxpayer carried on the trading operation and determined his compensation at $ 10,000, which he reported on his income tax return for that year. The profits remaining were then divided in equal shares among his family members, the amounts were reported in their respective tax returns. The question presented on review was whether the taxpayer created a valid trust, the income of which was taxable to the beneficiaries under § 162 of the Revenue Act of 1928. The court affirmed, concluding that the taxpayer's profits were not impressed with a trust when they first came into existence because the taxpayer only made a gratuitous declaration to his family. The board found that the trust first attached when the taxpayer credited them to the beneficiaries on his books. This act, the court found constituted the taxpayer's first expression of intention to become a trustee of the fund. Prior to that time, the declaration could not have been enforced because there was no consideration.

54 | L e v y

Page 55: Trust and Estates Outline

2) Speelman v. Pascal – Defendant appealed the grant of summary judgment for plaintiff, contending that the delivery of an agreement did not constitute a valid, complete, and present gift to plaintiff by way of assignment of a share in future royalties when and if it was collected from the exhibition of the musical stage version and film version of a play. The trial court found that plaintiff was entitled to receive the percentages set out in the agreement, required defendant to render plaintiff accountings from time to time of all moneys received from the musical play and the film version, and ordered defendant to make the payments required by the agreement. The decision was affirmed on appeal, because the assignment to plaintiff of a future interest which was not present at the time of the assignment could create an equitable title enforceable by the court.

c. Pour Over Will: Exception to requirement that there must be trust raised - funding trust from assets distributed by the will. Although trust not raised inter vivos, it will be once settlor dies, by will distribution.

3. Necessity of a Trustee – the trustee holds legal title to trust property and is bound by a plethora of fiduciary duties to deal with that property for the benefit of the beneficiaries.

a. If a trust is intended, but no trustee is named, or if a sitting trustee dies or resigns, a court will appoint a trustee rather than let the trust fail.

4. Necessity of Trust beneficiariesa. Trust beneficiaries hold equitable title to trust property and have standing to enforce a trust. A trust cannot exist

without a beneficiary. b. Beneficiary must be ascertainable – doesn’t have to be named, but you have to be able to figure it out.

a) Clark v. Campbell - Decedent passed and under the terms of her will bequeathed certain property to her "friends." Her "friends" as purported heirs, commenced suit against the state and sought to recover under the will. The court discharged the complaint. The court held that: (1) the bequest was invalid because it was a bequest to an indefinite person; (2) the term "friends" was too indefinite because it had no accepted statutory or controlling limitation and was not precise at all; and (3) the bequest was not a power to substitute the will of the testator for the will of the trustees to the undefined class of "friends" was irresponsible.

b) Restatement 2nd of Property § 12.1 comment e, seeks to reform the rule - says that rather than failing, a provision like that in Clark “should be construed to give the [trustees] a power of appointment exercisable within a reasonable period of time . . . .”

c. Pet Beneficiaries – An animal cannot be a beneficiary because it cannot hold the trustee accountablea) Exception – Honorary trusts

1) In re Searight’s Estate - The decedent bequeathed the sum of $ 1000 for the care of his dog, which was to be paid to the dog's caretaker at the rate of 75 cent per day. In determining the inheritance tax due from the estate of the decedent, trial court found that levying a tax on successions to property did not levy a tax upon the succession to any property passing to an animal; that the $ 1000.00 bequest to the dog was therefore not taxable; and that the remainder of the $ 1000.00, if any, after the death of the dog was taxable in the hands of the remaindermen. The department argued, inter alia, that the bequest to the extent it was paid to the caretaker for the care of the dog, was not a succession to property passing in trust or otherwise, to or for the use of a person; and in not holding that the bequest of $ 1000 to the extent it was to be paid to the caretaker was a bequest or succession to the caretaker, subject to Ohio inheritance taxes. The court held that a tax based on the amount expended for the care of the dog could not lawfully be levied against the monies so expended, since it was not property passing for the use of a "person, institution or corporation."

d. Trust for Pets and noncharitable purposes Honorary Trust Statutory Purpose Trust Transferee is not obligated to carry out settlor’s

purpose If transferee declines, she holds the property on

resulting trust and property reverts to settlor or settlor’s successors

Used in Searight’s Estate

Statutory Trust for pet animal or other noncharitable purpose

Authorized by UTC §§408-409 and UPC §2-907 Typically authorize court to reduce excessive trust

property and provide for enforcement by settlor or court appointee

d. Honorary Trusts - Recognizing that funds to care for a pet or an inanimate object—like a grave site—serve a social purpose, courts usually allow them under a theory of “honorary trusts.” The “trustee” who is given the money has the choice of honoring the trust or of returning the money to the estate. Uniform Trust Code sections 408(b) & 409 allow a settlor (or a court) to appoint someone to enforce the trust.

55 | L e v y

Page 56: Trust and Estates Outline

a) Majority jurisdiction - Honorary trusts would fail and the property would revert to the settlor or if the settlor has already died, to the settlor’s successors in interest under the resulting trust principles.

b) HOWEVER - Restatement 2 nd of Trusts provides in §124 that honorary trusts are enforceable BUT the “trustee: may use the property for the designated purpose.http://imagehost.vendio.com/bin/imageserver.x/00000000/bellacosajewelers/IMG_6860-1.jpg

1) UPC §2-907 sets up special provisions for both pet and non-pet honorary trusts and provides that the intended use of the principal or income can be enforced by an individual designated for that purpose in the trust instrument or if none, by an individual appointed by a court upon application to it by an individual.

a. UTC §§408 & 409 continues this approach 4. Necessity of a written instrument

a. In general, a trust does not need to be in writing requirement unless it is 1) a testamentary trust (will requirement) or, 2) are a trust involving real property (statute of frauds).

b. CL and Majority – a trust of personal property is enforceable even in absence of a writing.c. Modern trend (adopted by Restatement 3rd) finds that the common law outcome constitutes unjust enrichment of the

intended trustee. Restatement corrects this by imposing a constructive trust and by ordering the purported trustee to distribute the real property to the intended beneficiaries.

d. UTC §407 eliminates the writing for all trusts but requires oral trusts to be established by clear and convincing evidence.

e. Oral Trust for Disposition at deatha) In re Estate of Fournier - The settlor, the late brother of the prospective beneficiary, gave money to a couple

and told them that the money was to be held by them and given to the prospective beneficiary upon his death. After he died, the couple gave the money to her. Another sister of the settlor claimed that the settlor meant that the prospective beneficiary was to take the money as the estate's personal representative. The probate court agreed and entered judgment denying the prospective beneficiary's declaratory judgment action. On appeal, the state Supreme Court found that an oral trust had been created for her benefit and, thus, the money passed to her as a beneficiary.

1) A trust may be created by transfer of property to another person as trustee during the settlor's lifetime.

2) Although a trust need not be in writing, the creation of an oral trust must be established by clear and convincing evidence.

b) Resulting Trusts - Unjust enrichment can also arise when there is a semi-secret trust. These occur when will shows a trust intention on its face, but the details are oral. To prevent the “trustee” from becoming unjustly enriched by keeping the property, a court may impose a “resulting trust” under which the trustee holds the property for the benefit of the estate.

1) Oiliffe v. Wells - The residuary bequest to the executor gave him no beneficial interest. It required him to distribute the property bequeathed to him, but allowed him a discretionary authority with respect to distribution pursuant to the testatrix's intentions. The will declared a trust too indefinite to have been carried out, and plaintiffs were required to take by way of resulting trust, unless the facts agreed showed such an executable trust for the benefit of others. The testatrix had orally made known to the executor her intention that the residue be disposed of and distributed by him for charitable uses at his discretion. Intentions not formed by the testatrix and communicated to the executor before the making of the will could not have had any effect against plaintiffs. Plaintiffs could not have been deprived of their equitable interest that accrued to them directly from the testatrix by any conduct of the devisee executor, nor by the testatrix's intention, unless legally signified in writing. A trust not sufficiently declared on the face of the will could not have been set up by extrinsic evidence to defeat the rights of plaintiffs. A decree for plaintiffs was proper.

2) Types of trusts court identifies here:1. Secret trust – beneficiary appears to have rec’d property for his own use, but decedent

& beneficiary have previously orally agreed that it be held in trust for some secret purpose. Court will allow extrinsic evidence that sheds like on what was meant.

2. Semi-secret trust –will appears to create a trust with the beneficiary under will as trustee but purpose of trust not identified. Court will not allow extrinsic evidence.

56 | L e v y

Page 57: Trust and Estates Outline

c) Modern trend / Restatement 3rd - takes the position that a constructive trust in favor of the beneficiaries should be imposed in both the secret and semi-secret trust situation.

1) Constructive trusts - Not allowing an oral trust may enrich a grantee unjustly. To avoid unfair results, courts sometimes will apply a “constructive trust”. The constructive trust theory is a device for preventing unjust enrichment by moving legal title from a person who has title but should not, to someone who should.

b. Oral Inter Vivos Trust of Land - Some jurisdictions will use a constructive trust to validate an oral trust of land if the trustee is in a confidential relationship with the transferor.

a) For benefit of third party: O conveys land to X upon an oral trust to pay income to A for life, remainder to B.1) Majority: X can keep land unless X procured land by fraud, duress, abuse of confidential

relationship – then constructive trust 2) Minority: Constructive trust imposed on X in favor of A and B

b) For benefit of transferor: O conveys land to A. A orally agrees to reconvey land to O at a later time1) Pappas: transferee (A) is allowed to keep land if transferor does not have “clean hands”2) Hieble: constructive trust in favor of the transferor to prevent transferees unjust enrichment.

________________________________________________________________________________________________________________CHAPTER 9. RIGHTS TO DISTRIBUTIONS FROM TRUST FUND

A. RIGHTS OF THE BENEFICIARIES TO DISTRIBUTIONS (597-609)1. Mandatory Trust: Requires trustee to make pre-designated distributions. The trustee has no discretion to choose either the

persons who will receive the income or the amount to be distributed. a. Beneficiaries have right to alienate – can sell it, satisfy debtsb. Therefore, creditors may also reach the trust asset

2. Discretionary Trust: Trustee has discretion over payment of either the income or the principle or both. Beneficiary cannot get until trustee gives it (so creditors can’t get either, until trustee gives to beneficiary). Trustee determines to whom, and how much he pays and does NOT have to pay all of it

a. Courts require trustees to act in good faith, honesty, and for the purposes the settlor stated in the trust. UTC §814 b. Extent of discretion – Most courts require a trustee’s discretion to be exercised reasonably; there must be extremely

strong evidence of unreasonableness to overrule trustee’s honest exercise of discretion. c. Duties of Trustee :

a) Duty to inquire into financial status of beneficiary - If beneficiary is ambiguous about financial report, trustee must clear it up.

b) Duty to act reasonably (unless it is sole and absolute discretion, explicitly stated).c) Fiduciary Duty

1) Marsman v. Nasca – The trustee had the power under a testamentary trust to pay principal to a beneficiary for his support and maintenance. Aside from one payment, the trustee never made any payments to the beneficiary and, as a result, the beneficiary had to convey his house. The probate court agreed that the trustee should have made distributions to the beneficiary so that he could have kept the house. The court agreed with the probate court's finding but disagreed as to the remedy. The court found that the trustee failed in his duty of inquiry into the needs of the beneficiary. The court determined, however, that the proper remedy was not to set aside the conveyance but to determine the amounts which should have been paid to the beneficiary to enable him to keep the house, and to pay that amount from the trust to the beneficiary's estate. The court also found that the exculpatory clause which had been drafted by the trustee was effective and thus the trustee would not be personally charged.

a. Discretionary/support trust - Beneficiary goes to trustee and says that he is running low on funds and trustee said to put it in writing and he will consider it. Beneficiary never did and lived in a state of poverty. Trustee never inquired into his financial situation, never gives more money. Court says when trustee failed to check up on beneficiary and denied more payouts, this was a breach of the fiduciary duty.

d. Purely Discretionary v. Discretionary with a support standardTraditional /Restatement. 2nd Restatement 3rd § 60 UTC § 504

Discretionary Trust - trustee has discretion over income or interest.

Creditors cannot reach Creditor stands in shoes of bene & can compel distribution if Trustee abuses

Creditors cannot reach – unless: trustee abuses discretion or fails to comply

57 | L e v y

Page 58: Trust and Estates Outline

Creditor of the beneficiary cannot reach the trust assets, since beneficiary doesn’t have a recognizable interest. Trustee doesn’t have to make a payment.

discretion or fails to comply with distribution standard

with dist. stnd. & spouse, former spouse, or child of ben

Support Trust - requires trustee to make payments of income to the beneficiary in an amount necessary for education or support of beneficiary

Creditors cannot reach unless supplier of necessaries or child, spouse or former spouse

Creditor stands in shoes of bene & can compel distribution if Trustee abuses discretion or fails to comply with distribution standard

Creditors cannot reach – unless: trustee abuses discretion or fails to comply with dist. stnd. & spouse, former spouse, or child of beneficiary

3. Extended Discretion, Exculpatory Clauses, Mandatory Arbitration ClausesExtended Discretion Exculpatory Clause Mandatory Arbitration

Trustee discretion is “sole,” “absolute,” or “uncontrolled.”

In spite of extended discretion, trustee is still subject to judicial review.

Trustee must act in good faith & ?????

Trustee is excused from liability for breach of trust.

Enforceability against Trustee/draftsman???

Cannot excuse liability for bad faith, reckless indifference, or intentional or willful neglect.

Claims for breach of trust must be resolved by arbitration.

Whether clause is enforceable, precluding judicial review, is unresolved.

Authority is scarce and contradictory

B. RIGHTS OF THE BENEFICIARIES’ CREDITORS 1. Discretionary Trusts

a. Uniform Trust Code §504b. Duties of trustee in discretionary trusts: duty of inquiry, duty to act reasonably and in good faith in exercising

discretion, unless modified by the settlor in express terms of the trust, and the duty to 2. Spendthrift Trusts

a. UTC §502 A spendthrift clause is a provision of a trust that does 2 things:a) Prohibits the beneficiary from selling, giving away, or otherwise transferring the beneficiary’s interest.b) Prevents the beneficiary’s creditors from reaching the beneficiary’s assignees or creditors.

b. Spendthrift restrictions – Settlor as Beneficiarya) Scheffel v. Kruger – The tortfeasor was the beneficiary of a trust containing a spendthrift provision. The trial

court held that the spendthrift provision barred the mother's claim against the trust. Nothing in the language of the NH statute suggested that the legislature intended that a tort creditor should be exempted from a spendthrift provision. New Hampshire did not recognize a public policy tort creditor exception to the statute. Further, the statute did not place any limitation on the rights a beneficiary was granted under the trust instrument. Rather, by its plain language the statute applied where a trust's governing instrument provided a beneficiary was not able to transfer his or her right to future payments of income and principal, and a creditor of a beneficiary was not be able to subject the beneficiary's interest to the payment of its claim.

1) There are only two exceptions: One is if the beneficiary is the settlor and the trust is not a special need trust established for a person with disabilities and second, for fraud. None of these exist. Even though they had a judgment they could not collect on the trust. Settlor can put a restraint of alienation

c. Spendthrift restrictions – Spousal and Child Supporta) Shelley v. Shelley - The ex-husband had disappeared and his location was not known at the time of the suit.

The trust placed no conditions upon the right of the ex-husband to receive the trust income during his lifetime. The court held that public policy required that the interest of the beneficiary of a trust should be subject to the claims for support of his children. The court also held that the duty of the husband to support his former wife overrode the restriction called for by the spendthrift provision. The court held that the beneficiary's interest in the income of the trust was subject to the claims of the second wife for alimony and to the claims for the support of the ex-husband's children as provided for under both decrees for divorce. The court also held that the ex-husband's interest in the corpus was not made immune from the claims. However, by the terms of the trust, the disbursement of the corpus was within the discretion of the trustee, and therefore, the ex-husband's right to receive any part of the corpus did not arise until the trustee had exercised his discretion and had decided to invade the corpus.

58 | L e v y

Page 59: Trust and Estates Outline

d. Restatement. 3 rd § 59 - Spendthrift Trusts: Exceptions For Particular Types Of Claims – a) The interest of a beneficiary in a valid spendthrift trust can be reached in satisfaction of an enforceable claim

against the beneficiary for 1) support of a child, spouse, or former spouse; or2) services or supplies provided for necessities or for the protection of the beneficiary's interest in the

truste. Uniform Trust Code §§502, 503, 505

a) §502 - Creditor Claims; Spendthrift; and Discretionary Trusts1) (a) A spendthrift provision is valid only if it restrains both voluntary and involuntary transfer of a

beneficiary's interest.2) (b) A term of a trust providing that the interest of a beneficiary is held subject to a "spendthrift trust,"

or words of similar import, is sufficient to restrain both voluntary and involuntary transfer of the beneficiary's interest.

3) (c) A beneficiary may not transfer an interest in a trust in violation of a valid spendthrift provision and, except as otherwise provided in this [article], a creditor or assignee of the beneficiary may not reach the interest or a distribution by the trustee before its receipt by the beneficiary.

b) A settlor has the power to restrain the transfer of a beneficiary's interest, regardless of whether the beneficiary has an interest in income, in principal, or in both. Unless one of the exceptions under this article applies, a creditor of the beneficiary is prohibited from attaching a protected interest and may only attempt to collect directly from the beneficiary after payment is made.

c) §503 - Exceptions To Spendthrift Provision 1) (a) In this section, "child" includes any person for whom an order or judgment for child support has

been entered in this or another State.2) (b) Even if a trust contains a spendthrift provision, a beneficiary's child, spouse, or former spouse who

has a judgment or court order against the beneficiary for support or maintenance, or a judgment creditor who has provided services for the protection of a beneficiary's interest in the trust, may obtain from a court an order attaching present or future distributions to or for the benefit of the beneficiary.

3) (c) A spendthrift provision is unenforceable against a claim of this State or the United States to the extent a statute of this State or federal law so provides.

d) §505 - Creditor's Claim Against Settlor: (a) Whether or not the terms of a trust contain a spendthrift provision, the following rules apply:

1) (1) During the lifetime of the settlor, the property of a revocable trust is subject to the claims of the settlor's creditors. If a trust has more than one settlor, the amount the creditor or assignee of a particular settlor may reach may not exceed the settlor's interest in the portion of the trust attributable to that settlor's contribution.

2) (2) With respect to an irrevocable trust, a creditor or assignee of the settlor may reach the maximum amount that can be distributed to or for the settlor's benefit. If a trust has more than one settlor, the amount the creditor or assignee of a particular settlor may reach may not exceed the settlor's interest in the portion of the trust attributable to that settlor's contribution.

3) (3) After the death of a settlor, and subject to the settlor's right to direct the source from which liabilities will be paid, and, except as otherwise provided by other applicable statutes, the property of a trust that was revocable at the settlor's death is subject to claims of the settlor's creditors, costs of administration of the settlor's estate, the expenses of the settlor's funeral and disposal of remains, and allowances to a surviving spouse and children to the extent the settlor's probate estate is inadequate to satisfy those claims, costs, expenses and allowances.

f. Note: Bankruptcy Law and Trust Asset Protection3. Support Trust

a. A support trust directs the trustee to pay only so much of the income or principal as is necessary for the beneficiary’s support, food clothing, medical care, or education. A support trust may either be mandatory or discretionary in nature.

b. General View is that beneficiaries cannot alienate support trusts so creditors cannot reach beneficiaries’ interest.a) Exception (who can reach beneficiaries interest):

1) Suppliers of necessitiesa. Always in the UTC and Restatement b. Does not apply for purely discretionary trusts for the UTC

59 | L e v y

Page 60: Trust and Estates Outline

2) Children and spouse my enforce claim for child support and alimony against bfs interest in a support trust.

a. Under UTC, court can direct trustee to pay amount. b. Does not apply for purely discretionary trusts for the UTC

C. MODIFICATION AND TERMINATION 1. Introduction – A trust ends when all of the trust res is completely disbursed.

a. If all 3 groups (settlor, trustee, and the beneficiaries) all agree to modify or terminate the trust, the trust can be modified or terminated.

b. If settlor and beneficiaries consent, even is the trustee objects, the trust can be modified or terminated. c. Assuming the settlor has no interest in the trust (irrevocable trust), if all beneficiaries and the trustee consents, the

trust can be modified or terminated. 2. Deviation And Changed Circumstances

a. In Re Trust Stuchell – One of two surviving life-income beneficiaries of a testamentary trust sought modification of the trust. The trust will terminate on the death of the last income beneficiary, to be distributed equally to the beneficiary's children or their descendants. One of the beneficiary's four children is a mentally retarded 25 year old who is unable to live independently without assistance. He receives Medicaid and Social Security benefits, both of which have income and resource limitations for participants. The life-income beneficiary requested the court to approve, on behalf of the mentally retarded child, an agreement, the stated purpose of which is to ensure the child's continued qualification for public assistance. The court affirmed the trial court's dismissal of the petition for modification, and held that there was no basis on which the trial court could have granted the modification, solely to make the trust more advantageous to the income beneficiaries.

b. Modification – At CL, even if the trustee objects, if an unforeseen change of circumstances defeats or substantially frustrates settlor’s intent, and all the beneficiaries consent, the court will order modification of the trust. The assumption is that the modification is to further the settlor’s intent.

a) Restatement 3rd Of Trusts §66 -Modification Due to Unanticipated Circumstances/Equitable Deviation - The court may modify an administrative or distributive provision of a trust … if because of circumstances not anticipated by the Settlor the modification or deviation will further the purposes of the trust.

b) CL Approach - The Claflin Doctrine – Cannot terminate/modify terms of the trust prior to natural expiration even though all beneficiaries agree, if it is contrary to settlor’s intent. Can only modify for administrative purposes not related to dispositional aspect. (Stutchell mentally handicapped)

1) Exception (NY) - Poor surviving spouse & Trust protectors b) Modern trend – shift towards giving the beneficiaries greater control over the property in trust after the

settlor’s death.c) UTC § 412 - Court can modify both administrative & dispositional terms or terminate, if changed

circumstances not anticipated by settlor, & to further the purpose of the trust (settlor’s probable intention). Modify admin terms if impracticable or wasteful or impair trust (settlor’s intent irrelevant for admin).

d) UTC § 415 – Reformation to correct mistakese) UTC § 416 – Modification to achieve Settlor’s tax objectives

c. Administrative modification: In Re Riddell – The trustee's parents established trusts for the benefit of the trustee, his wife, and the settlors' grandchildren. The trusts provided that the grandchildren would receive the benefits until the age of 35 when the trusts would terminate and the trustee would distribute the principal to the grandchildren. The trustee's daughter suffered from schizophrenia affective disorder and bipolar disorder; she was not expected to live independently for the remainder of her life. The trustee sought to create a “special needs” trust on his daughter's behalf, instead of distributing the trust principal to her. remand was necessary because (1) there was no question that changed circumstances intervened to frustrate the settlors' intent, and (2) the trial court should not have considered any loss to the State in determining whether an equitable deviation was allowed since 42 U.S.C.S. § 1396p(d)(4)(A) invited, rather than discouraged, the creation of special needs trusts in such situations.

a) The Court has held that trial courts may use “equitable deviation” to make changes in the manner in which a trust is carried out. The Washington Supreme Court has outlined the two prong approach of “equitable deviation” used to determine if modification is appropriate. The court may modify an administrative or distributive provision of a trust, or direct or permit the trustee to deviate from an administrative or distributive provision, if

1) because of circumstances not anticipated by the settlor 2) the modification or deviation will further the purposes of the trust. The Washington Supreme Court

has adopted the Restatement (Third) of Trusts and has noted that the Restatement (Third) requires a

60 | L e v y

Page 61: Trust and Estates Outline

lower threshold finding than the older Restatement and gives courts broader discretion in permitting deviation of a trust

d. Trust Protectors - Relatively new development. A trust protector can be given powers and control over a trust similar to those held by the settlor. Example of protector powers:

a) Replace trustee;b) approve modifications to trust provisions;c) terminate trust; ord) select a successor trust protector

3. Trust terminationa. Claflin doctrine – consistent with the traditional American approach of being more protective of the settlor’s intent.

The trustee can block premature termination of the trust, even if all the beneficiaries consent, if the trust has an unfulfilled purpose.

b. Unfulfilled purpose – Test is very fact-sensitive turning on the language and apparent purpose of each trust. c. In re Estate of Brown - The trustee complied with the terms of the trust and used the trust proceeds to pay for the

education of the beneficiaries. Once the trustee determined that the beneficiaries' education was complete, he began to distribute the trust income to the beneficiaries in their capacity as lifetime beneficiaries. The beneficiaries petitioned the probate court for termination of the trust, arguing that the sole remaining purpose of the trust was to maintain their lifestyle and that distribution of the remaining assets was necessary to accomplish that purpose. The remainder men filed consents to the proposed termination, but the probate court denied the petition. However, on appeal, the superior court granted the petition. On appeal, the court concluded that the superior court's conclusion that the trust was terminable because the material purpose of the trust had been accomplished had an insufficient basis in the findings. The court held that the settlor's intention to assure a life-long income for the beneficiaries would have been defeated if termination of the trust were allowed so the judgment of the superior court was reversed.

d. Restatement 3 rd of trusts §65 – weakens material purpose, authorizes termination if reason outweighs material purpose, preserves requirement of beneficiaries’ unanimity.

e. Uniform Trust Code §411 – preserves material purpose, weakens requirement of beneficiary unanimity, authorizes termination if interests of absent beneficiary will be adequately protected.

f. Note: Revocable v. Irrevocable Trustsa) Irrevocable Trust (usually testamentary) - If settlor & all beneficiaries consent → may be modified or

terminated. Trustee cannot object b/c he has no beneficial interest. If settlor is dead → trust cannot be terminated/modified before time fixed for termination, even if all BFs agree, if termination/modification is contrary to material purpose of settlor (Claflin Doctrine)

b) Revocable Trust – can be modified/terminated if settlor consents.g. Termination - Generally, a trust cannot be terminated if → spendthrift trust, if delays receipt of principle until certain

age, if a discretionary trust, or support trust. This is b/c these types of provisions are considered (presumptively) the material purpose of settlor. Trust may not be terminated early even if all BFs agree, unless all material purposes of trust are achieved. (Brown)

4. Summary of Modification/termination rules:a. Modification/Termination by Consent

a) S and all B consent – can mod. or term. (Rstmt. 2/3rd,UTC§ 411(a))b) All B consent:

1) & not inconsistent material purpose of S – Claflin/Rstmt 2nd & UTC §411(b)2) & reason for mod./term. outweigh S material purpose – Rstmt. 3rd § 65(s)

c) Some B consent1) & interests of benes who don’t consent are adequately protected & not inconsistent material

purpose of S – UTC § 411(e)b. Unanticipated Circs./Equitable Deviation

a) Traditional – Restatement 2nd:1) Admin: if unantic. circs. & necessary to achieve purpose of trust

b) Restatement 3rd § 66/UTC § 412(a)1) Admin & Dispositive: if unantic. circs. & will further purpose of trust.

a. Settlor probable intent if the circs. in question had been anticipated c) UTC § 412(b) - Admin: if impracticable or wasteful or impair trust’s admin(UTC § 412(b))

5. Trustee Removal - Removal of trustee if a remedy for breach of trust, not modification of trust terms.

61 | L e v y

Page 62: Trust and Estates Outline

a. Traditional Approach: Courts are authorized to remove a trustee who is dishonest or if serious breach of trust. Cannot remove if not a serious breach or if a simple disagreement with BF. If settlor chose trustee – even harder to remove, and never if basis for removal was aware to settlor when he named the trustee, unless trustee entirely unfit to serve.

b. UTC Approach: can remove if (1) material breach of trust, (2) lack of cooperation among co-trustees substantially impairs administration of trust, (3) trustee is unfit, unwilling, or persistently fails to administer trust effectively & court determines removal in best interest of beneficiaries, or (4) substantial change in circumstances or all beneficiaries request removal, & court finds in best interest & not contrary to material purpose of trust, & other trustee available.

a) Uniform Trust Code § 706c. Davis v. U.S. Bank National Association - The bank argued that the grandson's failure to join all of the remainder

beneficiaries of the trust as parties to his lawsuit deprived the court of subject matter jurisdiction. The appellate court found that, based on the language of Mo. Rev. Stat., the grandson and his son and daughter were the permissible distributees if the bank's interest or the trust terminated at the time of the filing of the suit. As such, the grandson and his children were all of the qualified beneficiaries of the trust. The remote remainder beneficiaries of the trust were not qualified beneficiaries. All of the necessary parties were before the court. The grandson and children had substantially identical interests which were not in conflict with regard to removing the bank as trustee and implementing a successor. There was no conflict of interest or issue of fact with regard to payment of capital gains taxes. The grandson presented factually supported reasons why it would be beneficial to him and the children to remove the present trustee and the bank did not put any of those reasons into dispute. Removal of trustee affirmed.

a) UTC requires only the qualified beneficiaries to consent. A qualified beneficiary is one who would be entitled to distribution if the trust were terminated on the date petition is filed.

d. Note: Virtual Representation

________________________________________________________________________________________________________________CHAPTER 10. TRUST ADMINISTRATION: THE FIDUCIARY OBLIGATION

A. INTRODUCTION 1. Note: Agency Costs and the Fiduciary Obligation

a. Rise of Management Trust a) Maximum Empowermentb) UTC § 815 - General Powers Of Trustee

1) (a) A trustee, without authorization by the court, may exercise:1. (1) powers conferred by the terms of the trust; or2. (2) except as limited by the terms of the trust:

(A) all powers over the trust property which an unmarried competent owner has over individually owned property;

(B) any other powers appropriate to achieve the proper investment, management, and distribution of the trust property; and

(C) any other powers conferred by this [Code].2) (b) The exercise of a power is subject to the fiduciary duties prescribed by this [article]. Subject to

Fiduciary Obligation – 1. Loyalty2. Prudence3. Subsidiary Rules

2. Powers of the Trusteea. Common Law – the office of trustee has no inherent powers, only those that were expressly granted to trustee by the

deed or declaration of trust, or those implicitly provided in light of the express trust powers and or/purpose. The risk of trustee management is regulated primarily by limiting trustee’s powers over the property.

b. Modern trend - Trustee has been granted automatically all powers a prudent person would need to manage the trust in light of its purpose. The risk of trustee management is regulated primarily by imposing a duty of loyalty and prudent administration on the trustee.

c. Judicial authorization – Trustee can petition a court of equity for authorization to undertake an action not expressly or implicitly authorized under the terms of the trust.

B. DUTY OF LOYALTY

62 | L e v y

Page 63: Trust and Estates Outline

1. Duty of loyalty – the trustee owes a duty of absolute loyalty to the beneficiaries. Everything the trustee does must be done in the best interest of the beneficiaries.

a. Test: Courts have translated the duty of loyalty to act reasonably (objective standard) and in good faith (subjective standard).

2. Duty against self-dealing – self-dealing arises where the trust and the trustee engage in a transaction. Where a trustee engages in self-dealing, an irrebuttable presumption of breach of the duty of loyalty arises. Once it established that self-dealing has occurred, no further inquiry of the trustee’s reasonableness or good faith is necessary – per se breach of duty of loyalty.

a. Traditional exceptions – Duty against self-dealing can be waived either by the settlor in the terms of the trust or by all beneficiaries, following a full disclosure of proposed transaction.

b. UTC §802 – Even where the self-dealing is authorized, the transaction must still be reasonable and dair, if it is not, the trustee is liable for breaching duty of loyalty.

3. Hartman v. Hartle – the trustee must administer the trust solely in the interest of the beneficiary4. In re Gleeson’s Will - The executor was the trustee over farmland of the testatrix. The beneficiaries were the children of the

testatrix. The testatrix leased the farmland to the executor and another person as a partnership. They held over for a period of time. The partnership was supposed to pay rent and a partial value of the crops. On appeal, the court reversed. It was undisputed that the executor leased a portion of the real estate to himself as a partner and that the executor received a profit. The executor violated the rule that a trustee could not deal with trust property in an individual capacity and receive a profit. The court rejected the argument that the executor fell within an exception because of his good faith attempt to farm the land after farm tenants could not be found. First, there was no showing that the executor tried to find a satisfactory tenant. Second, the good faith and honesty of the executor even where the trust sustained no loss did not take the executor outside of the rule. Finally, the executor failed to confer with the children concerning the tenancy. Instead, he chose to proceed as a trustee and violated the rule against self-interest.

5. In re Rothko - An abstract expressionist painter whose works through the years gained him an international reputation died testate, and the principal asset of his estate consisted of 798 paintings of tremendous value. Within a period of only three weeks, the three executors had dealt with all of the paintings by virtue of two contracts with galleries in which two of the executors had an interest. The artist's children instituted proceedings to remove the executors, to enjoin the galleries from disposing of the paintings, to rescind the sales agreements, and to recover damages. The trial court, finding for the children, held that the appreciated value of the paintings at the time of trial was the measure of damages for the unreturned paintings. The court held that, due to the conflict of interest of two of the executors between the estate and the galleries, there was a breach of duty and trust to the estate. The third executor breached the trust by failing to exercise ordinary prudence in view of the others' divided loyalties. The use of appreciation damages was proper where estate assets were sold below value as a result of the conflict of interest.

6. Note: Co-Trusteesa. A trustee is liable for a breach of trust if the trustee 1) consents to the action that constitutes the breach, or 2)

negligently fails to act to stop or try to stop the other trustees from engaging in action that constitutes the breach. b. Traditional law – if there is more than one trustee of a private trust, the trustees must act as a group and with

unanimity, unless the trust instrument provides to the contrary. c. UTC §703(a)(2000) and in many state s, a majority can act if there are three or more trustees. However, even if the

unanimity is not required, a co-trustee still has a duty to take reasonable steps to prevent a breach of trust by her co-trustees.

C. THE DUTY OF PRUDENCE 1. Duty of Prudence– the trustee has a duty to administer the trust with such skill and care as a person of ordinary prudence would

use in dealing with his or her property. Duty is an objective standard of care, focusing on what a reasonable person would do, not the trustee’s subjective intentions when acting.

a. Prudent Man Rule – Model and Prudent Man Investment Act abolished the statutory list and permits any investment that a prudent man would make, barring “speculative” investment.

a) The most common statement of the prudent person standard is that the trustee should invest with the same care as a prudent person would use in dealing with his or her property.

b. Note: Problems with the Prudent Man Rule – puts too much emphasis on how risky investments are.2. Prudent Investor Rule And Modern Trust Investment Law

a. Prudent Investor Rulea) Uniform Prudent Investor Act– focuses on the actions that constitute a prudent investor and the duties that

go hand in hand with those actions. Adopts a number of express provisions:a. A trust account's entire investment portfolio is considered when determining the prudence of an

individual investment. Under the Prudent Investor Act standard, a fiduciary would not be held

63 | L e v y

Page 64: Trust and Estates Outline

liable for individual investment losses, so long as the investment, at the time of acquisition, is consistent with the overall portfolio objectives of the account.

b. Diversification is explicitly required as a duty for prudent fiduciary investing.c. No category or type of investment is deemed inherently imprudent. Instead, suitability to the trust

account's purposes and beneficiaries' needs is considered the determinant. As a result, junior lien loans, investments in limited partnerships, derivatives, futures, and similar investment vehicles, are not per se considered imprudent. However, while the fiduciary is now permitted, even encouraged, to develop greater flexibility in overall portfolio management, speculation and outright risk taking is not sanctioned by the rule either, and they remain subject to criticism and possible liability.

d. A fiduciary is permitted to delegate investment management and other functions to third parties.b) Portfolio – Under a prudent investor approach, there is more attention to balancing the portfolio among

investments that are unlikely to move in the same direction in the face of particular events. The insight that purchase of a “risky” stock actually can lower the overall risk to the portfolio means that the traditional, one-investment-at-a-time approach is counterproductive. Because the prudent investor must judge each investment decision in light of the overall portfolio and the needs of the trust, no investment is imprudent per se.

b. Restatement 3rd of Trusts: Chapter 17 (The “Prudent Investor Rule”) – adopted the prudent investor standard.3. Risk, Return And Diversification In Practice

a. In re Estate of Janes - Testator died leaving several trusts, including a marital deduction trust for the benefit of testator's wife. Petitioner executor failed to diversify the trust, which consisted of a large amount of one type of stock. Testator's wife, who was found to be very inexperienced when it came to financial decisions, sought to surcharge petitioner for losses incurred by the estate, during which time the value of the stock had dropped about one-third of its date-of-death value. Objectant personal representatives subsequently took over the cause of action against petitioner upon testator's wife's death. The court affirmed the order of the appeals court which affirmed a surcharge awarded against petitioner because maintaining a concentration of one stock under the circumstances presented violated certain critical obligations of a fiduciary in making investment decisions under the prudent person rule. Petitioner failed to consider the investment in one stock in relation to the entire portfolio. The court affirmed the award established by the appeals court because the correct determination of damages was the amount of the capital lost and not lost profits or market index measure of damages.

a) No precise formula for determining the prudent person standard. Each case turns on its own facts and circumstances, with the trustee’s investment decisions measured in light of the business and economic circumstances at the time they were made in light of the circumstances of the trust itself rather than the integrity of the particular investment.

b. Note: Calculating Damages for Imprudent Investmenta) Charge the trustee with resulting lossb) charge the trustee with any profit madec) charge trustee with any profit that would have accrues but for the breach.

c. Uniform Prudent Investor Act §4 – Duties At Inception of Trusteeship - Within a reasonable time after accepting a trusteeship or receiving trust assets, a trustee shall review the trust assets and make and implement decisions concerning the retention and disposition of assets, in order to bring the trust portfolio into compliance with the purposes, terms, distribution requirements, and other circumstances of the trust, and with the requirements of this act

d. Settlor authorization – if a settlor expressly authorizes all investments, regardless of their legality, the courts tend to construe such provision narrowly, granting a trustee some extra room for lapses in judgment, but not absolute immunity for improper (bad faith or reckless) investments under the prudent investor standard.

e. Wood v. U.S. Bank, N.A. - The trust held stock of the corporate trustee. The trust authorized the trustee to retain its own stock, but it was silent as to the trustee's obligation to diversify. Although the trust beneficiaries did not object to a plan to sell off some stock in order to pay the estate expenses, they later requested that the trustee diversify the holdings, which it did not do. Thereafter, the trustee's corporate stock price plunged, and the distributions to the beneficiaries were made from the decreased assets. The beneficiary filed suit, alleging that the trustee had a mandatory duty under Ohio Rev. Code Ann. § 1339.54 to diversify. The trial court adopted the trustee's abuse of discretion and estoppel instructions, and judgment was entered for the trustee. On appeal, the court held that the abuse of discretion standard was improperly provided to the jury, as an instruction pursuant to § 1339.54(B) was warranted. The power to retain the trustee's own stock was for the purpose of circumventing the rule of undivided loyalty only, and diversification was required absence special circumstances. There was at least enough evidence of special circumstances to avoid a directed verdict.

f. Note: Permissive v. Mandated Retention of Inception Assets and the Duty to Diversify

64 | L e v y

Page 65: Trust and Estates Outline

g. Social Investing

65 | L e v y